THIS IS IT AUDITING FINALllll

Ace your homework & exams now with Quizwiz!

Tracing

(Completeness) Source Document to Journal/Ledger

Vouching

(Occurrence) Journal/Ledger to Source Document

Name 3 of the 5 things that affect the reliability of AIS

1) Security 2) Confidentiality

Assertions about account balances at period end:

1. Existence 2. Completeness 3. Valuation and allocation 4. Rights and obligation

Elements of the professional judgement process:

1. Identify and define the issue 2. Gather the facts and info and identify the relevant literature 3. Perform the analysis and identify potential alternatives 4. Make the decision 5. Review and complete the documentation and rationale for the conclusion

Parts of the Standard unqualified Audit Report (Old)

1. Report title 2. Audit report address 3. Intro paragraph 4. Scope paragraph 5. Opinion paragraph 6. Name of the CPA firm 7. Audit report due

Financial statement cycles:

1. Sales and collection cycle 2. Acquisition and payment cycle 3. Payroll and personnel cycle 4. Inventory and warehousing cycle 5. Capital acquisition and repayment cycle (Look at slides and figure 6-6 on pg. 156)

Departures from an unqualified opinion

1. Scope limitation 2. GAAP departure 3. Auditor not independent

Qualified Opinion examples

1. Scope limitation- Modify scope and opinion and add paragraph before opinion 2. Unjustified departure from GAAP BUT still conclude overall fairly stated overall "except for"- Modify opinion, add paragraph before opinion

Reasons for doubt about going concern

1. Significant recurring operating losses or working capital deficiencies 2. Inability of the company to pay its obligations as they come due 3. Loss of major customers, the occurrence of uninsured catastrophes 4. Legal proceedings, legislation that might jeopardize the entity's ability to operate.

revenue recognition process

1. evidence of an agreement exists 2. delivery occured/ service rendered (risk of loss has passed) 3. price fixed and determinable 4. colectability must be reasonably assured

13. A debit memo A. reduces the amount of accounts payable due to a vendor. B. reduces accounts payable when payment is made. C. is used by vendors to record cash payments received. D. authorizes a debit to purchases when goods are received.

A

18. Unrecorded liabilities are most likely to be found during the review of which of the following documents? A. Unpaid bills. B. Shipping records. C. Bills of lading. D. Unmatched sales invoices.

A

32. For effective internal control, the accounts payable department should compare the information on each vendor's invoice with the A. receiving report and the purchase order. B. receiving report and the voucher. C. vendor's packing slip and the purchase order. D. vendor's packing slip and the voucher.

A

Attributes of business process reengineering methodology include

A process orientation

The purchasing process begins with

A purchase order

Distinguish between a qualified opinion, an adverse opinion, and a disclaimer of opinion, and explain the circumstances under which each is appropriate.

A qualified opinion states that there has been either a limitation on the scope of the audit of material accounts, transactions, or disclosures or a material departure from GAAP in the financial statements, but that the auditor believes that the overall financial statements are fairly presented. This type of opinion may not be used if the auditor believes the exceptions being reported upon are extremely material, in which case a disclaimer or adverse opinion would be used. An adverse opinion states that the auditor believes the overall financial statements are so materially misstated or misleading that they do not present fairly in accordance with GAAP the financial position, results of operations, or cash flows. A disclaimer of opinion states that the auditor has been unable to satisfy himself or herself as to whether or not the overall financial statements are fairly presented because of a significant limitation of the scope of the audit, or a non- independent relationship under the AICPA Code of Professional Conduct between the auditor and the client.

Qualified opinion

A qualified report can result from a limitation on the scope of the audit or failure to follow generally accepted accounting principles

The Sales process ends with

A receipt of cash

Amounts and other data relating to recorded transactions and events have been recorded appropriately

Accuracy

Recorded transactions are stated at the correct amounts

Accuracy

Accounting "ethics" means

Acting responsibly, no matter what

Distinguish between an unmodified opinion audit report that contains an emphasis-of-matter explanatory paragraph and a qualified report.

An unmodified opinion audit report with an explanatory paragraph or modified wording is the same as a standard unmodified opinion report except that the auditor believes it is necessary to provide additional information about the audit or the financial statements. For a qualified report, either there is a scope limitation (condition 1) or a failure to follow generally accepted accounting principles (condition 2). Under either condition, the auditor concludes that the overall financial statements are fairly presented.

Which of the following audit procedures is least likely to detect an unrecorded liability?

Analysis and recomputation of depreciation expense

Calculate the ratio of bad debt expense to credit sales

Analytical procedures

Give examples of specialists whose findings might provide appropriate evidence for independent auditors

Appraisers, attorneys, geologists, environmental consultant

The authority to accept incoming goods in receiving should be based on a(an)

Approved purchase order

Monitoring Activities Deifnition

Asses Quality over time

Measure of the level of certainty that the auditor has obtained at the completion of the audit

Assurance

Population size has no effect for _________ sampling?

Attribute

Audit Procedures for Obtaining Audit Evidence (3 types)

Audit Procedures for Obtaining Audit Evidence: Specific acts performed by the auditor to gather evidence about whether specific assertions are being met. 1. Risk Assessment Procedures 2. Tests of Controls 3. Substantive Procedures

a complete, ready-to-go system of computer hardware and software is also often called

Canned System

Transactions are related to financing the business, such as issuing stock/debt, paying dividends, and repaying debt

Capital acquisition and repayment cycle

Which of the following is not a phase in SDLC

Control

For audit purposes, a corporation's articles of incorporation are normally: Copied and placed on the owners' equity lead schedule. Copied and placed in the permanent file. Confirmed with the transfer agent. Ignored since they are not normally considered to be related to the internal control structure.

Copied and placed in the permanent file.

According to the text, all of the following are objectives of the human resource management process except:

Counseling employees on retirement and medical benefits

The cash disbursements journal is also called the

Check register

Financial and other info is appropriately presented and described and disclosure are clearly expressed

Classification and understandability

Mnemonic Codes

Codes that help the user remember what they represent

23. An auditor performs a test to determine whether all merchandise was received for which the entity was billed. The population for this test consists of all A. merchandise received. B. vendors' invoices. C. canceled checks. D. receiving reports.

B

9. In assessing control risk for purchases, an auditor vouches a sample of entries in the voucher register to the supporting documents. Which assertion would this test of controls most likely support? A. Completeness. B. Occurrence. C. Accuracy. D. Classification.

B. Occurrence.

Accounts payable confirmations are used to test

Both the existence and completeness audit assertions

11. A product cost is A. an expense allocated by a systematic procedure. B. recognized during the period in which a liability is incurred. C. recognized in the period during which related revenue is recognized. D. recognized in the period in which cash is spent.

C

________________ are effectiveness at providing evidence about the existence of accounts, but are less effective at addressing _________________ and valuation

Confirmation completeness

Potential judgment tendencies, traps, and biases:

Confirmation, overconfidence, anchoring, and availability

A change in accounting estimate is an example of an accounting change that affects comparability and requires an explanatory/emphasis-of-matter paragraph in the audit report. True False

False

Bank statement sent directly to the auditor by the bank

Directly from outsiders

Inability to evaluate internal control due to a circumstance-caused scope limitation relating to a significant account in a Sarbanes-Oxley Section 404 internal control audit has what result?

Disclaimer of opinion.

Business process reengineering is not:

Downsizing

What two steps must an auditor do if they have reservations about the audit client continuing as a going concern?

1. Evaluate management's plan to avoid bankruptcy 2. Feasibility of management achieving those plans

_____________ relationship between assertions and objectives, except for the _________- assertion because it has 2 objectives because of the need to provide auditors with guidance in testing transaction accuracy

One-to-one; accuracy

Information Risk

The risk that the information used by investors, creditors, and others to assess business risk is not accurate

Error vs. fraud

Error- unintentional misstatement of financial statements Fraud- intentional misstatement of financial statements

The choice of developing its own system or acquiring a software package from an external vendor is also called this type of decision:

Make-or-buy decision

Implied or expressed representations by management about classes of transactions and the related accounts and disclosures in the financial statements

Management assertions

Business process reeninnering efforts sometimes fail because

Management is too optimistic regarding its expectations from its implementations

For what matter should an auditor obtain written management representation?

Management's compliance with contractual agreements that may affect the financial statements.

If fieldwork of December 31, 2001 financial statements ended on March 6, 2002 and a subsequent event requiring adjustment to financial statements was realized in April for 2002 and the adjustment is made without disclosure, what should the report date be?

March 2, 2002. The date of the end of fieldwork.

Tolerable error

Margin of error. What the aduitor is willing to accept (ex. difference between expected error and materiality)

Divides audit by keeping closely related types/transactions and account balances in the same segment

Financial statement cycle approach

Entitys risk assessment process definition

How company identifies and responds to business risks in relation to achieving objectives.

Misstatements are ____________-- if the combined uncorrected errors and fraud in the financial statements would likely have changed or influence the decisions of a reasonable person using the statements.

Material

In an integrated audit, which of the following lead(s) to an adverse opinion on internal control?

Material weakness. Significant deficiencies do not lead to an adverse opinion.

If an auditor concludes there are contingent liabilities, then he or she must evaluate the: Materiality of the potential liability- Nature of the disclosure to be included in the financial statements-

Materiality of the potential liability-Yes Nature of the disclosure to be included in the financial statements-Yes

Sarbanes Oxley ____________ management's responsibility for financial statements

Increases

The introductory paragraph of an auditor's report contains the following sentences:

Indicate a division of responsibility

Currently, most of the workforce in the U.S. work in the

Information Sector

Paid checks returned with bank statement

Internal but validated externally

What procedure would an auditor most likely perform in obtaining evidence about subsequent events?

Investigate changes in long-term debt occurring after year-end.

1. Scope limitation

Not scope limitation if you can get around it

In testing controls over cash disbursements, an auditor most likely would determine that the person who signs the checks also

Is responsible for mailing the checks

which of the following statements best describes a business process?

It is a collection of activities in an organization that creates value

All of the following are reasons why information technology is important to accountants except:

It is important to keep up; if your competitor buys new computers, you should too

Assets are properly classified

Presentation and disclosure

The components of the financial statements are properly classified, described, and disclosed

Presentation and disclosure

Relates to the presentation and disclosure of info in the financial statements

Presentation and disclosure-related audit objectives

If completeness is a concern for accounts payable, auditors will send accounts payable confirmations to

Primarily vendors with small or zero accounts payable balances

. An auditor may issue the standard audit report when

Principal auditor assumes responsibility for the work of another auditor.

Fixed Asset Management

Process of acquiring, tracking, and disposing of fixed assets

A disposition to inquiry with some sense of doubt

Questioning mindset

With respect to a small company's system of purchasing supplies, an auditor's primary concern should be to obtain satisfaction that supplies ordered and paid for have been

Received, counted, and checked to quantities and amounts on purchase orders and invoices

For effective internal control, the accounts payable department should compare the information on each vendor's invoice with the

Receiving report and the purchase order

To determine whether accounts payable are complete, an auditor performs a test to verify that all merchandise received is recorded. The population of documents for this test consists of all

Receiving reports

A product cost is

Recognized in the period during which related revenue is recognized

An entity erroneously recorded a large purchase twice. Which of the following internal controls would be most likely to detect this error in a timely and efficient manner?

Reconciling vendors' monthly statements with subsidiary payable ledger accounts

When an auditor selects a sample of items for the vouchers payable register for the last month of the period under audit and traces these items to underlying documents, the auditor is gathering evidence primarily in support of the assertion

Recorded obligations were valid

A debit memo

Reduces the amount of accounts payable due to a vendor

When financial statements contain a departure from GAAP because, due to unusual circumstances, the statements would otherwise be misleading, the auditor should explain the unusual circumstances in separate paragraph and express an opinion that

Unqualified

Which of the following is not a procedure normally performed while completing the audit of a public company? Obtain a lawyer's letter. Obtain a representations letter. Perform an overall review using analytical procedures. Update internal control questionnaire.

Update internal control questionnaire.

Important source documents for the human resource management process do not include:

Valid Vendor lists

Assets, liabilities, and equity interests are included in the financial statements at appropriate amounts and any resulting valuation adjustments are appropriately recorded

Valuation and allocation

Assets are recorded at proper amounts

Valuation, allocation, and accuracy

A document that indicates such things as the description and quantity of goods and services received, the price including freight, or a cash discount

Vendor Invoice

Which of the following documents originates outside the company

Vendor Invoice

To provide assurance that each voucher is submitted and paid only once, an auditor most likely would examine a sample of paid vouchers and determine whether each voucher is

Stamped "paid" by the check signer

Transactions that occurred after the balance sheet date, which affect the fair presentation or disclosure of the statements being audited.

Subsequent events

The auditors' search for unrecorded liabilities is completed: During an interim period. At the balance sheet date. Subsequent to the balance sheet date. At any time during the examination.

Subsequent to the balance sheet date.

Auditors evaluate the client's recording of transactions by verifying the monetary amounts of transactions

Substantive tests of transactions

Withholding judgment until appropriate evidence is obtained

Suspension of judgment

Occurence Assertion

The occurrence assertion relates to whether all recorded transactions and events HAVE OCCURRED and pertain to the entity. Ex: all revenue transactions are valid transactions... no BS.

Which of the following is not a difference between the audit report of a nonpublic and public company? The public company report includes the word "Registered" in the title. The public company report refers to standards of the PCAOB. The public company report has an additional paragraph referring to the client's fraud prevention procedures. The public company report is more likely to refer to a critical audit matter.

The public company report has an additional paragraph referring to the client's fraud prevention procedures.

Explain the relationship among the initial assessed control risk, test of controls and substantive tests of transactions for cash receipts, and the test of details of cash balances.

The appropriate tests for the ending balance in the cash accounts depend heavily on the initial assessment of control risk, test of controls, and test of transactions for cash receipts. If the results of the evaluation of internal control, test of controls, and test of transactions are adequate, it is appropriate to reduce the tests of details of balances for cash, especially for the detailed tests of bank reconciliations. If the tests indicate that the client's controls are deficient, extensive year-end testing may be necessary.

Classification Assertion

The classification assertion is concerned with whether transactions and events have been recorded in the proper accounts.

Internal control over bonds payable is best when: The company utilizes the services of a bond trustee. The company segregates approval from issuance of the bonds. Bonds are countersigned by two officers. Bonds are serially numbered.

The company utilizes the services of a bond trustee.

Which of the following procedures would an auditor most likely perform prior to the balance sheet date? Review subsequent events. Perform search for unrecorded liabilities. Send inquiry letter to client's legal counsel. Review detail and test significant travel and entertainment expenses.

Review detail and test significant travel and entertainment expenses.

Scanning (Audit Procedures)

Review of accounting data to identify significant or unusual items.

An internal control questionnaire indicates that an approval receiving report is required to accompany every check request for payment of merchandise. Which of the following procedures provides the greatest assurance that this control is operating effectively?

Select and examine canceled checks and ascertain that the related receiving reports are dated no later than the checks.

Which of the following material events occurring subsequent to the balance sheet date would require an adjustment to the financial statements before they could be issued? Sale of long-term debt or capital stock. Loss of a plant as a result of a flood. Major purchase of a business which is expected to double the sales volume. Settlement of litigation in excess of the recorded liability.

Settlement of litigation in excess of the recorded liability.

Materiality Decisions

Somethings are not quantitatively material. Things such as fraud are always qualitatively material

A lease one ______________ should be included for each _______________-- unless the auditor believes that the general balance-related audit objective is not relevant or is unimportant for the account balance being considered

Specific balance-related audit objective; general balance-related audit objective

_______________________ are applied to each class of transactions but are stated in terms of tailored to a specific class of transactions.

Specific transaction-related audit objectives

Transactions are recorded on the correct dates

Timing

List the 8 parts of the standard unqualified opinion.

Title Client's address Report date introduction scope opinion management's responsibility cpa firm address

What is the purpose of the audit?

To provide financial statement users with an opinion by the auditor on whether the financial statements are presented fairly, in all material respects, in accordance with the applicable financial accounting framework

Within the context of systems studies, a point-scoring system is commonly used:

To rank competing systems when purchasing a system

COntrol environment definition

Tone at the top. Establishes foundation for all other controls.

Operating control over check signing normally should be the responsibility of the: Secretary. Chief accountant. Vice-president of marketing. Treasurer.

Treasurer.

A potential legal claim against a client where the condition for a claim exists but no claim has been filed.

Unasserted claim

Four Categories of Audit Reports under Clarity project

Unmodified opinions: 1. standard unqualified & 2. Unqualified with explanatory paragraph or modified wording Modified Opinions: 3. Qualified & 4. Adverse or Disclaimer

Access control

Who has access to what systems

Which of the following statements best describes the role of materiality in a financial statement audit? a) The lower the level at which the auditor assesses materiality, the greater the amount of evidence the auditor must gather. b) The level of materiality has no bearing on the amount of evidence the auditor must gather. c) Materiality refers to the "material" from which audit evidence is developed. d) The higher the level at which the auditor assesses materiality, the greater the amount of evidence the auditor must gather.

a) the lower the level at which the auditor assesses materiality, the greater the amount of evidence the auditor must gather

14-11 Internal controls which are likely to prevent the client from including as a business expense those transactions that primarily benefit management satisfy the specific audit objective of a. acquisitions are correctly valued. b. existing acquisitions are recorded. c. acquisitions are correctly classified. d. recorded acquisitions are for goods and services received.

c

On April 7, 2012, you discovered that a debtor of Schmidt went bankrupt on January 6, 2012, due to gradual declining financial health.

c. Request the client revise and reissue the December 31, 2011 financial statements. The revision should involve an adjustment to the December 31, 2011 financial statements.

Classification and Understandability (Presentation and Disclosure)

address whether the financial information is appropriately presented and described, and disclosures are clearly expressed.

Estimated transactions

create an account estimate: allowance for doubtful accounts, bad debt expense, goodwill, estimated warranty reserved

What is sampling risk?

possibility that sample drawn is not respresentitive of population

The client _______________ ________________ should never be used as a substitute for performing audit procedures. It is required on every engagement and is dated as of the _____________________________.

presentation letter date of auditors' report

Reperformance (Audit Procedures)

the independent execution by the auditor of procedures or controls that were originally performed by company personnel. --Viewed as highly reliable... auditor is reperforming

Properly prepared audit documentation will help the auditors should litigation arise, just as improperly prepared documentation __________________________

will work against them

14-9 An auditor learns that his client has paid a vendor twice for the same shipment, once based upon the original invoice and once based upon the monthly statement. A control procedure that should have prevented this duplicate payment is a. pre-numbering of receiving reports. b. use of a limit or reasonable test of payments. c. pre-numbering of disbursement vouchers. d. attachment of the receiving report to the disbursement report.

d

Substantive procedures include analytical procedures and tests of ______________________________________________.

details of account balances, transaction, and disclosures

Vouchers in client's unpaid voucher file, credit memo in customer's file. management working papers in making accounting estimates, memo in customer file from treasurer authorizing the write-off of the account

entirely internal

Sampling population

ie all sales invoices for the year

Sampling unit

ie by invoice, by time period, by customer, by price

Define control deviation

ie price on invoice varies form aproved price list

In the audit of the allocation of income for a partnership, the auditors would be most interested in reviewing: Federal Partnership Law. State Partnership Law. Board of directors minutes. The partnership agreement.

The partnership agreement.

Which of these is not a common selection criterion when choosing an AIS?

(all of these) Performance capabilities of each proposed system Compatibility of each proposed system with existing systems Vendor stability and support The cost and benefits of each proposed system

sample size

(book value/ tolerable- expected) x confidence level

The document flowchart symbol for filing is

(triangle)

Sufficiency/Appropriateness of Audit Evidence

***Sufficiency - the measure of the quantity of audit evidence. -Greater risk of misstatement requires higher quantity of audit evidence. -Higher quality audit evidence results in a lower quantity of audit evidence. ***Appropriateness - the measure of the quality of audit evidence. -Relevance: is it relevant? -Reliability: is it reliable? Inverse relationship between sufficiency and appropriateness.... high appropriateness... low sufficiency. Low appropriateness... high sufficiency.

Situations where you would add an emphasis of a matter

**add explanatory paragraph before or after opinion paragraph** 1. related party transactions 2. Subsequent events 3. Comparability issues 4. Footnote disclosures

Unqualified report with explanatory paragraph

1. Lack of consistent application of GAAP 2. Substantial doubt about going concern 3. Auditor agrees with a departure from promulgated accounting principles 4. Emphasis of a matter **1-4 need an extra opinion paragraph** 5. Reports involving other auditors **5- just change the wording**

The 3 options involving other auditors

1. Make no reference in the audit report 2. Make reference in the report (modified wording report) 3. Qualify the opinion

Auditor's ability to detect material misstatements arising from failure to comply with laws and regulations is impacted by the following factors:

1. Many laws and regulations primarily relate to operating aspects of the business and typically don't affect the financial statements and aren't captured by the client's info systems related to financial reporting 2. noncompliance may involve actions to conceal it, such as collusion, forgery, deliberate failure to record transactions, management override of controls, or intentional misrepresentations made to the auditor 3. whether an act constitutes noncompliance is a matter for legal determination, such as by a court of law

Assertions about presentation and disclosure (4):

1. Occurrence and rights and obligations 2. Completeness 3. Classification and understandability 4. Accuracy and Valuation

Elements of professional skepticism:

1. Questioning mindset 2. Suspension of judgment 3. Search for knowledge 4. Interpersonal understanding 5. Autonomy 6. Self-esteem

Matching Audit Procedures to Assertions: Understandability and Classification

1. Inspection of documentation supporting transactions. 2. Review of all related disclosures for compliance with GAAP. 3. Inquiry of management regarding disclosures for the account and for related accounts.

Parts of Modified Audit Report

1. Intro paragraph on F/S 2. MGMT response 3. Auditors response 4. Opinion

Computer-assisted audit techniques

Auditor may perform procedures: )perform calculations and comparisons used in analytical procedures )select a sample of A/c Rec. for confirmation )scan a file to determine all documents in a series have been accounted for )re-performing a variety of calculations )compare data elements in different files for agreement

2. GAAP

Auditor proposed adjustments- client says no-departure from GAAP-yes- then unqualified

An audit report on a public company is least likely to include a paragraph titled: Auditor responsibility Basis for Opinion. Critical Audit Matters. Opinion on the Financial Statements.

Auditor responsibility

14. In assessing control risk for purchases, an auditor vouches a sample of entries in the voucher register to the supporting documents. Which assertion would this test of controls most likely support? A. Completeness. B. Occurrence. C. Accuracy. D. Classification.

B

22. An entity erroneously recorded a large purchase twice. Which of the following internal controls would be most likely to detect this error in a timely and efficient manner? A. Footing the purchases journal. B. Reconciling vendors' monthly statements with subsidiary payable ledger accounts. C. Tracing totals from the purchases journal to the ledger accounts. D. Sending written quarterly confirmations to all vendors.

B

37. With respect to a small company's system of purchasing supplies, an auditor's primary concern should be to obtain satisfaction that supplies ordered and paid for have been A. requested and approved by authorized individuals who have no incompatible duties. B. received, counted, and checked to quantities and amounts on purchase orders and invoices. C. properly recorded as assets and systematically amortized over the estimated useful life of the supplies. D. used in the course of business and solely for business purposes during the year under audit.

B

42. An entity's internal control requires that for every check request there be an approved voucher, supported by a prenumbered purchase order and a prenumbered receiving report. To determine whether checks are being issued for unauthorized expenditures, an auditor most likely would select items for testing from the population of all A. purchase orders. B. canceled checks. C. receiving reports. D. approved vouchers.

B

43. To provide assurance that each voucher is submitted and paid only once, an auditor most likely would examine a sample of paid vouchers and determine whether each voucher is A. supported by a vendor's invoice. B. stamped "paid" by the check signer. C. prenumbered and accounted for. D. approved for authorized purchases.

B

48. Substantive procedures to examine the completeness assertion for accounts payable include A. selecting a sample of vouchers and agreeing them to authorized purchase orders. B. selecting a sample of vouchers and tracing them to the purchases journal. C. comparing dates on vouchers to dates in the purchases journal. D. recomputing the mathematical accuracy of a sample of vendor invoices.

B

54. Which of the following procedures would an auditor least likely perform before the balance sheet date? A. Assessment of inherent risk. B. Observation of merchandise inventory. C. Assessment of control risk. D. Identification of related parties.

B

55. An examination of the balance in the accounts payable account is ordinarily not designed to A. determine that the amounts represent obligations of the company. B. verify that accounts payable were properly authorized. C. ascertain the reasonableness of recorded liabilities. D. determine that all existing liabilities at the balance sheet date have been recorded.

B

57. If completeness is a concern for accounts payable, auditors will send accounts payable confirmations to A. primarily vendors with large accounts payable balances. B. primarily vendors with small or zero accounts payable balances. C. all vendors. D. a random sample of all vendors.

B

67. If payables turnover has increased significantly since the prior year, this is an indication that which of the following assertions for accounts payable might be violated? A. Existence or occurrence. B. Completeness. C. Rights and obligations. D. Valuation and allocation.

B

Recalculation

Checking the mathematical accuracy of records and documents. Involving checking invoices, ledgers, listings are correctly added

Transactions and events have been recorded in proper accounts

Classification

Transactions included in the client's journals are properly classified

Classification

What are closing entries and why are they necessary:

Closing entries are prepared to transfer the balances of nominal accounts to capital (retained earnings) after the adjusting entries have been recorded and the financial statements prepared. Closing entries are necessary to reduce the balances in nominal accounts to zero in order to prepare the accounts for the next period's transactions

List 2 primary objectives of the sales process

Collect cash in a timely and efficient manner

Group Codes

Combining two or more codes to create a new code

Agreements that the entity will hold to a fixed set of conditions, such as the purchase or sale of merchandise at a stated price.

Commitments

What is the distinction between comparability and consistency:

Comparability facilitates comparisons between information about two different enterprises at a particular point in time. Consistency, a type of comparability, facilitates comparisons between information about the same enterprise at two different points in time.

Which of the following procedures would an auditor most likely perform in searching for unrecorded payables?

Compare cash payments occurring after the balance sheet date with the accounts payable trial balance.

Which of the following is an analytical procedure that should be applied to the income statement? Select sales and expense items and trace amounts to related supporting documents. Ascertain that the net income amount in the statement of cash flows agrees with the net income amount in the income statement. Obtain from the proper client representatives, the beginning and ending inventory amounts that were used to determine costs of sales. Compare the actual revenues and expenses with the corresponding figures of the previous year and investigate significant differences.

Compare the actual revenues and expenses with the corresponding figures of the previous year and investigate significant differences.

An auditor traced a sample of purchase orders and the related receiving reports to the purchases journal and the cash disbursements journal. The purpose of this substantive procedure most likely was to

Determine the purchases were properly recorded

Auditor's Decision Process

Determine whether any condition exists requiring a departure from a standard unqualified report 1. Decide the materially for each condition 2. Decide the appropriate type of report 3. Write the audit report

a material weakness has to occur for you to report it to the sec? T/f

F, it only has to be possible

Standard Auditing Procedures = FIVE *CARROT* CARS

FIVE *CARROT* CARS C - Confirmation: obtaining representations from independent third parties. A - Analytical Procedures: evaluations of financial information made by a study of meaningful relationships among data. R - Reperformance: auditor independently performs procedures or controls that were originally performed as part of an entity's internal control. R - Reconciliation: substantiates the existence and valuation of accounts. O - Observation: auditor observes others performing a process or procedure. T - Tracing: starts with source documents and traces forward to provide assurance the the event is being given proper recognition in the books and records.

Standard Auditing Procedures = FIVE CARROT *CARS*

FIVE CARROT *CARS* C - Cut-off Review: review of year end transactions. A - Auditing Related Accounts Simultaneously: ex) long term liabilities and interest expense R - Representation Letter: must obtain from management. S - Subsequent Events Review: auditor must perform certain procedures after the balance sheet date up to the report date.

Specific misstatement in one of a client's 2,000 accounts receivable is referred to as a(n): Multiple Choice Extrapolation difference. Factual misstatement. Redundancy effect misstatement. Projected misstatement.

Factual misstatement.

An opinion based in part on the report of another auditor requires an explanatory/emphasis-of-matter paragraph be added to the standard unqualified audit report. True False

False

Assertions by management are directly related to ______________-- that forms the criteria that management uses to record and disclosure accounting info in financial statements.

Financial reporting framework (US GAAP or IFRS)

information and communication definition

Information is necessary to carry out internal control responsibilties, as is sharing information with others (includes information systems and processes to make sure this happens)

An auditor obtains evidence of stockholders' equity transactions for a publicly traded company by reviewing the entity's: Minutes of board of directors meetings. Registrar's record of interbank transfers. Canceled stock certificates. Treasury stock certificate book.

Minutes of board of directors meetings.

Accounts Receivable Confirmations: Negative Confirmations

Negative confirmations are not as good as positive. Are used for low risk, small balances and expect customer attention. An answer is only requested if the stated amount is incorrect.

When an auditor qualifies an opinion because of inadequate disclosure, the auditor should describe the nature of the omission in a separate explanatory paragraph and modify what?

Opinion paragraph.

PCAOB Form AP involves disclosure of: The engagement partner for the audit. Audit firm's liability for accounts payable to the audit client. Names of all staff members on the audit. Audit client executives.

The engagement partner for the audit.

Assets, liabilities, equity, revenue, and expense components have been included in the financial statements at appropriate amounts

Valuation/allocation

An auditor performs a test to determine whether all merchandise was received for which the entity was billed. The population for this test consists of all

Vendors' invoices

An examination of the balance in the accounts payable account is ordinarily not designed to

Verify that accounts payable were properly authorized

Give two specific examples that demonstrate how this emphasis affects the auditor's evidence accumulation in auditing year-end cash.

Verifying whether cash transactions are properly recorded. Obtaining bank confirmations.

Which of the following procedures would an auditor most likely perform in searching unrecorded liabilities?

Vouch a sample of cash disbursements recorded just after year-end to receiving reports and vendor invoices

Which of the following is a substantive procedure that an auditor most likely would perform to verify the existence of recorded accounts payable?

Vouching selected entries in the accounts payable subsidiary ledger to purchase orders and receiving reports

Observation

Watching/witnessing the performance of some activity or process. It may be safeguarding cash or care in counting inventory.

Audit objectives that must be met for each account balance

Balance-related audit objectives

___________-- are applied to account balances rather than classes of transactions.

Balance-related objectives

______________ are almost always applied to the ending balance in B/S accounts

Balance-related objectives

2. A purchase transaction usually begins with the preparation of a purchase order. True False

FALSE

3. A receiving report is used to document the ordering of goods. True False

FALSE

4. The purchase journal is referred to as a check register. True False

FALSE

Tests of ________ _________ directly address whether there are misstatements in the ending balances of an account

details balances

Accounts Receivable Confirmations: Positive Confirmations

Positive Confirmations are best and are used for large $ accounts, when errors and or disputes are expected and with weak internal control. Usually result in a lower response rate because a greater effect is req'd. If no response send 2nd request then consider alt audit procedures(subsequent receipts).

Which of the following is not a broad category of threat to auditor independence? Familiarity. Positive work relationship. Financial self interest. Undue Influence.

Positive work relationship.

Recorded transactions are properly included in the mater files and are correctly summarized; deals with the accuracy of transfer of info from recorded transactions in journals to subsidiary records and the general ledger

Posting and summarization

COVER-U Assertions relevant to Transactions and events

"COVEU" "C" Completeness: All transactions and events that should have been recorded have been recorded. "O" Cut-Off: Transactions and events have been recorded in the correct accounting period. "V" Valuation, Allocation and Accuracy: Amounts and other data relating to recorded transactions and events have been recorded appropriately. "E" Existence and Occurrence: Transactions and events that have been recorded have occurred and pertain to the entity. "U" Understandability and Classification: Transactions and events have been recorded in the proper accounts.

What is meant by the term "qualitative characteristics of accounting information":

"Qualitative characteristics of accounting information" are those characteristics which contribute to the quality or value of the information. The overriding qualitative characteristic of accounting information is usefulness for decision making.

Audit Procedures for Obtaining Audit Evidence

-Inspection of records or documents -Inspection of tangible assets -Observation -Inquiry -Confirmation -Recalculation -Re-performance -Analytical Procedures -Scanning

What's the definition of fair value:

"the price that would be received to sell an asset or paid to transfer a liability in an orderly transaction between market participants at the measurement date." * Is therefore a market-based measure.

Management estimates the company's allowance for doubtful accounts as $200,000, and the auditors develop an estimate that suggests that the amount should be between $230,000 and $250,000. The factual misstatement in this situation is: $0. $30,000. $40,000. $50,000.

$0.

Statement of Financial Accounting Concepts No. 5 identifies four characteristics that an item must have before it is recognized in the financial statements. What are these four characteristics:

(1) Definitions—The item meets the definition of an element of financial statements. (2) Measurability—It has a relevant attribute measurable with sufficient reliability. (3) Relevance—The information is capable of making a difference in user decisions. (4) Reliability—The information is representationally faithful, verifiable, and neutral.

Expenses, losses, and distributions to owners are all decreases in net assets. What are the distinctions among them:

(1) Distributions to owners differ from expenses and losses in that they represent transfers to owners, and they do not arise from activities intended to produce income. (2) Expenses differ from losses in that they arise from the entity's ongoing major or central operations. (3) Losses arise from peripheral or incidental transactions

Tests of Details: Directional testing

*Vouching* backward to assess the risk of asset and revenue overstatement. Going backward from journal entries to source documents provides evidence of existence, support and occurence *Tracing* forward to assess the risk of expense and liability understatement. Going forward from source documents to journal entries provides evidence of completeness, and coverage.

Name 3 of the 6 parts of an AIS

1) Internal Controls 2) Procedure 3) Software

AICPA and IAASB standards describe 3 categories of assertions:

1. Assertions about classes of transactions and events 2. Assertions about account balances 3. Assertions about presentation and disclosure

Consistency of GAAP- Issues

1. Changes in principle (LIFO to FIFO) **Add explanatory paragraph before or after opinion paragraph** 2. Change in Reporting Entity (acquisition) 3. Error correction involving GAAP change

What are the 5 principles of Control envornment

1. Commitment to integrit and ethical values (Remove incentives to cheat and enforce ethical behavior) 2. Exercise oversight responsibility ( B.o.D. demonstrates independence and exercises oversight 3. Management must establish structure, authority, and responsiblity. 4. Demonstrates a commitment to competence. (right level of experience and right number of people) 5. Enforce Accountability (establish a mechanism to communicate and hold people responsible)

Matching Audit Procedures to Assertions: Existence and Occurrence

1. Confirmation of accounts with third parties. 2. Observation, inspection and examination of assets, processes and procedures. 3. Vouching of transactions from financial statements back to supporting documents.

Matching Audit Procedures to Assertions: Cut-Off

1. Cut-Off procedures to analyze transactions before and after year end for proper accounting period recognition.

Reliability 5 Sources

1. Independent Source outside the entity - Viewed as more reliable than evidence obtained solely from within the entity. 2. Effectiveness of Internal Control: If internal control is considered low risk, evidence generated by that accounting system is viewed as reliable. 3. Auditor's Direct Personal Knowledge: Evidence obtained directly by the auditor is generally considered to be more reliable. ex: auditor's physical inspection of the entity's inventory... considered reliable. 4. Documentary Evidence: more reliable when it exists in documentary form, whether paper, electronic, or other medium. 5. Original Documents: Original documents better than photocopies...

Types of sampling tests

1. Insepction of documents 2. reperformance/recaluation 3.Inspect of tangible assets (inventory) 4. confirmations

What does AICPA auditing standards state the overall objectives of the auditor in conducting an audit of financial statements are?

1. Obtain reasonable assurance about whether the financial statements as a whole are free from material misstatement, whether due to fraud or error, thereby enabling the auditor to express an opinion on whether the financial statements are fairly presented, in all material respects, in accordance with an applicable financial reporting framework 2. report on the financial statements and communicate as required by auditing standards, in accordance with the auditor's findings

Assertions about classes of transactions and events for the period under audit:

1. Occurrence 2. Completeness 3. Accuracy 4. Classification 5. Cutoff

General transaction-related audit objectives:

1. Occurrence 2. Completeness 3. Accuracy 4. Posting and summarization 5. Classification 6. Timing

Assertions about classes of transactions and events for the period under audit (6)

1. Occurrence 2. Completeness 3. Authorization 4. Accuracy 5. Cutoff 6. Classification

Assertions about presentation and disclosure:

1. Occurrence and rights and obligations 2. Completeness 3. Accuracy and valuation 4. Classification and understandability

What are the three required conditions for a contingent liability to exist?

1. There is potential for future payment to an outside party or the impairment of an asset that resulted from an existing condition. 2. There is uncertainty about the amount of the future payment or impairment. 3. The outcome will be resolved by some future event or events.

Matching Audit Procedures to Assertions: Completeness

1. Tracing of transactions forward starting from source documents through their account recognition to financial statements. 2. Analytical review procedures: consider how certain items might be omitted from the account balance. 3. Observation of processes and procedures.

Sampling steps

1. determine your test objective (what do we want to accomplish)ie see if client uses an approved price liest for sales invoices 2. define population characteristics (define sampling population, define sampling unit, define control deviation) 3. select a sample size (use 3 important factors for sampling) 4. select sample items (use random number generator or systematic selection, ie use every 100th item) 5. perform audit procedures (issues with voided docuemnts. understand what it was and if it was authorized. if several errors, stop!) 6. calculate sample deviation and upper deviation rate 7. draw final conclusion

a sample is non statistical if

1. judgement is used to determine sample size. 2. haphazard sample selection process 3. sample results evalauted judgementally If any of these three apply, the testing is non statistical

Why are auditors responsible for reasonable assurance but not absolute?

1. most audit evidence results from testing a sample of a population 2. accounting presentations contain many complex estimates, which inherently involve uncertainty and can be affected by future events 3. fraudulently prepared financial statements are often extremely difficult, if not impossible, for the auditor to detect, especially when there is collusion among management

Risk assessment procedures include the following:

1. obtain an understanding of the entity and its environment 2. understand internal control and assess control risk 3. assess risk of material misstatement

4 phases of the audit process:

1. plan and design an audit approach 2. perform tests of controls and substantive tests of transactions 3. perform substantive analytical procedures and tests of details of balances 4. complete the audit and issue an audit report

2 overriding considerations affect the approach the auditor selects:

1. sufficient appropriate evidence must be accumulated to meet the auditor's professional responsibility 2. the cost of accumulating the evidence should be minimized

Block Code

A chart of accounts is a good example for this code

The sales process begins

A customer order for goods or services

An Important input to the sales process

A customer purchase order

Describe what is meant by a cutoff bank statement and state its purposes.

A cutoff bank statement is a partial period bank statement with the related cancelled checks, duplicate deposit slips, and other documents included in bank statements, which is mailed by the bank directly to the auditor. The bank may provide direct electronic access to the auditor to allow the auditor to review transactions through a certain date. The purpose of the cutoff bank statement is to verify the reconciling items on the client's year-end reconciliation with evidence that is inaccessible by the client.

When an auditor expresses an adverse opinion, the opinion paragraph should include

A direct reference to a separate paragraph disclosing the basis for the opinion

Which of the following describes a permanent difference?

A fundamental difference in what constitutes revenue or expense for GAAP and tax purposes

A registrar/transfer agent system relating to capital stock is most likely used by: A small, nonpublic company. A large, publicly traded company. All companies must use this type of system. No companies use this system anymore.

A large, publicly traded company.

The refusal of a client's attorney to provide information requested in an inquiry letter generally is considered what?

A limitation on the scope of the audit.

Define materiality as it is used in audit reporting. What conditions will affect the auditor's determination of materiality?

A misstatement in the financial statements can be considered material if knowledge of the misstatement would affect a decision of a reasonable user of the statements. Conditions that affect the auditor's determination of materiality include: Potential users of the financial statements, Dollar amounts of the following items: net income before taxes, total assets, current assets, current liabilities, and owners' equity, Nature of the potential misstatements-certain misstatements, such as fraud, are likely to be more important to users of the financial statements than other misstatements.

When an adverse opinion is expressed on the financial statements of a nonpublic company, the opinion paragraph should include a direct reference to: A note to the financial statements which discusses the basis for the opinion. The auditor's responsibility section of the audit report which discusses the basis for the opinion rendered. A separate basis for modification paragraph (section). The consistency in the application of generally accepted accounting principles.

A separate basis for modification paragraph (section).

Communications between the auditor and those charged with governance should include all of the following except A. Major issues discussed with management before the auditor was retained B. Significant audit adjustments C. Consultations with other accountants D. A summary of specific audit procedures used

A summary of specific audit procedures used

Which of the following is not a general objective when designing an AIS

A system should have sufficient capacity to accommodate levels of normal activity; any additional capacity proves too costly in the long run

Which of the following describes a temporary difference?

A timing difference between the recognition of revenue or expense under GAAP and tax purposes

The auditor has completed her assessment of subsequent events. The PROPER accounting for subsequent events that have a DIRECT EFFECT on the financial statements is to: A) adjust the financial statements for the year under audit. B) disclose in the notes to financial statement the amount of the adjustment. C) duly note in the audit workpapers that next year's financial statements need to be adjusted. D) make no adjustment of the financial statements for the year under audit

A) ADJUST the financial statements for the year under audit.

21. Substantive procedures to examine the occurrence assertion for accounts payable include A. Selecting a sample of vouchers and agreeing them to authorized purchase orders. B. Selecting a sample of vouchers and tracing them to the purchases journal. C. Comparing dates on vouchers to dates in the purchases journal. D. Recomputing the mathematical accuracy of a sample of vendor invoices.

A. Selecting a sample of vouchers and agreeing them to authorized purchase orders.

Most AISs perform all of these functions except

AISs perform all of these functions: collect raw accounting data, store data for future use, process data into information

Describe the major constraint inherent in the presentation of accounting information:

Accounting information is subject to the cost constraint. Information is not worth providing unless the benefits exceed the costs of preparing it.

Valuation and Allocation (Period End/Balance Sheet)

Address whether assets, liabilities, and equity interests included in the financial statements are at appropriate amounts and any resulting valuation or allocation adjustments are appropriately recorded.

What are adjusting entries and why are they necessary:

Adjusting entries are prepared prior to the preparation of financial statements in order to bring the accounts up to date and are necessary: (1) to achieve a proper recognition of revenues and expenses in measuring income and (2) to achieve an accurate presentation of assets, liabilities and stockholders' equity.

Is it necessary that a trial balance be taken periodically, What purpose does it serve:

Although it is not absolutely necessary that a trial balance be taken periodically, it is customary and desirable. The trial balance accomplishes two principal purposes: (1) It tests the accuracy of the entries in that it proves that debits and credits of an equal amount are in the ledger. (2) It provides a list of ledger accounts and their balances which may be used in preparing the financial statements and in supplying financial data about the concern.

What should the auditor understand in regards to the use of management's written representations as audit evidence about the completeness assertion?

An auditor should understand that such representations complement, but do not replace, substantive tests designed to support the assertion.

A client is a defendant in a patent infringement lawsuit against a major competitor. What would least likely be included in the attorney's response to the auditor's letter of inquiry?

An evaluation of the ability of the client to continue as a going concern if the verdict is unfavorable and maximum damages are awarded.

An example of a Type I subsequent event is A. The entity's Board of Directors unexpectedly resigns after the balance sheet date B. Notification of an IRS audit after the balance sheet date C. A tornado that destroys an entity's factory after the balance sheet date D. An event after the balance sheet date that confirms the auditor's belief (documented prior to the end of the entity's fiscal year) that a large portion of the entity's inventory is obsolete

An event after the balance sheet date that confirms the auditor's belief (documented prior to the end of the entity's fiscal year) that a large portion of the entity's inventory is obsolete

44. Which of the following is the most effective control activity to detect vouchers prepared for the payment of goods that were not received? A. Counting of goods upon receipt in the storeroom. B. Matching of purchase order, receiving report, and vendor invoice for each voucher in the accounts payable department. C. Comparison of goods received with goods requisitioned in the receiving department. D. Verification of vouchers for accuracy and approval in the internal audit department.

B

Subsequent events affecting the REALIZATION of assets ordinarily will require adjustments of the financial statements under examination because such events typically represent: A) the culmination of conditions that existed at the balance sheet date. B) additional new information related to events that were in existence on the balance sheet date. C) final estimates of losses relating to casualties occurring in the subsequent events period. D) preliminary estimate of losses relating to new events that occurred subsequent to the balance sheet date

B) ADDITIONAL new information related to events that were in existence on the balance sheet date.

52. Which of the following is a substantive procedure that an auditor most likely would perform to verify the existence of recorded accounts payable? A. Investigating the open purchase order file to ascertain that prenumbered purchase orders are used and accounted for. B. Receiving the entity's mail, unopened, for a reasonable period of time after the year-end to search for unrecorded vendor's invoices. C. Vouching selected entries in the accounts payable subsidiary ledger to purchase orders and receiving reports. D. Confirming accounts payable balances with known suppliers who have zero balances.

C

58. In auditing accounts payable, an auditor's procedures most likely would focus primarily on management's assertion of A. existence. B. rights and obligations. C. completeness. D. valuation and allocation.

C

Which of the following statements is correct? A) A letter of representation is documentation of management's acceptance of responsibility for the financial statements and is deemed to be reliable evidence. B) A letter of representation is not deemed to be reliable evidence because of the potential incompetence of management. C) A letter of representation is not deemed to be reliable evidence because of the lack of independence of the preparers. D) A letter of representation is documentation of the CPA's acceptance of responsibility for the audit of the financial statement and is deemed to be reliable.

C) A letter of representation is not deemed to be reliable evidence because of the lack of independence of the preparers.

Which event that occurred after the end of the fiscal year under audit but prior to issuance of the auditor's report would NOT require disclosure in the financial statements? A) Sale of a bond or capital stock issue B) Loss of plant or inventories as a result of fire or flood C) A significant decline in the market price of the corporation's stock D) A merger or acquisition

C) A significant decline in the market price of the corporation's stock

When searching for unrecorded liabilities at year-end, the population identified for sample would be

Cash disbursements recorded in the period subsequent to year-end

What following procedures would an auditor most likely perform in obtaining evidence about subsequent events?

Compare the latest available interim financial information with the financial statements being reported upon.

All assets have been recorded

Completeness

Tests designed to detect purcahses made before the end of the year that have been recorded in the subsepquent year most likely would provide assurance about management's assertion of

Cutoff

The transactions are recorded in the correct accounting period

Cutoff

If the auditor concludes that there are contingent liabilities, he or she must evaluate the significance of the potential liability and the nature of the disclosure needed in the financial statements. Which of the following statements is not true? A) The potential liability is sufficiently well known in some instances to be included in the financial statements as an actual liability. B) Disclosure may be unnecessary if the contingency is highly remote or immaterial. C) Frequently, the CPA firm obtains a separate evaluation of the potential liability from its own legal counsel rather than relying on management or management's attorneys. D) Answers B and C are correct, but answer A is not.

D) Answers B and C are correct, but answer A is not.

32. Which of the following questions would most likely be included in an internal control questionnaire concerning the completeness assertion for purchases? A. Is an authorized purchase order required before the receiving department can accept a shipment or the vouchers payable department can record a voucher? B. Are purchase requisitions prenumbered and independently matched with vendor invoices? C. Is the unpaid voucher file periodically reconciled with inventory records by an employee who does not have access to purchase requisitions? D. Are purchase orders, receiving reports, and vouchers prenumbered and periodically accounted for?

D. Are purchase orders, receiving reports, and vouchers prenumbered and periodically accounted for?

22. Which account is LEAST likely to be affected by purchase transactions? A. Accounts payable. B. Inventory. C. Cost of goods sold. D. Freight Expense.

D. Freight Expense.

System Flowcharts

Describes the logical flow of data within a computer system and how decisions are made to control events

Details in the account balance agree with related master file amounts, foot to the total in the account balance, and agree with the total in the general ledger; objective is concerned that the details on lists are accurately prepared, correctly added, and agree with the general ledger

Detail tie-in

The confirmation of accounts payable is most closely associated with: Assertion risk. Detection risk. Inherent risk. Relative risk.

Detection risk.

A common audit procedure in the audit of payroll transactions involves tracing selected items from the payroll journal to employee time cards that have been approved by supervisory personnel. This procedure is designed to provide evidence in support of the audit proposition that: Only bonafide employees worked and their pay was properly computed. Jobs on which employees worked were charged with the appropriate labor cost. Internal control relating to payroll disbursements are operating effectively. Employees worked the number of hours for which their pay was computed.

Employees worked the number of hours for which their pay was computed.

For fraud, missappropriation of assets is usually committed by __________- while fraudulent financial reporting is usually committed by _____________.

Employees; management

Management must issue an assertion on whether or not icfr is effective for the end of the fiscal year

End of year time frame allows for management to correct deficiency

The client changed from FIFO to LIFO inventory valuation in the current year and reflected this change in their financial statements. How should this be reflected in the auditor's report?

Even though this change has been reflected in the financial statements, a separate explanatory paragraph is required to explain the change in generally accepted accounting principles in the first year in which the change took place.

Assets, liabilities and equity interests exist

Existence

List and define the assertions about classes of transactions and events for the period under audit.

Existence Assets, liabilities, and equity interests exist. Rights and Obligations:The entity holds or controls the rights to assets, and liabilities are the obligations of the entity. Completeness: All assets, liabilities, and equity interests that should have been recorded have been recorded. Valuation and Allocation: Assets, liabilities, and equity interests are included in the financial statements at appropriate amounts and any resulting valuation or allocation adjustments are recorded appropriately.

There is such an asset

Existence and occurrence

Assets or liabilities of the public company exist at a given date, and recorded transactions have occurred during the period

Existence or occurrence

Assertions about account balances at the period end include

Existence, rights and obligations, and completeness

You Can do control reliance testing without doing substantive testing? T/F

F

Differentiate between "financial statements" and "financial reporting.":

Financial statements generally refer to the four basic financial statements: balance sheet, income statement, statement of cash flows, and statement of changes in owners' or stockholders' equity. Financial reporting is a broader concept; it includes the basic financial statements and any other means of communicating financial and economic data to interested external parties. Examples of financial reporting other than financial statements are annual reports, prospectuses, reports filed with the government, news releases, management forecasts or plans, and descriptions of an enterprise's social or environmental impact.

Tracing

Following a transaction from a source document to recorded entries (completeness)

Be alert for info that either confirms or contradicts facts/info, in addition to evaluating the sources of that info

Gather the facts and info and identify the relevant literature

Expected Error

How many problems you (the auditor) anticipate. How to calculate? talk to client, past auditrs, or industry averages.

Give one example in which the conclusions reached about internal controls in cash disbursement will affect the tests of cash balances.

If controls over the issuance of blank checks, the review of payees, amounts, and supporting documentation, the signing of checks, and the reconciliation of bank statements and vendors' statements are adequate, the auditor's review of outstanding checks on the year-end bank reconciliation may be greatly reduced. The year-end outstanding checks can be verified by testing a sample of checks returned with the cutoff bank statement rather than tracing all paid outstanding checks and the final monthly checks in the cash disbursement journal to the last month's cleared checks and the bank reconciliation.

What are the two components of risk of material misstatements?

Inherent risk x control risk

What procedure would an auditor most likely perform to obtain evidence about the occurrence of subsequent events?

Inquiring as to whether any unusual adjustments were made after year-end

Which of the following procedures most likely would assist an auditor in identifying conditions and events that may indicate substantial doubt about an entity's ability to continue as a going concern? Performing cutoff tests of sales transactions with customers with long-standing receivable balances. Evaluating the entity's procedures for identifying and recording related party transactions. Inspecting title documents to verify whether any real property is pledged as collateral. Inquiring of the entity's legal counsel about litigation, claims, and assessments.

Inquiring of the entity's legal counsel about litigation, claims, and assessments.

Tracing

Inspecting documents created when transactions were executed and determines that information from the documents was properly recorded in accounting records.

Scanning

Less careful scrutiny of documents and records. Inspect tangible resources so auditor has personal knoweledge

A written communication from the client to the auditor formalizing statements that the client has made about matters pertinent to the audit.

Letter of representation

This best explains why we have incomplete information on cybercrime

Most cybercrime is not cauht

Hieracrchy of audit evidence

Most reliable to least reliable: A - Auditor's direct personal knowledge E - External evidence I - Internal evidence O - Oral evidence U won't forget it!

Which of the following is true (regarding AIS)

NONE of these

Which of the following manipulations would understate accounts payable on the financial statements? Overstatement of purchases. Closing the cash disbursements journal prior to year-end. Leaving the cash receipts journal open after year-end. Omission of expenses

Omission of expenses

Which type of data flow diagram illustrates the internal and external entities who are participating in a given system

Physical DFD

Audit must be planned/performed with an attitude of ________________ in all aspects of the engagement, recognizing the possibility that a material misstatement could exist regardless of the auditor's prior experience with the integrity and honestly of client management and those charged with governance.

Professional skepticism

Contingency fee based pricing of accounting services is: Always strictly prohibited in public accounting practice. Never restricted in public accounting practice. Prohibited for clients for whom attestation services are provided. Considered an act discreditable to the profession.

Prohibited for clients for whom attestation services are provided.

Reperformance

Providing that accuracy of a client-performed activity

All of these are terms normally associated with data flow diagrams except:

Pseudo DFD

Which of the following is an authoritative body that the AICPA Code of Professional Conduct authorizes to promulgate auditing and attestation standards? Auditing Standards Commission. Financial Accounting Standards Board. International Accounting Standards Board. Public Company Accounting Oversight Board.

Public Company Accounting Oversight Board.

Briefly describe the two fundamental qualities of useful accounting information:

Relevance and faithful representation are the two primary qualities of useful accounting information. (1) For information to be relevant, it should should be capable of making a difference in a decision by helping users to form predictions about the outcomes of past, present, and future events or to confirm or correct expectations. (2) Faithful representation of a measure rests on whether the numbers and descriptions match what really existed or happened

Relevance and Reliability (Appropriateness of Audit Evidence)

Relevance of audit evidence refers to its relationship to the assertion being tested. Reliability of evidence refers to whether a particular type of evidence can be relied upon to signal the TRUE state of an assertion.

Ues of Analytical Procedures

Req'd for planning and overall review of audit and can be use as substantive testing but is NOT req'd.

which of these is a disadvantage of prototyping?

Requires an iterative testing process that takes some time and patience

What are management's responsibilities?

Responsibility for adopting sound accounting policies, maintaining adequate internal control, and making fair representations in the financial statements

Why are revenue and expense accounts called temporary or nominal accounts:

Revenue and expense accounts are referred to as temporary or nominal accounts because each period they are closed out to Income Summary in the closing process. Their balances are reduced to zero at the end of the accounting period; therefore, the term temporary or nominal is given to these accounts

Which of the following audit procedures is best for identifying unrecorded trade accounts payable? Reviewing cash disbursements recorded subsequent to the balance sheet date to determine whether the related payable applies to the prior period. Investigating payables recorded just prior to and just subsequent to the balance sheet date to determine whether they are supported by receiving reports. Examining unusual relationships between monthly accounts payable balances and recorded cash payments. Reconciling vendors' statements to the file of receiving reports to identify items received just prior to the balance sheet date.

Reviewing cash disbursements recorded subsequent to the balance sheet date to determine whether the related payable applies to the prior period.

The company legally owns the asset

Rights and Obligations

What is Rule 203 of the Code of Professional Conduct:

Rule 203 of the Code of Professional Conduct prohibits a member of the AICPA from expressing an opinion that financial statements conform with GAAP if those statements contain a material departure from an accounting principle promulgated by the FASB, or its predecessors, the APB and the CAP, unless the member can demonstrate that because of unusual circumstances the financial statements would otherwise have been misleading. Failure to follow Rule 203 can lead to a loss of a CPA's license to practice. This rule is extremely important because it requires auditors to follow FASB standards

who do you report to if you find a material weakness

SEC

Which of the following is not a broad category of safeguards that mitigate or eliminate threats to independence? Safeguards created by the profession, legislation, or regulation. Safeguards created to assure proper training within both the client and attest environment. Safeguards implemented by the attest client. Safeguards implemented by the firm, including policies and procedures to implement professional and regulatory requirements.

Safeguards created to assure proper training within both the client and attest environment.

The mailing of disbursement checks and remittance advices should be controlled by the employee who

Signed the checks last

In a PERT network diagram, the amount of delay time that can occur in a non-critical activity is called the

Slack time

Tolerable must exceed expected error T/F

T

You Can do substantive testing without doing control reliance strategy? T/F

T

Which of the following test(s) of details of transactions can be used as a dual-purpose test in conjunction with tests of controls

Test a sample of purchase requisitions for proper authorization

Which of the following best describes the auditors' approach to the audit of accrued liabilities? Test computations. Confirmation. Observation. A low planned assessed level of control risk.

Test computations.

Auditors often integrate procedures for presentation and disclosure objectives with: Tests for planning objectives- Tests for balance-related objectives-

Tests for planning objectives-No Tests for balance-related objectives-Yes

Determine whether disbursements are properly approved

Tests of controls

Which of the following organizations issue international ethics standards for auditors? The AICPA. The IFAC. The SEC. The FASB.

The IFAC.

The AICPA regularly conducts a voluntary survey to identify a list of important IT trends or factors is called

The Top Ten Technologies

What has been the effect on accountants in those organizations which have computerized their data processing functions?

The accountants have become involved in more decision-making activities

Auditors must communicate internal control "significant deficiencies" to: The audit committee. The shareholders. The SEC. The Federal Trade Commission.

The audit committee.

Distinguish between the component of audit risk that the auditors gather evidence to ASSESS vs. the component of audit risk that they collect evidence to RESTRICT

The auditor ASSESS IR and CR The auditor RESTRICTS DR

In which of the following circumstances would a covered member be considered independent when performing the audit of the financial statements of a new client for the year ended December 31, 20X3? The covered member resigned on January 17, 20X3 from the board of directors of the client, prior to accepting the new audit engagement. The covered member continues to hold an immaterial indirect financial interest in the client. The covered member continues to serve as a trustee for the client's pension plan and has the authority to make investment decisions. The covered member's spouse owns an immaterial amount of shares of common stock in the client.

The covered member continues to hold an immaterial indirect financial interest in the client.

Cutoff Assertion

The cutoff assertion relates to whether transactions and events have been recorded in the correct accounting period. Ex: the auditor may want to test proper cutoff of revenue transactions at Dec 31, 2015...

What are the four basic assumptions that underlie the financial accounting structure:

The four basic assumptions that underlie the financial accounting structure are: (1) An economic entity assumption. (2) A going concern assumption. (3) A monetary unit assumption. (4) A periodicity assumption

An auditor includes a separate paragraph in an otherwise unmodified report to emphasize that the entity being reported on had significant transactions with related parties.

The inclusion of this separate paragraph is appropriate and would not negate the unqualified opinion.

What are the purposes of the opinion paragraph in the auditor's report? Identify the most important information included in the opinion paragraph

The purpose of the opinion paragraph is to state the auditor's conclusions based upon the results of the audit evidence. The most important information in the opinion paragraph includes: 1. The words "in our opinion," which indicate that the conclusions are based on professional judgment. 2. A statement about whether the financial statements were presented fairly and in accordance with generally accepted accounting principles along with indication of the fiscal year(s) associated with those statements.

In which of the following circumstances would an auditor of financial statements be most likely to express an adverse opinion? The statements are not in conformity with FASB requirements regarding goodwill impairment. Information comes to the auditor's attention that raises substantial doubt about the entity's ability to continue in existence. The chief executive officer refuses the auditor access to minutes of board of directors' meetings. Tests of controls show that the entity's internal control is so poor that it cannot be relied upon.

The statements are not in conformity with FASB requirements regarding goodwill impairment.

Sequence of events and documents

The top of the house / \ Financial statements Trial Balance General Ledger Subsidiary ledger Books of Original Entry Source Documents Execution of event Transaction Approved ------------------------------- Floor of the house

Tests of controls for the occurrence assertion for purchases include all of the following except:

Tracing a sample of vouchers to purchases journal.

The term "except for" in an audit report is: Used in an adverse opinion. No longer considered appropriate. Used in a qualified opinion. Used for an unmodified opinion when an emphasis-of-matter paragraph is added.

Used in a qualified opinion.

Decision Table

Used to model complex rule sets and their corresponding actions

Observation (Audit Procedures)

Watching a process or procedure being performed by others.

Completeness Assertion (Balance Sheet/Period End)

Whether all assets, liabilities, and equity interests that should have been included as ending balances on the financial statements have been included.

The extent to which the auditor will use the results of analytical procedures to reduce detailed tests depends on what?

Whether it supports the correctness of the recorded account balance (precision) and the effectiveness of analytical procedure

3. Auditor not independent

Will result in disclaimer for audit (drop opinion)- may have to withdraw prior filings with clients.

14-2 When goods are received, the receiving clerk should match the goods with a. The purchase order and the requisition form. b. The vendor invoice and the receiving report. c. The vendor shipping document and the purchase order. d. The receiving report and the vendor shipping document.

a

14-47 Which of the following procedures would an auditor least likely perform before the balance sheet date? a. confirmation of accounts payable. b. observation of physical inventory count. c. confirmation of accounts receivable. d. identification of related parties transactions.

a

Which of the following characteristics most likely would heighten an auditor's concern about the risk of intentional manipulation of financial statements? a) Management places substantial emphasis on meeting earnings projections. b) The rate of change in the entity's industry is slow. c) Turnover of senior accounting personnel is low. d) Insiders recently purchased additional shares of the entity's stock.

a) Management places substantial emphasis on meeting earnings projections.

Which of the following is correct regarding the types of audits over which the ASB and the PCAOB, respectively, have standard-setting authority in the United States? ASB PCAOB a) Nonpublic company audits Public company audits b) Public company audits Public company audits c) Nonpublic company audits Nonpublic company audits d) Public company audits Nonpublic company audits

a) Nonpublic company audits Public company audits

Before accepting an audit engagement, a successor auditor should make specific inquiries of the predecessor auditor regarding the predecessor's a) understanding as to the reasons for the change of auditors. b) opinion of any subsequent events occurring since the predecessor's audit report was issued. c) awareness of the consistency in the application of generally accepted accounting principles between periods. d) evaluation of all matters of continuing accounting significance.

a) understanding as to the reasons for the change of auditors.

An entity changed from the​ straight-line method to the​ declining-balance method of depreciation for all newly acquired assets. This change has no material effect on the current​ year's financial statements but is reasonably certain to have a substantial effect in later years. If the change is disclosed in the notes to the financial​ statements, the auditor should issue a report with​ a(n) a) unmodified opinion b) unmodified opinion with explanatory paragraph c) qualified opinion with explanatory paragraph regarding consistency d) qualified opinion

a) unmodified opinion

A public entity changed from the straight-line method to the declining balance method of depreciation for all newly acquired assets. This change has no material effect on the current year's financial statements but is reasonably certain to have a substantial effect in later years. The client's financial statements contain no material misstatements and the auditor concurs with this change. If the change is disclosed in the notes to the financial statements, the auditor should issue a report with a(n): a) unqualified opinion. b) explanatory paragraph. c) consistency modification. d) "except for" qualified opinion.

a) unqualified opinion.

On February 19, 2008, Treble settled a lawsuit out of court that had originated in 2002 and is currently listed as a contingent liability.

a. Adjust the December 31, 2007 financial statements.

Occurrence and Rights and Obligations (Presentation and Disclosure)

address whether disclosed events, transactions, and other matters have occurred and pertain to the entity.

Accuracy and valuation (Presentation and Disclosure)

address whether financial and other information is disclosed fairly and in appropriate amounts.

Why do auditors generally use a sampling approach to evidence gathering? a) Auditors are experts and do not need to look at much to know whether the financial statements are correct or not. b) Auditors must balance the cost of the audit with the need for precision. c) Auditors must limit their exposure to their auditee to maintain independence. d) The auditor's relationship with the auditee is generally adversarial, so the auditor will not have access to all of the financial information of the company.

b) Auditors must balance the cost of the audit with the need for precision.

Which of the following would be considered a change that affects consistency? a) Change in accounting estimate. b) Change in accounting principle. c) Change in classification and reclassification. d) All of the other options are correct.

b) Change in accounting principle.

In which of the following situations would an auditor ordinarily issue an unqualified/unmodified financial statement audit opinion with no explanatory (or emphasis-of-matter/other-matter) paragraph? a) The entity issues financial statements that present financial position and results of operations but omits the statement of cash flows. b) The auditor decides to refer to the report of another auditor as a basis, in part, for the auditor's opinion. c) The auditor wishes to emphasize that the entity had significant related-party transactions. d) The auditor has substantial doubt about the entity's ability to continue as a going concern, but the circumstances are fully disclosed in the financial statements.

b) The auditor decides to refer to the report of another auditor as a basis, in part, for the auditor's opinion.

Which of the following would an auditor most likely use in determining overall materiality when planning the audit? a) The anticipated sample size of the planned substantive tests. b) The entity's income before taxes for the period-to-date (e.g., 6 months). c) The results of tests of controls. d) The contents of the engagement letter.

b) The entity's income before taxes for the period-to-date (e.g., 6 months).

The​ auditor's report contains the​ following: "We did not audit the financial statements of​ EZ, Inc., a wholly owned​ subsidiary, which statements reflect total assets and revenues constituting 27 percent and 29​ percent, respectively, of the consolidated totals. Those statements were audited by other auditors whose report has been furnished to​ us, and our​ opinion, insofar as it relates to the amounts included for​ EZ, Inc., is based solely on the report of the other​ auditors." These sentences: a) assume responsibility for the other auditor b) indicate a division of responsibility c) are an improper form of reporting d) require a departure from an unmodified opinion

b) indicate a division of responsibility

An auditor includes a separate paragraph in an otherwise unmodified financial statement audit report to emphasize that the entity being reported upon had significant transactions with related parties. The inclusion of this separate paragraph: a) necessitates a revision of the opinion paragraph to include the phrase "with the foregoing explanation." b) is appropriate and would not negate the unmodified opinion. c) violates generally accepted auditing standards if this information is already disclosed in footnotes to the financial statements. d) is considered an "except for" qualification of the opinion.

b) is appropriate and would not negate the unmodified opinion.

Tolerable misstatement is a) the amount of misstatement that management is willing to tolerate in the financial statements. b) materiality used to establish a scope for the audit procedures for the individual account balance or disclosures. c) materiality for the balance sheet as a whole. d) materiality for the income statement as a whole.

b) materiality used to establish a scope for the audit procedures for the individual account balance or disclosures.

Which of the following would not be considered to be an analytical procedures? a) developing the expected current year sales based on the sales trend of the prior five years b) projecting the error rate by comparing the results of a statistical sample with the actual population characteristics c) estimating payroll expense by multiplying the number of employees by the average hourly wage rate and the total hours worked d) computing accounts receivable turnover by dividing credit sales by the average net receivables

b) projecting the error rate by comparing the results of a statistical sample with the actual population characteristics

A client has used an inappropriate method of accounting for its pension liability on the balance sheet. The resulting misstatement is material, but the auditor does not consider its effect to be pervasive. The auditor is unable to convince the client to alter its accounting treatment. The rest of the financial statements are fairly stated in the auditor's opinion. Which kind of audit report would an auditor most likely issue under these circumstances? a) Standard unqualified opinion. b) Qualified opinion due to departure from GAAP. c) Adverse opinion. d) No opinion at all.

b) qualified opinion due to departure from GAAP

King, CPA, was engaged to audit the financial statements of Chang Company, a private company, after its fiscal year had ended. King neither observed the inventory count nor confirmed the receivables by direct communication with debtors but was satisfied that both were fairly stated after applying appropriate alternative procedures. King's financial statement audit report most likely contained a(n): a) disclaimer of opinion. b) unmodified opinion. c) unmodified opinion with an emphasis-of-matter paragraph. d) qualified opinion.

b) unmodified opinion.

On February 2, 2008, you discovered an uninsured lawsuit against Treble that had originated on August 30, 2007.

b. Disclose the information in a footnote in the December 31, 2007 financial statements.

On January 16, 2008, a lawsuit was filed against Treble for a patent infringement action that allegedly took place in early 2005. In the opinion of Treble's attorneys, there is a reasonable (but not probable) danger of a significant loss to Treble.

b. Disclose the information in a footnote in the December 31, 2007 financial statements.

In the context of agency theory, information asymmetry refers to the idea that a) Information can vary in its reliability. b) Information can vary in its relevance. c) Management has more information about the entity's true financial position than do the absentee owners (i.e. stockholders). d) Management likely will not act in the best interests of the absentee owners.

c) Management has more information about the entity's true financial position than do the absentee owners (i.e. stockholders).

Which of the following best describes the concept of risk assessment on which auditors can provide independent assurance? a) The risk that financial statements are misstated because of fraud. b) The risk that financial statements are misstated because of error or fraud. c) Whether management has systems in place to evaluate and effectively manage the entity's business risks. d) Developing client acceptance and continuance practices that minimize the likelihood of lawsuits against the auditor.

c) Whether management has systems in place to evaluate and effectively manage the entity's business risks.

When an auditor is asked to express an opinion on an entity's rent and royalty revenues, he or she may: a) accept the engagement, provided distribution of the auditor's report is limited to the entity's management. b) not accept the engagement unless also engaged to audit the full financial statements of the entity. c) accept the engagement, provided the auditor's opinion is expressed in a special report. d) not accept the engagement because to do so would be tantamount to agreeing to issue a piecemeal opinion.

c) accept the engagement, provided the auditor's opinion is expressed in a special report.

Which of the following is not a cash equivalent? a) certificates of deposit b) money market funds) c) marketable securities d) time deposits

c) marketable securities

Which of the following situations has the best chance of being detected when a CPA compares 2016 revenues and expenses with the prior year and investigates all changes exceeding a fixed percent? a) Because of worsening conditions, the 2016 provision for collectible accounts was inadequate b) an increase in property tax rates has not been recognized in the company's 2016 accrual c) the company changed its capitalization policy for small tools in 2016 d) the cashier began lapping accounts receivable in 2016

c) the company changed its capitalization policy for small tools in 2016

When the financial statements are fairly stated but the auditor concludes there is substantial doubt whether the client can continue in​ existence, the auditor should issue​ a(n) a) adverse opinion b) unqualified opinion c) unqualified opinion with explanatory paragraph d) qualified opinion only

c) unqualified opinion with explanatory paragraph

Inspection

careful scrutiny/detailed examination of entity's documents, records and physical exam of tangible resources.

The clerk who opens mail is assigned responsibility for preparing a document which identifies all cash collections received in the mail for a given day, this document is called

cash receipts prelist form

Types of documentations

company manuals and organization charts. Narratives. Internal Control Questionaires FLowcharts

When performing analytical analysis, auditors should carefully consider the ____________ and ____________ if the data before drawing conclusions based upon comparisons with industry averages.

comparatively timeliness

Inquiry (Audit Procedures)

consists of seeking information of knowledgeable persons (both financial and nonfinancial) within the entity or outside the entity.

Application controls

control specific to a particular system. ex: limit one employee to a certain amount they can wire.

General Controls

controls applicable to all systems. Example: change control (no one can go in and change programming without certain protocols)

Audit evidence (or evidential matter) may be defined as any information that ...

corroborates or refutes a premise

In which of the following situations would an auditor ordinarily issue an unqualified/unmodified financial statement audit opinion with no explanatory (or emphasis-of-matter/other-matter) paragraph? a) The auditor has substantial doubt about the entity's ability to continue as a going concern, but the circumstances are fully disclosed in the financial statements. b) The auditor wishes to emphasize that the entity had significant related-party transactions. c) The entity issues financial statements that present financial position and results of operations but omits the statement of cash flows. d) The auditor decides to refer to the report of another auditor as a basis, in part, for the auditor's opinion.

d) The auditor decides to refer to the report of another auditor as a basis, in part, for the auditor's opinion.

A written understanding between the auditor and the entity concerning the auditor's responsibility for fraud is usually set forth in a(n) a) management letter. b) internal control letter. c) letter of audit inquiry. d) engagement letter.

d) engagement letter.

An auditor would issue an adverse opinion if a) the audit was begun by other independent auditors who withdrew from the engagement. b) a qualified opinion cannot be given because the auditor lacks independence. c) a restriction on the scope of the audit was significant. d) the statements taken as a whole do not fairly present the financial condition and results of operations of the company.

d) the statements taken as a whole do not fairly present the financial condition and results of operations of the company.

A public entity changed from the straight-line method to the declining balance method of depreciation for all newly acquired assets. This change has no material effect on the current year's financial statements but is reasonably certain to have a substantial effect in later years. The client's financial statements contain no material misstatements and the auditor concurs with this change. If the change is disclosed in the notes to the financial statements, the auditor should issue a report with a(n): a) "except for" qualified opinion. b) consistency modification. c) explanatory paragraph. d) unqualified opinion.

d) unqualified opinion.

On April 7, 2008, you discovered that a debtor of Treble went bankrupt on January 22, 2008, due to a major uninsured fire that occurred on January 2, 2008.

d. Request the client revise and reissue the December 31, 2007 financial statements. The revision should involve the addition of a footnote, but no adjustment, to the December 31, 2007 financial statements.

Assertions with high inherent risk involve _________ ___ _______ transactions, complex calculations, difficult ___________________, significant ________________________________, or valuations that vary significantly based on _____________________.

difficult to audit accounting issues judgement by management economic factors

Substantive testing

does not rely on controls. use substantive procedeures for bulk of testing set control risk to max still must document understanding of internal control

On April 5, 2012, you discovered that, on March 30, 2012, a fire destroyed one of Schmidt's 13 plants.

e. No action is required.

On March 30, 2008, Treble settled a lawsuit out of court that had originated in 2004 and is currently listed as a contingent liability.

e. No action is required.

Because they involve management judgements or assumptions, ____________ _________ are generally the transactions with the highest level of inherent risk

estimation transactions

Analytical Procedures (Audit Procedures)

evaluations of financial information through analysis of plausible relationships among both financial and non-financial data.

Definition of control deficiency

exists when the design or operation of a control does not allow management or employees to prevent or detect misstatement on a timely basis

Assertions

expressed or implied representations by management regarding the recognition, measurement, presentation, and disclosure of information in the financial statements and related disclosures.

Audit risk refers to possibility that the auditors may unknowingly fail to appropriately modify their opinion on financial statements that are materially or immaterially misstated

false

Inquiries of management regarding the possibility of unrecorded contingencies will be useful in uncovering: managements intentional failure to disclose existing contingencies- when management does not comprehend accounting disclosure requirements-

managements intentional failure to disclose existing contingencies-No when management does not comprehend accounting disclosure requirements-Yes

Auditing Procedures

methods and techniques used by auditor to gather and evaluate audit evidence. Must balance cost with the objectives of the procedure.

major functions during revenue system

order entry, credit authorization, shipping, billing, cash receipts., accounts receivable, general ledger

Overall risk of revenue system

overstatement (expenses under, assets over, laibility under)

In relying upon the work of a specialist, the auditors must ascertain the professional _________________ and reputation of the specialist and review the reasonableness of underlying ______________ made by the specialists.

qualifications methods

Appropriateness is measure of _____________________ and _______________ relates to the assertion being addressed

quality being obtained relevance

The ____________ of audit evidence needed is affected by the risk of misstatements and the reliability of the evidence. The ___________ the risk, the more audit evidence required.

quantity greater

haphazard

random

A primary challenge for auditors is identifying any related party transactions that management has not disclosed, because they may be ___________________________________ with all other transactions

recorded in the account records

Nonrountine transactions

recur only periodically: depreciation, inventory

Routine transactions

recurring financial activities: sales, payroll, and purchases

Nature of Audit Evidence

refers to the form or type of information, which includes accounting records and other available information.

major transactions during revenue system

sales occur, cash collected, returns processed

Detection risk is restricted by performing __________ _________

substantive procedures

Is confirmations a test of control or substantive test

substantive test

is inspection of tangible assets a test of control or substantive test

substatnive test

The acronym SDLC stands for

systems development life cycle

An intensive investigation of a companys present information system in order to discover systems weaknesses is termed a

systems study

Is inspection of documents a test of control or substantive test

test of control

Regarding the use of another auditor:

the report may be qualified if the principal auditor is not willing to assume any responsibility for the work of the other auditor. A disclaimer may be issued if the segment audited by the other CPA is highly material.

Auditors should be cautious when accepting photo copies or tax copies because _____________________________________

they can be altered in anyway, making detection difficult.

Basic precsion

tolerable- expected

Absent any other changes, an increase in the risk of material misstatements results in an increase in audit risk

true

Both inherent risk and control risk exist independently of the audit of financial statements

true

Detection risk does not exist when no audit is performed

true

Inherent risk is the possibility of material misstatement before considering the client's internal control

true

Tests of Controls (Audit Procedures for Obtaining Audit Evidence)

used to test the operating effectiveness of controls in preventing, or detecting and correcting, material misstatements at the relevant assertion level.

No reference needs to be made to the other auditor when?

when the other auditor is known or closely supervised, they have audited an immaterial portion of the statement, and the principle auditor has thoroughly reviewed the other auditor's work

When should you use substantive testing strategy?

when there is few controls or ineffective controls

Negatives of m.u.s

zero or negative balacnces cause major issues (will never be selected) Error of 100% or more causes major issues (almost always makes you go over tolerable amount) more than a couple of errors causes that sample to blow up)

Revenues, gains, and investments by owners are all increases in net assets. What are the distinctions among them:

(1) Investments by owners differ from revenues and gains in that they represent transfers by owners to the entity, and they do not arise from activities intended to produce income. (2) Revenues differ from gains in that they arise from the entity's ongoing major or central operations. (3) Gains arise from peripheral or incidental transactions.

For a large publicly traded client, the auditors' examination of capital stock accounts will not normally include: Analysis of capital stock accounts. Confirmation of shares issued with the independent registrar. Accounting for the proceeds of major stock issues. Reconciliation of a stock certificate book with the general ledger.

Reconciliation of a stock certificate book with the general ledger.

When the auditors select a sample from the vouchers payable register at the end of the period and trace them to underlying documents, the auditors are gathering evidence primarily to support that: Recorded obligations were paid. Incurred obligations were recorded in the correct period. Recorded obligations occurred prior to year-end. Cash disbursements were recorded as incurred obligation.

Recorded obligations occurred prior to year-end.

Which of the following is least likely to be considered an act discreditable to the accounting profession? Disclosing confidential client information. Failure to comply with federal laws regarding the filing of tax returns. Knowingly disclosing CPA exam questions. Refusing to provide the client with copies of working papers the client prepared for the auditor.

Refusing to provide the client with copies of working papers the client prepared for the auditor.

Control Reliance Strategy

Rely on internal controls. required to test controls to ensure they are in place if testing is god, this reduces but does not eliminate substantive testing

At present, we think that cybercrime is:

Rising

________ ___________ ______________ are designed to obtain an understanding of the client and its environment to assess the risks of material misstatements.

Risk Assessment Procedures

Follow and are closely related to management's assertions about classes of transactions

Transaction-related audit objectives

Authorization of which of the following is least likely to be found during a review of the minutes of the board of directors? Dividends. New debt issuance. New bank accounts. Write-off of trade accounts receivable.

Write-off of trade accounts receivable.

List four specific matters that should be included in a client representation letter.

• Management's acknowledgment of its responsibility for the fair presentation in the statements of financial position, results of operations, and cash flows in conformity with applicable accounting standards. • Availability of all financial records and related data. • Completeness and availability of all minutes of meetings of stockholders, directors, and committees of directors. • Management's belief that the effects of any uncorrected financial statement misstatements are immaterial to the financial statements. • Information concerning fraud involving (a) management, (b) employees who have significant roles in internal control, or (c) others where fraud could have a material effect on the financial statements. • Information concerning related-party transactions and related amounts receivable or payable. • Unasserted claims or assessments that the entity's lawyer has advised are probable of assertion and must be disclosed in accordance with accounting standards. • Bankruptcy of a major customer with an outstanding account receivable at the balance sheet date. • A merger or acquisition after the balance sheet date.

State two of the three purposes of the client letter of representation.

• To impress upon management its responsibility for the assertions in the financial statements. • To remind management of potential misstatements or omissions in the financial statements. • To document the responses from management to inquiries about various aspects of the audit.

State the two primary types of subsequent events that require consideration by management and evaluation by the auditor, and give an example of each type.

• Type 1: Events that have a direct effect on the financial statements and require adjustment of the current year's financial statement amounts. Examples include declaration of bankruptcy by a customer with an outstanding accounts receivable balance due to deteriorating financial condition; settlement of litigation at an amount different from the amount recorded on the books, disposal of equipment not being used in operations at a price below the current book value; sale of investments at a price below recorded cost. • Type 2: Events that have no direct effect on the financial statements but for which disclosure is required. Examples include a decline in the market value of securities held for temporary investment or resale; issuance of bonds or equity securities; a decline in the market value of inventory as a consequence of government action barring further sale of a product; the uninsured loss of inventories as a result of fire; a merger or an acquisition.

What was the percentage of cases in 2016 referred to law enforcement that pled guilty or no contest?

56.8%

27. The audit procedures used to verify accrued liabilities differ from those employed for the verification of accounts payable because A. accrued liabilities usually pertain to services of a continuing nature, while accounts payable are the result of completed transactions. B. accrued liability balances are less material than accounts payable balances. C. evidence supporting accrued liabilities is nonexistent, while evidence supporting accounts payable is readily available. D. accrued liabilities at year-end will become accounts payable during the following year.

A

28. The auditor is most likely to verify accrued commissions payable in conjunction with the A. sales cutoff review. B. verification of employees. C. review of post balance sheet date disbursements. D. examination of trade accounts payable.

A

31. An entity's procurement system ends with the assumption of a liability and the eventual payment of the liability. Which of the following best describes the auditor's primary concern with respect to liabilities resulting from the procurement system? A. Accounts payable are not materially understated. B. Authority to incur liabilities is restricted to one designated person. C. Acquisition of materials is not made from one vendor or one group of vendors. D. Commitments for all purchases are made only after established competitive bidding procedures are followed.

A

38. As an in-charge auditor, you are reviewing a summary of control weaknesses in cash disbursement procedures. Which one of the following weaknesses, standing alone, should cause you the least concern? A. Checks are signed by only one person. B. Signed checks are distributed by the controller to approved payees. C. Treasurer fails to establish validity of names and addresses of check payees. D. Cash disbursements are made directly out of cash receipts.

A

Auditing Standards (SAS No. 59) requires auditors to evaluate whether there is a substantial doubt about a client's ability to continue as a going concern. One of the most important audit procedures to perform to assess the going concern question is: A) analytical procedures. B) confirmations of creditors. C) statistical sampling procedures. D) inquiries of client and its legal counsel.

A) analytical procedures.

15. The occurrence assertion for accounts payable includes A. determining whether all accounts payable are recorded. B. determining whether all accounts payable actually are liabilities. C. determining whether all accounts payable are recorded in the proper period. D. determining whether all accounts payable are properly classified in the financial statements.

B

19. To determine whether accounts payable are complete, an auditor performs a test to verify that all merchandise received is recorded. The population of documents for this test consists of all A. payment vouchers. B. receiving reports. C. purchase requisitions. D. vendors' invoices.

B

21. An important primary purpose of the auditor's review of the entity's procurement system should be to determine the effectiveness of the activities to protect against A. improper materials handling. B. unauthorized persons issuing purchase orders. C. mispostings of purchase returns. D. excessive shrinkage or spoilage.

B

30. Which of the following audit procedures is least likely to detect an unrecorded liability? A. Analysis and recomputation of interest expense. B. Analysis and recomputation of depreciation expense. C. Mailing of standard bank confirmation forms. D. Reading of the minutes of meetings of the board of directors.

B

When using the probability THRESHOLD for contingencies, the LIKELIHOOD of the occurrence of the event is classified as: A) not likely, likely, or highly likely. B) remote, reasonably possible, or probable. C) slight, moderate, great. D) remote, likely, possible

B) remote, reasonably possible, or probable.

16. The cutoff assertion for accounts payable includes A. determining whether all accounts payable are recorded. B. determining whether all accounts payable actually are liabilities. C. determining whether all accounts payable are recorded in the proper period. D. determining whether all accounts payable are properly classified in the financial statements.

C

17. The accounts payable department receives the purchase order form to accomplish all of the following except to: A. compare invoice price to purchase order price. B. ensure that the purchase had been properly authorized. C. ensure that the goods had been received by the party requesting the goods. D. compare quantity ordered to quantity purchased.

C

After the balance sheet date, but prior to the issuance of the audit report, the client suffers an uninsured loss of their inventory as a result of a fire. The amount of the loss is material. The auditor should: A) adjust the financial statements for the year under audit. B) add a paragraph to the audit report. C) advise the client to disclose the event in the notes to the financial statements. D) advise the client to delay issuing the financial statements until the economic loss can be determined

C) ADVISE the client to DISCLOSE the event in the notes to the financial statements.

With which of the following client personnel would it generally NOT be appropriate to inquire about commitments or contingent liabilities? A) Controller B) President C) Accounts receivable clerk D) Vice president of sales

C) Accounts receivable clerk

24. An auditor compares information on canceled checks with information contained in the cash disbursements journal. The objective of this test is to determine that A. recorded cash disbursement transactions are properly authorized. B. proper cash purchase discounts have been recorded. C. cash disbursements are for goods and services actually received. D. no discrepancies exist between the data on the checks and the data in the journal.

D

25. Which of the following procedures would an auditor most likely perform in searching for unrecorded payables? A. Reconcile receiving reports with related cash payments made just prior to year-end. B. Contrast the ratio of accounts payable to purchases with the prior year's ratio. C. Vouch a sample of creditor balances to supporting invoices, receiving reports and purchase orders. D. Compare cash payments occurring after the balance sheet date with the accounts payable trial balance.

D

33. The authority to accept incoming goods in receiving should be based on a(an) A. vendor's invoice. B. materials requisition. C. bill of lading. D. approved purchase order.

D

The "S" in the Acronym "AIS" stands for

System

Interpret the following credit terms: 4/15, Net 30

a 4 percent discount can be taken if the balance is paid within 15 days; otherwise, the balance is due in 30 days

Who do you report to if you find a significant definciecy

audit committee

List 2 types of information normally contained in a legal letter to the client's attorneys.

• List and evaluation of pending or threatened litigation to which the attorney has devoted significant attention • A list of unasserted claims and assessments considered by management to probable and reasonably possible of an unfavorable outcome • A description and evaluation of the outcome of each pending or threatened litigation • Comments on unasserted claims where their views are different than managements

Prepare a flowchart of internal control over sales

Risk Assessment procedures (other than analytical procedures)

____________ procedures are a critical component to planning and designing an audit approach

Risk assessment

Re-performance

Check calculations and reconciliations made by the entity. Amounts calculated for depn and accrued interest may be recalculated.

Indicate whether each of the following items is a real or nominal account and whether it appears in the balance sheet or the income statement: (a) Prepaid Rent. (b) Salaries and Wages Payable. (c) Inventory. (d) Accumulated Depreciation—Equipment. (e) Equipment. (f) Service Revenue. (g) Salaries and Wages Expense. (h) Supplies.

(a) Real account; balance sheet. (b) Real account; balance sheet. (c) Inventory is generally considered a real account appearing on the balance sheet. (d) Real account; balance sheet. (e) Real account; balance sheet. (f) Nominal account; income statement. (g) Nominal account; income statement. (h) Real account; balance sheet

(a) Qualitative characteristic being employed when companies in the same industry are using the same accounting principles. (b) Quality of information that confirms users' earlier expectations. (c) Imperative for providing comparisons of a company from period to period. (d) Ignores the economic consequences of a standard or rule. (e) Requires a high degree of consensus among individuals on a given measurement. (f) Predictive value is an ingredient of this fundamental quality of information. (g) Four qualitative characteristics that are related to both relevance and faithful representation. (h) An item is not recorded because its effect on income would not change a decision. (i) Neutrality is an ingredient of this fundamental quality of accounting information. (j) Two fundamental qualities that make accounting information useful for decisionmaking purposes. (k) Issuance of interim reports is an example of what enhancing quality of relevance?

(a) comparability (b) confirmatory value (c) comparability (consistency) (d) neutrality (e) verifiability (f) relevance (g) comparability, verifiability, timeliness, understandability (h) materiality (i) faithful representation (j) relevence, faithful representation (k) timeliness

(a) Arises from peripheral or incidental transactions. (b) Obligation to transfer resources arising from a past transaction. (c) Increases ownership interest. (d) Declares and pays cash dividends to owners. (e) Increases in net assets in a period from nonowner sources. (f) Items characterized by service potential or future economic benefit. (g) Equals increase in assets less liabilities during the year, after adding distributions to owners and subtracting investments by owners. (h) Arises from income statement activities that constitute the entity's (i) Residual interest in the assets of the enterprise after deducting its liabilities. (j) Increases assets during a period through sale of product. (k) Decreases assets during the period by purchasing the company's own stock. (l) Includes all changes in equity during the period, except those resulting from investments by owners and distributions to owners.

(a) gains, losses (b) liabilities (c) investments by owners, comprehensive income (d) distributions to owners (e) comprehensive income (f) assets (g) comprehensive income (h) revenues, expenses (i) equity (j) revenues (k) distributions to owners (l) comprehensive income

A poorly-planned information system can result in

(all of the above) Employee resistance and even sabotage inflexible systems that are hard to maintain or modify Systems that solve the wrong problems

COVER-U Assertions relevant to Account Balances

*CVER* "C" Completeness: all assets, liabilities, and equity interests that should have been recorded have been recorded "V" Valuation, Allocation, and Accuracy: assets, liabilities and equity interests are recorded fairly and at appropriate amounts and any resulting valuation or allocation adjustments are appropriately recorded. "E" Existence and Occurrence: Assets, liabilities and equity interests exist. "R" Rights and Obligations: The entity holds or controls the rights to assets and liabilities are the obligations of the entity.

Standard Auditing Procedures = *FIVE* CARROT CARS

*FIVE* CARROT CARS F - Footing, Crossfooting and Recalculation: verify mathematical accuracy I - Inquiry: requesting information from knowledgeable parties internally and externally (AR & AP) V - Vouching: examines support for info recorded in financial statements back to supporting documents. E -Examination/Inspection: provides evidence about Existence assertion.

Types of Audit Evidence

-Records or initial entries and supporting records -Spreadsheet supporting allowance calculations -Invoices -General and Subsidiary ledgers -Contracts -Results of inquiries -Worksheets -Reconciliations

The public company holds/controls rights to the assets and liabilities are obligations of the company at a given date

Rights and obligations

An auditor identified a material weakness in December. The client was informed and corrected it shortly after the "as of date" (December 31); the auditor agrees that the correction eliminates the material weakness as of January 31. The appropriate report under a PCAOB Standard 5 audit of internal control is

. Adverse

Expenditure Cycle: Internal control - Purchases

/\ | Trace = biggest concern is understatement of expenses & liabilities Purchase Requisition uses the Pre numbered req orders that are properly approved. *PA* of PAID TIPS

What is the 8 types of audit procedures?

1) Analytical procedures 2) External confirmation 3) Inspection of tangible assets 4) Inspection of records and documents 5) Inquiries of knowledge about persons within or outside the entity 6) Observation of process or procedures being performed by others 7) Recalculation of mathematical accuracy 8) Reperformance of procedures

An audit may include computations of various balance sheet and operating ratios for comparisons to prior years and industry averages. Discuss the validity and limitations of ratio analysis in an audit

1) Board overview 2) Ranks lower in reliability and validity than direct evidence 3) Usually considered circumstantial

In the course of an audit, the auditor asks many questions of client officers and employees. 1) Describe the factors that the auditor should consider in evaluating inquiry and oral evidence provided by the client officers and employees

1) Competence of person we're asking 2) Disinterestedness (objectiveness) 3) Logic and reasonableness of response

Three conditions requiring a departure from an unqualified standard report. Give examples of each.

1) auditor is not independent - auditor owns stock in the client's business 2) financial statements are not in accordance with GAAP - the client insists on using replacement costs for fixed assets 3) scope has been restricted - the client will not permit the auditor to confirm material receivables

What are the three options available to the primary auditor responsible for the opinion​ (principal auditor), and when should each be​ used? (in the case that other auditors are used)

1) reference is made to the other auditor 2) issue a shared opinion or report 3) qualify the opinion

Describe the broad purposes of analytical procedures.

1) understanding the client's business and industry 2) assessment of the entity's ability to continue as a going concern 3) reduction of detailed audit tests 4) indication of the presence of possible misstatements in the financial statements

Matching Audit Procedures to Assertions: Valuation, Allocation and Accuracy

1. Inspection of documentation supporting transactions. 2. Footing and crossfooting of schedules. 3. Independent recalculation of ex) depreciation charges or estimates made by client. 4. Reconciliation of supporting schedules to general ledger line items.

Assertions about account balances at the period end (4):

1. Existence 2. Rights and obligations 3. Completeness 4. Valuation and allocation

Managements Assertions Process

1. Identify the financial reporting risk and the related controls. 2. Consider which locations to include in their review (should look for pervasive, entity level controls) 3. Evaluate the evidence about the operating effectiveness of ICFR ( perform and document their tests)

Matching Audit Procedures to Assertions: Rights and Obligations

1. Inspection of documentation supporting transaction, inspection of contracts, etc.

Auditor should perform the following procedures to identify instances of noncompliance with other laws and regulations that may have a material effect on the financial statements:

1. inquire of management and those charged with governance about whether the entity is in compliance with such laws and regulations 2. inspect correspondence, if any, with the relevant licensing or regulatory authorities

Four categories of audit report

1. standard unqualified - clean opinion.. no paragraphs or changes. 3 paragraphs 2. Unqualified with explanatory paragraph or modified wording 3. Qualified- something wrong with financial statements. Auditors are satisfied but there is something material that auditors do not agree with. 4. Adverse or disclaimer- Causes delisting. Doesn't accept GAAP. Financial statements are not reliable. Disclaimer is no opinion on financial statements

When to use non statistical sampling

1. when there are items above materiality (you have to look at material items) 2. relatively small population 3. if you expect multiple errors 4. more expedient to do so

General-balance related audit objectives:

1.Existence 2. Completeness 3. Accuracy 4. Classification 5. Cutoff 6. Detail tie-in 7. Realizable value 8. Rights and obligations

Types of Procedures for managements assertions

1.Walkthrough (usually required) Inspection of documents (examine 25 invoices, looking for approval) 2.Reperformance (the act of an auditor reperforming a transacation to make sure proper controls are in place) 3.Inquiry and observation (i.e. observation = review of account reconcilliations) 4.Review of period end journal entries (looking for cuttoff journal entries)

Control Activities 3 principles

10. Select and develop the control activities themselves. i.e: performance reviews (not individual, but for company as a whole). other examples include: Physical controls, segregation of duties, and information processing controls. 11. Select and develop general controls over technology. (general controls, access controls, application controls) 12. Deploy control activities through policies and procedures (in other words, put them into action)

Information and communication 3 principles

13. uses relevant information (information systems includes infrastructure (hardware), software, people, procedures (manual and automated), and data 14. communicate internally 15. communicate externally

Monitoring activities 2 principles

16.Conduct ongoing and/or separate evaluations 17. Evaluate and communicate deficincies (to b.o.d. and management)

A client company has not paid its 20X6 audit fees. According to the AICPA Code of Professional Conduct, in order for the auditor to be considered independent with respect to the 20X7 audit, the 20X6 audit fees must be paid before the: 20X6 report is issued. 20X7 fieldwork is started. 20X7 report is issued. 20X8 fieldwork is started.

20X7 report is issued.

Which of the following categories of fraud was caught by external audit in 2016

3.8%

How do the eight parts of a standard unmodified opinion audit report for nonpublic companies differ from those found in a qualified opinion report?

5. Auditor's responsibility: The first two auditor responsibility paragraphs are the same as the standard unmodified opinion report. The third paragraph is modified to state that the audit evidence obtained provides a sufficient and appropriate basis for the qualified audit opinion. That paragraph is following by a new paragraph that describes the basis for the qualified opinion. 6. Opinion paragraph: The opinion paragraph is modified to include the term except for in the opinion paragraph.

Entitys risk assesment process's 4 principles

6. Organization specific suitable objective (needs clear objectives to be able to identify and assess risk) 7. Identify and Assess risk ( consider possible risks that threaten the achievment of objectives) 8. Assess Fraud Risk (Utilize fraud triangle) 9. identify and analyze significant changes (what impact could the change have on the control)

47. Substantive procedures to examine the occurrence assertion for accounts payable include A. selecting a sample of vouchers and agreeing them to authorized purchase orders. B. selecting a sample of vouchers and tracing them to the purchases journal. C. comparing dates on vouchers to dates in the purchases journal. D. recomputing the mathematical accuracy of a sample of vendor invoices.

A

50. Purchase cutoff procedures should be designed to test whether or not all inventory A. purchased and received before the year-end was recorded before year-end. B. on the year-end balance sheet was carried at lower of cost or market. C. on the year-end balance sheet was paid for by the company. D. owned by the company is in the possession of the company.

A

56. Accounts payable confirmations are used to test A. both the existence and completeness audit assertions. B. only the existence audit assertion. C. only the completeness audit assertion. D. either existence or completeness, depending upon the response rate.

A

62. The mailing of disbursement checks and remittance advices should be controlled by the employee who A. signed the checks last. B. approved the vouchers for payment. C. matched the receiving reports, purchase orders and vendors' invoices. D. verified the mathematical accuracy of the vouchers and remittance advices.

A

65. Which of the following test(s) of details of transactions can be used as a dual-purpose test in conjunction with tests of controls? A. Test a sample of purchase requisitions for proper authorization. B. Obtain selected vendors' statements and reconcile to vendor accounts. C. Obtain listing of accounts payable and compare total to general ledger. D. Review results of confirmations of selected accounts payable.

A

Distinguish between a report qualified due to a GAAP departure and one qualified due to a scope limitation.

A qualified report due to a scope limitation is issued when the auditor can neither perform procedures that he or she considers necessary nor satisfy himself or herself by using alternative procedures, usually due to the existence of conditions beyond the client's or the auditor's control, but the amount involved in the financial statements is not highly material. An important part of qualified opinion due to a scope limitation is that it results from not accumulating sufficient appropriate audit evidence, either because of the client's request or because of circumstances beyond anyone's control. When the opinion is qualified due to a scope limitation, the auditor modifies both the scope and opinion paragraphs. The scope paragraph is modified to indicate that the auditor's scope has been restricted and the opinion paragraph is modified to include the qualified opinion. A report qualified as to opinion only results when the auditor has accumulated sufficient appropriate evidence but has concluded that the financial statements are not correctly stated. The only circumstance in which an opinion only qualification is appropriate is for material, but not highly material, departures from GAAP. When the opinion is qualified due to a GAAP departure, only the opinion paragraph is modified to include the qualified opinion. The scope paragraph is not modified because there has been no limitation on the auditor's scope.

Professional skepticism consists of 2 primary components:

A questioning mind and a critical assessment of the audit evidence

What is the relationship between detection risk and audit risk?

Audit risk uses a formula Detection is within audit risk Audit risk=IR x CR x DR

What four circumstances are required for a standard unmodified opinion audit report to be issued?

A standard unmodified opinion audit report may be issued under the following circumstances: 1. All statements-balance sheet, income statement, statement of retained earnings, and statement of cash flows-are included in the financial statements. 2. Sufficient appropriate evidence has been accumulated and the auditor has conducted the engagement in a manner that enables him or her to conclude that the audit was performed in accordance with auditing standards. 3. The financial statements are presented in accordance with appropriate accounting standards such as U.S. generally accepted accounting principles or IFRS. This also means that adequate disclosures have been included in the footnotes and other parts of the financial statements. 4. There are no circumstances requiring the addition of an explanatory paragraph or modification of the wording of the report.

For clients that distribute checks or cash payments and have significant payroll control weakness, which of the following audit procedures is aimed at determining whether every name on the company payroll is a bona fide employee actually on the job? A surprised observation of a paycheck distribution, while establishing the identity of each employee receiving payment. A test of payroll extensions. Analytical comparisons of budgeted to actual payroll expense. Comparison of payee names on canceled payroll checks with the payroll register.

A surprised observation of a paycheck distribution, while establishing the identity of each employee receiving payment.

What is one of the main reasons an attorney may REFUSE to provide auditors with complete information about contingent liabilities? The attorneys refuse to disclose information they consider confidential. The attorneys refuse to respond due to a lack of knowledge about matters involving contingent liabilities. A)Yes Yes B) No No C) Yes No D) No Yes

A) CONFIDENTIAL. (YES) LACK OF KNOWLEDGE (YES)

The audit procedures for the subsequent events review can be divided into two categories: (1) procedures normally integrated as a part of the verification of year-end account balances, and (2) those performed specifically for the purpose of discovering subsequent events. Which of the following procedures is in the SECOND category? A) Correspond with attorneys. B) Test the collectability of accounts receivable by reviewing subsequent period cash receipts. C) Subsequent period sales and purchases transactions are examined to determine whether the cutoff is accurate. D) Compare the subsequent-period purchase price of inventory with the recorded cost as a test of lower of cost or market valuation

A) Correspond with attorneys.

The audit procedures for the subsequent events review can be divided into two categories: (1) procedures normally integrated as a part of the verification of year-end account balances, and (2) those performed specifically for the purpose of discovering subsequent events. Which of the following procedures is in category 2? A) Correspond with attorneys. B) Test the collectability of accounts receivable by reviewing subsequent period cash receipts. C) Subsequent period sales and purchases transactions are examined to determine whether the cutoff is accurate. D) Compare the subsequent-period purchase price of inventory with the recorded cost as a test of lower-of-cost-or-market valuation.

A) Correspond with attorneys.

Which type of subsequent event requires consideration by management and evaluation by the auditor? Subsequent events that have a direct effect on the financial statements and require adjustment. Subsequent events that do not have a direct effect on the financial statements but for which disclosure may be required. A)Yes Yes B) No No C) Yes No D) No Yes

A) DIRECT EFFECT (YES) DISCLOSURE REQUIRED (YES)

Which of the following procedures and methods are important in assessing a company's ability to continue as a GOING CONCERN? -Discussions with management regarding potential financial difficulties. -Evaluation of management's plans to avoid bankruptcy. A) Yes Yes B) No No C) Yes No D) No Yes

A) DISCUSSIONS (YES) EVALUATION (YES)

Which of the following is not required to be communicated to the audit committee or similarly designated body under auditing standards? A) Disagreements with the company over two acceptable accounting treatments for a significant transaction. B) Disagreements with management about the scope of the audit, applicability of accounting principles, or wording of the audit report. C) Difficulties encountered in performing the audit, such as lack of availability of client personnel and failure to provide necessary information. D) Auditor's responsibilities under generally accepted auditing standards, including responsibility for evaluating internal control and the concept of reasonable rather than absolute assurance.

A) Disagreements with the company over two acceptable accounting treatments for a significant transaction.

An auditor's decision concerning whether or not to "dual date" the audit report is based upon the auditor's willingness to: A) extend auditing procedures and assume responsibility for a greater period of time. B) accept responsibility for subsequent events. C) permit inclusion of a footnote captioned: event (unaudited) subsequent to the date of the auditor's report. D) assume responsibility for events subsequent to the issuance of the auditor's report

A) EXTEND auditing procedures and ASSUME responsibility for a greater period of time.

If an auditor concludes there are contingent liabilities, then he or she must EVALUATE the: Materiality of the potential liability. Nature of the disclosure to be included in the financial statements. A) Yes Yes B) No No C) Yes No D) No Yes

A) MATERIALITY of the potential liability. (YES) NATURE of the disclosure to be included in the financial statements. (YES)

Which of the following is a contingent liability with which an auditor is particularly CONCERNED? -Notes receivable discounted -Product warranties A) Yes Yes B) No No C) Yes No D) No Yes

A) Notes receivable discounted (YES) Product warranties (YES)

In connection with the ANNUAL AUDIT, which of the following is NOT a "subsequent events" procedure? A) Prepare any necessary closing journal entries. B) Examine the minutes of stockholders and directors meetings subsequent to the balance sheet date. C) Review journals and ledgers. D) Obtain a letter of representation

A) Prepare any necessary closing journal entries.

In connection with the annual audit, which of the following is not a "subsequent events" procedure? A) Review available interim financial statements. B) Read available minutes of meetings of stockholders, directors, and committees and, for meetings where minutes are not available, inquire about matters dealt with at such meetings. C) Make inquiries with respect to the financial statements covered by the auditor's previously issued report if new information has become available during the current examination that might affect that report. D) Discuss with officers the current status of items in the financial statements that were accounted for on the basis of tentative, preliminary, or inconclusive data.

A) Review available interim financial statements.

A company guarantees the debt of an affiliate. Which of the following best describes the audit procedure that would make the auditor aware of the guarantee? A) Review minutes and resolutions of the board of directors. B) Review prior year's audit files with respect to such guarantees. C) Review the possibility of such guarantees with the chief accountant. D) Review the legal letter returned by the company's outside legal counsel.

A) Review minutes and resolutions of the board of directors.

Which of the following statements is most correct about an auditor's required communication with management and those charged with corporate governance? A) The auditor is required to inform those charged with governance about significant errors discovered and subsequently corrected by management. B) Any significant matter reported to those charged with governance must also be communicated to management. C) Communication is required before the audit report is issued. D) Auditor does not have any requirement to communicate with anyone other than the company's senior management.

A) The auditor is required to inform those charged with governance about significant errors discovered and subsequently corrected by management.

The auditor has completed her assessment of subsequent events. The proper accounting for subsequent events that have a direct effect on the financial statements is to: A) adjust the financial statements for the year under audit. B) disclose in the notes to financial statement the amount of the adjustment. C) duly note in the audit workpapers that next year's financial statements need to be adjusted. D) make no adjustment of the financial statements for the year under audit.

A) adjust the financial statements for the year under audit.

A commitment is best described as: A) an agreement to commit the firm to a set of fixed conditions in the future. B) an agreement to commit the firm to a set of fixed conditions in the future that depends on company profitability. C) an agreement to commit the firm to a set of fixed conditions in the future that depends on current market conditions. D) a potential future obligation to an outside party for an as yet to be determined amount.

A) an agreement to commit the firm to a set of fixed conditions in the future.

A COMMITMENT is best described as: A) an agreement to commit the firm to a set of fixed conditions in the future. B) an agreement to commit the firm to a set of fixed conditions in the future that depends on company profitability. C) an agreement to commit the firm to a set of fixed conditions in the future that depends on current market conditions. D) a potential future obligation to an outside party for an as yet to be determined amount

A) an agreement to commit the firm to a set of fixed conditions in the future. REGARDLESS of what happens to profits or the economy.

Sufficiency of audit evidence: Making decisions about the sufficiency of audit evidence requires significant professional judgement. Explain how the following factors might impact the auditor's professional judgement regarding the sufficiency of audit evidence involved in the audit of inventory for a manufacturer: A) Materiality B) Risk of material misstatement C) Size and characteristics of the population

A) assertions that are more material to financial statements users require more sufficient evidence B) Assertions that have a higher risk of material misstatements (RMM) require more sufficient evidence C) Larger more diverse population require more sufficient evidence

When a client will not permit inquiry of outside legal counsel, the audit report will ordinarily contain a(n): A) disclaimer of opinion. B) qualified opinion. C) standard unqualified opinion. D) unqualified opinion with a separate explanatory paragraph.

A) disclaimer of opinion.

An auditor's decision concerning whether or not to DUAL DATE an audit report is primarily based on the auditor's decision to: A) extend appropriate audit procedures. B) assume responsibility for events after the date of the auditor's report. C) assume responsibility for event from fiscal year end to the date of the audit report. D) roll the dice and hope for a successful outcome

A) extend appropriate audit procedures.

An auditor's decision concerning whether or not to dual date an audit report is primarily based on the auditor's decision to: A) extend appropriate audit procedures. B) assume responsibility for events after the date of the auditor's report. C) assume responsibility for event from fiscal year end to the date of the audit report. D) roll the dice and hope for a successful outcome.

A) extend appropriate audit procedures.

An auditor's decision concerning whether or not to "dual date" the audit report is based upon the auditor's willingness to: A) extend auditing procedures and assume responsibility for a greater period of time. B) accept responsibility for subsequent events. C) permit inclusion of a footnote captioned: event (unaudited) subsequent to the date of the auditor's report. D) assume responsibility for events subsequent to the issuance of the auditor's report.

A) extend auditing procedures and assume responsibility for a greater period of time.

Refusal by a client to prepare and sign the representation letter would require the auditor to issue a: A) qualified opinion or a disclaimer. B) adverse opinion or a disclaimer. C) qualified or an adverse opinion. D) unqualified opinion with an explanatory paragraph.

A) qualified opinion or a disclaimer.

Which of the following audit procedures would most likely assist an auditor in identifying conditions and events that may indicate there could be substantial doubt about an entity's ability to continue as a going concern? A) review compliance with the terms of debt agreements B) confirmation of accounts receivable from principal customers C) reconciliation of interest expense with debt outstanding D) confirmation of bank balances

A) review compliance with the terms of debt agreements

An auditor performs interim work at various times throughout the year. The auditor's subsequent events work should be extended to the date of: A) the auditor's report. B) a post-dated footnote. C) the next scheduled interim visit. D) the final billing for audit services rendered

A) the auditor's report.

An auditor performs interim work at various times throughout the year. The auditor's subsequent events work should be extended to the date of: A) the auditor's report. B) a post-dated footnote. C) the next scheduled interim visit. D) the final billing for audit services rendered.

A) the auditor's report.

Contingent liability DISCLOSURE in the footnotes of the financial statements would normally be made WHEN: A) the outcome of the accounting event is deemed probable, but a reasonable estimation as to the amount cannot be made by the client or auditor. B) a reasonable estimation of the loss can be made, but the outcome is not probable. C) the outcome of the accounting event is deemed probable, and a reasonable estimation as to the amount can be made. D) the outcome of the accounting event as well as a reasonable estimation of the loss cannot be made

A) the outcome of the accounting event is deemed PROBABLE, but a reasonable estimation as to the AMOUNT CANNOT BE MADE by the client or auditor.

Contingent liability disclosure in the footnotes of the financial statements would normally be made when: A) the outcome of the accounting event is deemed probable, but a reasonable estimation as to the amount cannot be made by the client or auditor. B) a reasonable estimation of the loss can be made, but the outcome is not probable. C) the outcome of the accounting event is deemed probable, and a reasonable estimation as to the amount can be made. D) the outcome of the accounting event as well as a reasonable estimation of the loss cannot be made.

A) the outcome of the accounting event is deemed probable, but a reasonable estimation as to the amount cannot be made by the client or auditor.

30. Purchase cutoff procedures should be designed to test whether or not all inventory. A. Purchased and received before the year-end was recorded before year-end. B. On the year-end balance sheet was carried at lower of cost or market. c. On the year-end balance sheet was paid for by the company. D. Owned by the company is in the possession of the company.

A. Purchased and received before the year-end was recorded before year-end.

11. Which of the following is not considered an important element of segregation of duties in the purchasing process? A. Separating the check signing and check mailing functions. B. Separating the accounts payable and check signing functions. C. Separating the accounts payable and receiving functions. D. Separating the purchasing function from the receiving function.

A. Separating the check signing and check mailing functions.

6. Accounts payable confirmations are used to test A. Both the existence and completeness audit assertions B. Only the existence audit assertion. C. Only the completeness audit assertion. D. Either existence or completeness, depending upon the response rate.

A. Both the existence and completeness audit assertions

How detailed should documentation be? It depends on several factors, such as A. the ____________________________ being performed B. the ______________________________ involved in the area being tested C. the ______________________________ to the overall audit D. the ________________________ in performing the work E. the _________________________________

A. nature of auditing procedure B. risk of misstatement C. Sufficiency of the evidence D. Extent of judgement involved E. nature of the findings/results

An entity's procurement system ends with the assumption of a liability and the eventual payment of the liability. Which of the following best describes the auditor's primary concern with respect to liabilities resulting from the procurement system?

Accounts payable are not materially understated

Which of the following best describes the specific accounts payable that are selected for confirmation? Accounts with large balances. Accounts with zero balances. Accounts with a large amount of activity regardless of their balance. Accounts for which vendor statements are available.

Accounts with a large amount of activity regardless of their balance.

The audit procedures used to verify accrued liabilities differ from those employed for the verification of accounts payable because

Accrued liabilities usually pertain to services of continuing nature, while accounts payable are the result of completed transactions.

Financial and other info is disclosed appropriately and at appropriate amounts

Accuracy and valuation

Transactions include the purchase of inventory, supplies, and other goods/services related to operations

Acquisition and payment cycle

In a PERT network diagram, the letters represent

Activities

If management insists on financial statement disclosure that the auditor finds unacceptable, the auditor can either issue a(n) ____________ or withdraw from the engagement.

Adverse or qualified opinion

A control deficiency that is more than a significant deficiency is most likely to result in what form of audit opinion relating to internal control?

Adverse.

Which of the following forms of advertising would most likely be considered a violation of the AICPA Code of Professional Conduct? Advertising including the types of services offered and the standard fees for the services. Advertising including the experience of the firm's professional staff. Advertising including an indication that the firm has a close relationship with several tax court judges. Advertising including the percentage of the firm's staff that have CPA certificates.

Advertising including an indication that the firm has a close relationship with several tax court judges.

Which of the following is not an example of a source document?

Aging Report

Audit Evidence

All information, from whatever source, used by the auditor in arriving at the conclusions on which the audit opinion is based. Includes the accounting records and other info.

A Type I subsequent event usually requires A. No adjustment to the financial statements B. An adjustment to the financial statements C. Withdrawal from the engagement D. No action

An adjustment to the financial statements

The basic symbols used in data flow diagram are:

An open ended triangle, line and arrow, circle, and square

Compare current financial information with comparable prior periods

Analytical procedures

When are analytical procedures required during an audit? Explain why auditors use analytical procedures extensively in all parts of the audit.

Analytical procedures are required in the planning and completion phases of the audit. Auditors use analytical procedures extensively because of their relatively low cost and effectiveness in identifying potential misstatements.

Tendency to make assessments by starting from an initial value and then adjusting insufficiently away from the initial value

Anchoring

It is important to be able to recognize the symptoms of employee fraud. In practice, which of the following might be the clue that fraud might be occurring?

Anomalies that, together, seem unreasonable

What is meant by reports involving the use of other auditors?

Another CPA firm has performed part of the audit.

If group auditors make no reference to component auditors whose work they have relied on as a part of the basis for their report, the group auditors: Are not required to investigate the professional reputation of the component auditors. Are issuing an inappropriate report. Are assuming responsibility for the work of the component auditors. Are issuing a qualified opinion.

Are assuming responsibility for the work of the component auditors.

Accrued liabilities generally differ from accounts payable in that accrued liabilities: Are often based on estimates. Are usually confirmed at year-end. Depend upon the existence of a transaction for original recording of the account. Are never included in cost of goods sold.

Are often based on estimates.

Which of the following questions would most likely be included in an internal control questionnaire concerning the completeness assertion for purchases?

Are purchase orders, receiving reports, and vouchers prenumbered and periodically accounted for?

The primary objective of analytical procedures used near the end of an audit is to: Obtain evidence from details tested to corroborate particular assertions. Identify areas that represent specific risks relevant to the audit. Assist the auditor when forming overall conclusions about the financial statements. Satisfy doubts when questions arise about a client's ability to continue in existence.

Assist the auditor when forming overall conclusions about the financial statements.

If, after issuing an audit report, the auditors find that they have failed to perform certain significant audit procedures they should first: Attempt to determine whether their report is still being relied upon by third parties. Notify regulatory agencies. Notify legal counsel. Wait until the beginning of the next year's audit to determine whether misstatements have occurred.

Attempt to determine whether their report is still being relied upon by third parties.

Distinguish between attention-directing analytical procedures and those intended to eliminate or reduce detailed substantive procedures

Attention-directing analytical procedures occur when significant, unexpected differences are found between current year's unaudited financial data and other data used in comparisons. If an unusual difference is large, the auditor must determine the reason for it, and satisfy himself or herself that the cause is a valid economic event and not an error or misstatement due to fraud. Substantive analytical procedures are designed to reduce or eliminate detailed substantive tests. The effectiveness of these procedures in providing substantive evidence depends on the predictability of the relationship and the reliability of underlying data used to support the calculation.

Independence is required of a CPA performing: Audits, but not any other professional services. Attestation services, but not other professional services. Attestation and tax services, but not other professional services. All professional services.

Attestation services, but not other professional services.

List and define the assertions about account balances at the period end.

Audit evidence is the information used by the auditor in arriving at the conclusions on which the audit opinion is based, and includes the information contained in the accounting records underlying the financial statements and other information. Accounting records include the records of initial entries and supporting records, such as checks and records of electronic fund transfers; invoices; journal entries, and other adjustments to the financial statements that are not reflected in formal journal entries; and records such as work sheets and spreadsheets supporting cost allocations, computations, reconciliations, and disclosures. Many times the entries in the accounting records are initiated, recorded, processed, and reported in electronic form. Other information is audit evidence that includes minutes of meetings; confirmations from third parties; industry analysts' reports; comparable data about competitors (benchmarking); controls manuals; information obtained by the auditor from such audit procedures as inquiry, observation, and inspection; and other information developed by, or available to, the auditor that permits the auditor to reach conclusions through valid reasoning.

After relevant assertions have been identified, the auditor can then develop ______________- for each category of assertions.

Audit objectives

Management assertions lead to the __________________-. Therefore, auditors must have a thorough understanding of management assertions to perform quality audits.

Audit objectives

An auditor is concerned with completing various phases of the examination after the balance sheet date. This "subsequent period" involving formal audit procedures extends to the date of the A. Public issuance of the financial statements B. Final review of the audit working papers C. Auditor's report D. Delivery of the auditor's report to the entity

Auditor's report

Explain why the auditor divides the financial statements into components or segments in order to test management's assertions.

Auditors typically divide the financial statements into components or segments in order to make the audit more manageable. A component can be a financial statement account or a business (transaction cycle) process. This approach allows the auditor to gather evidence by examining the processing of related transactions through the accounting system from their origin to their ultimate disposition in the accounting journals and ledgers. Thus, the auditor can examine an accounting transaction from the time it is initiated by the entity until its final recording in the financial statement accounts.

Explain why auditors' reports are important to users of financial statements and why it is desirable to have standard wording.

Auditors' reports are important to users of financial statements because they inform users of the auditor's opinion as to whether or not the financial statements are fairly stated or whether no conclusion can be made with regard to the fairness of their presentation. Users especially look for any deviation from the wording of the standard unmodified report and the reasons and implications of such deviations. Having standard wording improves communications for the benefit of users of the auditor's report. When there are departures from the standard wording, users are more likely to recognize and consider situations requiring a modification or qualification to the auditor's report or opinion.

Who may identify matters to be included in a letter of inquiry sent to a client's legal counsel? Auditors Company mgmt-

Auditors-Yes Company mgmt-Yes

An example of an internal control weakness is to assign the payroll department the responsibility for: Preparing the payroll expense distribution. Preparing the payroll checks. Authorizing increases in pay. Preparing journal entries for payroll expense.

Authorizing increases in pay.

Tendency to consider info that is easily retrievable or what's easily accessible as being more likely or more relevant

Availability

Which of the following subsequent events is most likely to result in an ADJUSTMENT to a company's financial statements? A) Merger or acquisition activities B) Bankruptcy (due to deteriorating financial condition) of a customer with an outstanding accounts receivable balance C) Issuance of common stock D) An uninsured loss of inventories due to a fire

B) Bankruptcy (due to deteriorating financial condition) of a customer with an outstanding accounts receivable balance

The date of the management representation letter received from the client should coincide with which of the following? A) Date of latest subsequent event disclosed in the notes to the financial statements B) Date of the auditor's report C) Balance sheet date D) Engagement agreement

B) Date of the auditor's report

Auditors will generally send a standard inquiry letter to: A) only those attorneys who have devoted substantial time to client matters during the year. B) every attorney that the client has been involved with in the current or preceding year, plus any attorney the client engages on occasion. C) those attorneys whom the client relies on for advice related to substantial legal matters. D) only the attorney who represents the client in proceeding where the client is defendant

B) EVERY ATTORNEY that the client has been involved with in the current or preceding year, plus any attorney the client engages on occasion.

Which of the following is NOT a common audit procedure used to search for contingent liabilities? A) Examine letters of credit. B) Examine payroll reports. C) Review internal revenue agent reports. D) Analyze legal expense

B) Examine PAYROLL reports.

If the auditor determines that a subsequent event that affects the current period financial statements occurred after fieldwork was completed but before the audit report was issued, what date(s) may the auditor use on the report? - The date of the original last day of fieldwork only. - The date of the subsequent event only. - The date on which the last day of fieldwork occurred along with the date of the subsequent event. A) Yes Yes No B) No Yes Yes C) No Yes No D) No No Yes

B) FIELDWORK ONLY (NO) EVENT DATE (YES) FIELDWORK DATE AND EVENT DATE (YES)

Management furnishes the independent auditor with information concerning litigation, claims, and assessments. Which of the following is the auditor's primary means of initiating action to CORROBORATE such information? A) Request that client lawyers undertake a reconsideration of matters of litigation, claims, and assessments with which they were consulted during the period under examination. B) Request that client management send a letter of inquiry to those lawyers with whom management consulted concerning litigation, claims, and assessments. C) Request that client lawyers provide a legal opinion concerning the policies and procedures adopted by management to identify, evaluate, and account for litigation, claims, and assessments. D) Request that client management engage outside attorneys to suggest wording for the text of a footnote explaining the nature and probable outcome of existing litigation, claims, and assessments

B) Request that client MANAGEMENT SEND A LETTER of inquiry to those lawyers with whom management consulted concerning litigation, claims, and assessments.

Management furnishes the independent auditor with information concerning litigation, claims, and assessments. Which of the following is the auditor's primary means of initiating action to corroborate such information? A) Request that client lawyers undertake a reconsideration of matters of litigation, claims, and assessments with which they were consulted during the period under examination. B) Request that client management send a letter of inquiry to those lawyers with whom management consulted concerning litigation, claims, and assessments. C) Request that client lawyers provide a legal opinion concerning the policies and procedures adopted by management to identify, evaluate, and account for litigation, claims, and assessments. D) Request that client management engage outside attorneys to suggest wording for the text of a footnote explaining the nature and probable outcome of existing litigation, claims, and assessments.

B) Request that client management send a letter of inquiry to those lawyers with whom management consulted concerning litigation, claims, and assessments.

Elise-Greer, LLP is an affiliate of the audit client and is audited by another firm of auditors. Which of the following is most likely to be used by the auditor to obtain assurance that all guarantees of the affiliate's indebtedness have been detected? A) Send the standard bank confirmation request to all of the client's lender banks. B) Review client minutes and obtain a representation letter. C) Examine supporting documents for all entries in intercompany accounts. D) Obtain written confirmation of indebtedness from the auditor of the affiliate.

B) Review client minutes and obtain a representation letter.

The auditor's responsibility with respect to events occurring between the balance sheet date and the end of the audit examination is best expressed by which of the following statements? A) The auditor is fully responsible for events occurring in the subsequent period and should extend all detailed procedures through the last day of fieldwork. B) The auditor is responsible for determining that a proper cutoff has been made and performing a general review of events occurring in the subsequent period. C) The auditor's responsibility is to determine that a proper cutoff has been made and that transactions recorded on or before the balance sheet date actually occurred. D) The auditor has no responsibility for events occurring in the subsequent period unless these events affect transactions recorded on or before the balance sheet date

B) The auditor is responsible for determining that a PROPER CUTOFF has been made and performing a GENERAL REVIEW of events occurring in the subsequent period.

The auditor's responsibility with respect to events occurring between the balance sheet date and the end of the audit examination is best expressed by which of the following statements? A) The auditor is fully responsible for events occurring in the subsequent period and should extend all detailed procedures through the last day of fieldwork. B) The auditor is responsible for determining that a proper cutoff has been made and performing a general review of events occurring in the subsequent period. C) The auditor's responsibility is to determine that a proper cutoff has been made and that transactions recorded on or before the balance sheet date actually occurred. D) The auditor has no responsibility for events occurring in the subsequent period unless these events affect transactions recorded on or before the balance sheet date.

B) The auditor is responsible for determining that a proper cutoff has been made and performing a general review of events occurring in the subsequent period.

Which of the following subsequent events is most likely to result in an adjustment to a company's financial statements? A) merger or acquisition activities B) bankruptcy (due to deteriorating financial condition) of a customer with an outstanding accounts receivable balance C) issuance of common stock D) an uninsured loss of inventories due to a fire

B) bankruptcy (due to deteriorating financial condition) of a customer with an outstanding accounts receivable balance

Auditors, as part of completing the audit, will request the client to send a letter of inquiry to those attorneys the company has been consulting with during the year under audit regarding legal matters of concern to the company. The primary reason the auditor requests this information is to: A) determine the range of probable loss for asserted claims. B) corroborate of information supplied by management concerning litigation, claims, and assessments. C) outside opinion of probability of losses in determining accruals for contingencies. D) outside opinion of probability of losses in determining the proper footnote disclosure

B) corroborate of information supplied by management concerning litigation, claims, and assessments.

Auditors, as part of completing the audit, will request the client to send a letter of inquiry to those attorneys the company has been consulting with during the year under audit regarding legal matters of concern to the company. The primary reason the auditor requests this information is to: A) determine the range of probable loss for asserted claims. B) corroborate of information supplied by management concerning litigation, claims, and assessments. C) outside opinion of probability of losses in determining accruals for contingencies. D) outside opinion of probability of losses in determining the proper footnote disclosure.

B) corroborate of information supplied by management concerning litigation, claims, and assessments.

The date of the management representation letter received from the client should coincide with which of the following? A) date of latest subsequent event disclosed in the notes to the financial statements B) date of the auditor's report C) balance sheet date D) engagement agreement

B) date of the auditor's report

The letter of representation obtained from an audit client should be: A) dated as of the end of the period under audit. B) dated as of the audit report date. C) dated as of any date decided upon by the client and auditor. D) dated as of the issuance of the financial statement.

B) dated as of the audit report date.

Whenever subsequent events are used to evaluate the amounts included in the statements, care must be taken to distinguish between conditions that existed at the balance sheet date and those that come into being after the balance sheet date. The subsequent information should not be incorporated directly into the statements if the conditions causing the change in valuation: A) took place before the balance sheet date. B) did not take place until after the balance sheet date. C) occurred both before and after the balance sheet date. D) are reimbursable through insurance policies

B) did not take place until after the balance sheet date.

Whenever subsequent events are used to evaluate the amounts included in the statements, care must be taken to distinguish between conditions that existed at the balance sheet date and those that come into being after the end of the year. The subsequent information should not be incorporated directly into the statements if the conditions causing the change in valuation: A) took place before year-end. B) did not take place until after year-end. C) occurred both before and after year-end. D) are reimbursable through insurance policies.

B) did not take place until after year-end.

You are auditing Rodgers and Company. You are aware of a potential loss due to non-compliance with environmental regulations. Management has assessed that there is a 40% chance that a $10M payment could result from the non-compliance. The appropriate financial statement treatment is to: A) accrue a $4 million liability. B) disclose a liability and provide a range of outcomes. C) since there is less than a 50% chance of occurrence, ignore. D) since there is greater that a remote chance of occurrence, accrue the $10 million

B) disclose a liability and provide a range of outcomes.

You are auditing Rodgers and Company. You are aware of a potential loss due to non-compliance with environmental regulations. Management has assessed that there is a 40% chance that a $10M payment could result from the non-compliance. The appropriate financial statement treatment is to: A) accrue a $4 million liability. B) disclose a liability and provide a range of outcomes. C) since there is less than a 50% chance of occurrence, ignore. D) since there is greater that a remote chance of occurrence, accrue the $10 million.

B) disclose a liability and provide a range of outcomes.

Subsequent events affecting the realization of assets ordinarily will require adjustments of the financial statements under examination because such events typically represent the: A) culmination of conditions that existed at the balance sheet date. B) discovery of new conditions occurring in the subsequent events period. C) final estimates of losses relating to casualties occurring in the subsequent events period. D) preliminary estimate of losses relating to new events that occurred subsequent to the balance sheet date.

B) discovery of new conditions occurring in the subsequent events period.

The general cash account is considered a significant account in almost all audits A) where the ending balance is material. B) even when the ending balance is immaterial. C) except those of not-for-profit organizations. D) where either the beginning or ending balance is material.

B) even when the ending balance is immaterial.

1) Auditors will generally send a standard inquiry letter to: A) only those attorneys who have devoted substantial time to client matters during the year. B) every attorney that the client has been involved with in the current or preceding year, plus any attorney the client engages on occasion. C) those attorneys whom the client relies on for advice related to substantial legal matters. D) only the attorney who represents the client in proceeding where the client is defendant.

B) every attorney that the client has been involved with in the current or preceding year, plus any attorney the client engages on occasion.

The auditor's primary concerns relative to presentation and disclosure-related objectives is: A) accuracy. B) existence. C) completeness. D) occurrence.

B) existence.

If a potential loss on a contingent liability is REMOTE, the liability usually is: A) disclosed in footnotes, but not accrued. B) neither accrued nor disclosed in footnotes. C) accrued and indicated in the body of the financial statements. D) disclosed in the auditor's report but not disclosed on the financial statements

B) neither accrued nor disclosed in footnotes.

If a potential loss on a contingent liability is remote, the liability usually is: A) disclosed in footnotes, but not accrued. B) neither accrued nor disclosed in footnotes. C) accrued and indicated in the body of the financial statements. D) disclosed in the auditor's report but not disclosed on the financial statements.

B) neither accrued nor disclosed in footnotes.

The process of "final evidence accumulation" is always done late in the engagement. Which one of the following would be done the earliest in the engagement? A) final analytical procedures B) search for contingent liabilities C) evaluate the going concern assumption D) acquire the client's letter of representation

B) search for contingent liabilities

When dealing with contingencies: A) all material contingencies must be disclosed or footnoted. B) the auditor must exercise considerable professional judgment when evaluating whether the client has applied the appropriate treatment. C) it is easy for the auditor to uncover contingencies without management's cooperation. D) the review for contingent liabilities is only performed at the beginning and the end of the audit

B) the auditor must exercise considerable PROFESSIONAL JUDGEMENT when evaluating whether the client has applied the appropriate treatment.

An attorney is AWARE of a violation of a patent agreement that could result in a significant loss to the client IF IT WERE KNOWN. This is an example of a(n): A) commitment. B) unasserted claim. C) pending litigation. D) subsequent event

B) unasserted claim.

A CPA has received an attorney's letter in which no significant disagreements with the client's assessments of contingent liabilities were noted. The resignation of the client's lawyer shortly after receipt of the letter should alert the auditor that: A) an adverse opinion will be necessary. B) undisclosed unasserted claims may have arisen. C) the auditor must begin a completely new examination of contingent liabilities. D) the attorney was unable to form a conclusion with respect to the significance of litigation, claims, and assessments.

B) undisclosed unasserted claims may have arisen.

2. An entity erroneously recorded a large purchase twice. Which of the following internal controls would be most likely to detect this error in a timely and efficient manner? A. Footing the purchases journal. B. Reconciling vendors' monthly statements with subsidiary payable ledger accounts. C. Tracing totals from the purchases journal to the ledger accounts. D. Sending written quarterly confirmations to all vendors.

B. Reconciling vendors' monthly statements with subsidiary payable ledger accounts.

24. A purchase transaction usually begins with: A. A purchase order. B. A purchase requisition. C. A vendor invoice. D. A receiving report

B. A purchase requisition.

37. If the ratio of repairs and maintenance expense to property, plant, and equipment is higher than expected, which of the following is a plausible explanation? A. Routine maintenance on an important piece of machinery was charged to repairs and maintenance expense. B. An addition to a building was charged to repairs and maintenance expense. C. The company forgot to depreciate all of its equipment. D. The company purchased an unusual amount of new equipment.

B. An addition to a building was charged to repairs and maintenance expense.

1 . The occurrence assertion for accounts payable includes: A. Determining whether all accounts payable are recorded. B. Determining whether all accounts payable actually are liabilities. C. Determining whether all accounts payable are recorded in the proper period. D. Determining whether all accounts payable are properly classified in the financial statements.

B. Determining whether all accounts payable actually are liabilities.

28. The occurrence assertion for accounts payable includes: A. Determining whether all accounts payable are recorded. B. Determining whether all accounts payable actually are liabilities. C. Determining whether all accounts payable are recorded in the proper period. D. Determining whether all accounts payable are properly classified in the financial statements.

B. Determining whether all accounts payable actually are liabilities.

35. In testing plant and equipment balances, an auditor examines new additions listed on an analysis of plant and equipment. This procedure most likely obtains evidence concerning management's assertion of A. Completeness. B. Occurrence. C. Classification. D. Accuracy.

B. Occurrence.

14. Which of the following is not one of the major steps in setting control risk for the purchasing process? A. Understand and document the purchasing process. B. Plan and perform analytical procedures on accounts used in the purchasing process. C. Plan and perform tests of controls on purchase transactions. D. Set and document the control risk for the purchasing process.

B. Plan and perform analytical procedures on accounts used in the purchasing process.

19. With respect to a small company's system of purchasing supplies, an auditor's primary concern should be to obtain satisfaction that supplies ordered and paid for have been A. Requested and approved by authorized individuals who have no incompatible duties. B. Received, counted, and checked to quantities and amounts on purchase orders and invoices. C. Properly recorded as assets and systematically amortized over the estimated useful life of the supplies. D. Used in the course of business and solely for business purposes during the year under audit.

B. Received, counted, and checked to quantities and amounts on purchase orders and invoices.

29. Which of the following point about substantive test of transactions is incorrect? A. Substantive test of transactions are tests conducted to detect monetary misstatements in the individual transactions processed through all accounting applications. B. Substantive test of transactions focus mainly on the purchases and cash disbursement transactions. C. Substantive test of transactions concentrate on the detailed amounts or estimates. D. Substantive test of transactions can provide evidence about the fairness of the recorded account.

B. Substantive test of transactions focus mainly on the purchases and cash disbursement transactions.

33. Complex accounting issues for property, plant, and equipment include all of the following, except: A. Lease accounting. B. Testing goodwill for impairment. C. Capitalized interest. D. Self-constructed assets.

B. Testing goodwill for impairment.

Follow from management assertions and they provide a framework to help the auditor accumulate sufficient appropriate evidence related to account balances

Balance related and presentation and disclosure-related audit objectives

1. Park, CPA, was engaged to audit the financial statements of Tech Co., a new client, for the year ended December 31, 2001. Park obtained sufficient audit evidence for all of Tech's financial statement items except Tech's opening inventory. Due to inadequate financial records, Park could not verify Tech's January 1, 2001 inventory balances. Park's opinion on Tech's 2001 financial statements most likely will be

Balance sheet unqualified and Income statement Disclaimer

For a particular entity's financial statements to be presented fairly in conformity with generally accepted accounting principles, it is not required that the principles selected: Be appropriate in the circumstances for the particular entity. Reflect transactions in a manner that presents the financial statements within a range of acceptable limits. Present information in the financial statements that is classified and summarized in a reasonable manner. Be applied on a basis consistent with those followed in the prior year.

Be applied on a basis consistent with those followed in the prior year.

When no independent stock transfer agent is employed and the corporation issues its own stocks and maintains stock records, canceled stock certificates should: Be defaced to prevent reissuance and attached to their corresponding stubs. Not be defaced but segregated from other stock. Be destroyed to prevent fraudulent reissuance. Be defaced and sent to the Secretary of State.

Be defaced to prevent reissuance and attached to their corresponding stubs.

20. The cash disbursements journal is also called the A. voucher register. B. purchases journal. C. check register. D. accounts payable subsidiary ledger.

C

26. Tests designed to detect purchases made before the end of the year that have been recorded in the subsequent year most likely would provide assurance about management's assertion of A. accuracy. B. occurrence. C. cutoff. D. classification.

C

29. Which of the following procedures relating to the examination of accounts payable could the auditor delegate entirely to the entity's employees? A. Test footings in the accounts payable ledger. B. Reconcile unpaid invoices to vendors' statements. C. Prepare a schedule of accounts payable. D. Mail confirmations for selected account balances.

C

41. An auditor wishes to perform tests of controls on an entity's cash disbursements procedures. If the control activities leave no audit trail of documentary evidence, the auditor most likely will test the procedures by A. inquiry and analytical procedures. B. confirmation and observation. C. observation and inquiry. D. analytical procedures and confirmation.

C

45. When an auditor selects a sample of items from the vouchers payable register for the last month of the period under audit and traces these items to underlying documents, the auditor is gathering evidence primarily in support of the assertion that A. recorded obligations were paid. B. incurred obligations were recorded in the correct period. C. recorded obligations were valid. D. cash disbursements were recorded as incurred obligations.

C

46. An auditor traced a sample of purchase orders and the related receiving reports to the purchases journal and the cash disbursements journal. The purpose of this substantive procedure most likely was to A. identify unusually large purchases that should be investigated further. B. verify that cash disbursements were for goods actually received. C. determine that purchases were properly recorded. D. test whether payments were for goods actually ordered.

C

49. Substantive procedures to examine the cutoff assertion for accounts payable include A. selecting a sample of vouchers and agreeing them to authorized purchase orders. B. selecting a sample of vouchers and agreeing them to the purchases journal. C. selecting a sample of receiving reports around year-end and comparing dates on related vouchers to dates in the purchases journal. D. recomputing the mathematical accuracy of a sample of vendor invoices.

C

51. When searching for unrecorded liabilities at year-end, the population identified for sampling would be A. cash receipts from related parties recorded before year-end. B. creditors whose accounts appear on a subsidiary trial balance of accounts payable. C. cash disbursements recorded in the period subsequent to year-end. D. invoices dated a few days before and after year-end.

C

53. Which of the following procedures would an auditor most likely perform in searching for unrecorded liabilities? A. Trace a sample of accounts payable entries recorded just before year-end to the unmatched receiving report file. B. Compare a sample of purchase orders issued just after year-end with the year-end accounts payable trial balance. C. Vouch a sample of cash disbursements recorded just after year-end to receiving reports and vendor invoices. D. Scan the cash disbursements entries recorded just before year-end for indications of unusual transactions.

C

60. Which of the following control activities is not usually performed in the accounts payable department? A. Determining the mathematical accuracy of the vendor's invoice. B. Having an authorized person approve the voucher. C. Controlling the mailing of the check and remittance advice. D. Matching the receiving report with the purchase order.

C

63. Assertions about classes of transactions and events for the period under audit include A. existence, completeness, and accuracy. B. existence, completeness, and classification. C. occurrence, completeness, and cutoff. D. occurrence, completeness, and valuation and allocation.

C

64. Assertions about account balances at the period end include A. existence, completeness, and accuracy. B. existence, completeness, and classification. C. existence, rights and obligations, and completeness. D. existence, rights and obligations, and classification.

C

69. Which of the following describes a permanent difference? A. A difference that will be corrected in an amended tax return. B. A difference arising from an uncertain tax position. C. A fundamental difference in what constitutes revenue or expense for GAAP and tax purposes. D. A timing difference between the recognition of revenue or expense under GAAP and tax purposes.

C

With which of the following client personnel would it generally not be appropriate to inquire about commitments or contingent liabilities? A) Controller B) President C) Accounts receivable clerk D) Vice president of sales

C) Accounts receivable clerk

Which of the following groups has the responsibility for IDENTIFYING and DECIDING the appropriate accounting treatment for recording or disclosing contingent liabilities? A) Auditors B) Legal counsel C) Management D) Management and the auditors

C) Management

What needs to be INCLUDED in a letter of inquiry sent to a client's LEGAL counsel? Any pending threatened litigation with which the attorney has had significant involvement The amount of legal fees paid by the client to the attorney A) Yes Yes B) No No C) Yes No D) No Yes

C) PENDING (YES) AMOUNT (NO)

Which of the following material events occurring subsequent to the balance sheet date WOULD require an ADJUSTMENT to the financial statements before they could be issued? A) Loss of a plant as a result of a flood B) Sale of long-term debt or capital stock C) Settlement of litigation in excess of the recorded liability D) Major purchase of a business that is expected to double the sales volume

C) Settlement of litigation in excess of the recorded liability

The audit procedures for the subsequent events review can be divided into two categories: (1) procedures integrated as a part of the verification of year-end account balances, and (2) those performed specifically for the purpose of discovering subsequent events. Which of the following procedures is in the FIRST category? A) Inquire of client regarding contingent liabilities. B) Obtain a letter of representation written by client. C) Subsequent period sales and purchases transactions are examined to determine whether the cutoff is accurate. D) Review journals and ledgers of year 2 to determine the existence of any transactions related to year 1

C) Subsequent period sales and purchases transactions are examined to determine whether the CUTOFF is accurate.

The audit procedures for the subsequent events review can be divided into two categories: (1) procedures integrated as a part of the verification of year-end account balances, and (2) those performed specifically for the purpose of discovering subsequent events. Which of the following procedures is in category 1? A) Inquiries of client regarding contingent liabilities. B) Obtain a letter of representation written by client. C) Subsequent period sales and purchases transactions are examined to determine whether the cutoff is accurate. D) Review journals and ledgers of year 2 to determine the existence of any transaction related to year 1.

C) Subsequent period sales and purchases transactions are examined to determine whether the cutoff is accurate.

An auditor must obtain written client representations that might be signed by all but which of the following? A) Treasurer B) Chief financial officer C) Vice president of operations D) Chief executive officer

C) Vice president of operations

A client has a calendar year-end. Listed below are four events that occurred after December 31. Which one of these subsequent events might result in ADJUSTMENT of the December 31 financial statements? A) Sale of a major subsidiary B) Adoption of accelerated depreciation methods C) Write-off of a substantial portion of inventory as obsolete D) Collection of 90% of the accounts receivable existing at December 31

C) Write-off of a substantial portion of inventory as obsolete

Which event that occurred after the end of the fiscal year under audit but prior to issuance of the auditor's report would not require disclosure in the financial statements? A) sale of a bond or capital stock issue B) loss of plant or inventories as a result of fire or flood C) a significant decline in the market price of the corporation's stock D) settlement of litigation when the event giving rise to the claim took place after the balance sheet date

C) a significant decline in the market price of the corporation's stock

While there is no professional requirement to do so on audit engagements, CPAs frequently issue a formal "management" letter to clients. The primary purpose of this letter is to provide: A) evidence indicating whether the auditor is reasonably certain that internal accounting control is operating as prescribed. B) a permanent record of the internal accounting control work performed by the auditor during the course of the engagement. C) a written record of discussions between auditor and client concerning the auditor's observations and suggestions for improvements. D) a summary of the auditor's observations that resulted from the auditor's special study of internal control.

C) a written record of discussions between auditor and client concerning the auditor's observations and suggestions for improvements.

A significant customer of the firm suffers a large economic loss after year end, but prior to completion of field work. The audit client believes this event will have material effect on the financial statements. The auditor should: A) adjust the financial statements for the year under audit. B) add a paragraph to the audit report. C) advise the client to disclose the event in the notes to the financial statements. D) advise the client to delay issuing the financial statements until the economic loss can be determined.

C) advise the client to disclose the event in the notes to the financial statements.

An auditor has the responsibility to actively search for subsequent events that occur subsequent to the: A) balance sheet date. B) date of the auditor's report. C) balance sheet date, but prior to the audit report. D) date of the management representation letter

C) balance sheet date, but prior to the audit report.

An auditor has the responsibility to actively search for subsequent events that occur subsequent to the: A) balance sheet date. B) date of the auditor's report. C) balance sheet date, but prior to the audit report. D) date of the management representation letter.

C) balance sheet date, but prior to the audit report.

The auditor's primary concern relative to PRESENTATION and disclosure-related objectives is: A) accuracy. B) existence. C) completeness. D) occurrence

C) completeness.

Which of the following groups has the responsibility for identifying and deciding the appropriate accounting treatment for recording or disclosing contingent liabilities? A) auditors B) legal counsel C) management D) management and the auditors

C) management

Which of the following would the auditor expect to find in the client's management representation letter? A) management's recommendations for internal control effectiveness improvements B) management's plans for improving product quality C) management's compliance with contractual arrangements that impact the financial statements D) management's goals for improving earnings per share

C) management's compliance with contractual arrangements that impact the financial statements

The auditor is responsible for communicating significant internal control deficiencies to the audit committee, or those charged with governance. This communication: A) may be oral or written. B) must be oral. C) must be written. D) must be oral via direct communication.

C) must be written.

The audit firm issues an audit report for its client. The auditor's have NO obligation to make further inquiries with respect to the client's audited financial statements unless: A) a development occurs that may affect the company's long term viability as a company. B) final resolution was made on disclosed contingency for which no liability needed to be accrued. C) new information comes to the auditor's attention concerning an event that occurred prior to the date of the audit report that, if known, would have impacted the audit opinion. D) a lawsuit, in which the risk of loss was considered remote, was resolved in the company's favor.

C) new information comes to the auditor's attention concerning an event that occurred prior to the date of the audit report that, if known, would have impacted the audit opinion.

Which of the following auditing procedures is ordinarily performed last? A) reading minutes of the board of directors' meetings B) confirming accounts payable C) obtaining a client representation letter D) testing the purchasing function

C) obtaining a client representation letter

Auditing standards (SAS No. 59) requires the auditor to evaluate whether there is a substantial doubt about a client's ability to continue as a going concern for at least: A) one quarter beyond the balance sheet date. B) one quarter beyond the date of the auditor's report. C) one year beyond the balance sheet date. D) one year beyond the date of the auditor's report.

C) one year beyond the balance sheet date.

One of the primary approaches in dealing with UNCERTAINTIES in loss contingencies uses a ________ threshold. A) monetary B) materiality C) probability D) analytical

C) probability

Which of the following material events occurring subsequent to the balance sheet date would require an adjustment to the financial statements before they could be issued? A) loss of a plant as a result of a flood B) sale of long-term debt or capital stock C) settlement of litigation in excess of the recorded liability D) major purchase of a business that is expected to double the sales volume

C) settlement of litigation in excess of the recorded liability

A client representation letter is: A) prepared on the CPA's letterhead. B) addressed to the client. C) signed by high-level officials (e.g., the president and chief financial officer). D) dated as of the client's year-end.

C) signed by high-level officials (e.g., the president and chief financial officer).

When should auditors generally assess a client's ability to continue as a going concern? A) upon completion of the audit B) during the planning stages of the audit C) throughout the entire audit process D) during testing and completion phases of the audit

C) throughout the entire audit process

A client has a calendar year-end. Listed below are four events that occurred after December 31. Which one of these subsequent events might result in adjustment of the December 31 financial statements? A) sale of a major subsidiary B) adoption of accelerated depreciation methods C) write-off of a substantial portion of inventory as obsolete D) collection of 90% of the accounts receivable existing at December 31

C) write-off of a substantial portion of inventory as obsolete

PCAOB standards describe the 5 categories of management assertions:

CEO APROVED Completeness Existence/occurrence Occurrence/existence Allocation/valuation Presentation and disclosure Rights and obligation Obligation and rights Valuation/allocation E(WASH) Disclosure and presentation

23. An entity's purchase documents include all of the following EXCEPT: A. Purchase requisition. B. Accounts payable subsidiary ledger. C. Cash receipts journal. D. Vendor statement.

C. Cash receipts journal.

7. In auditing accounts payable, an auditor's procedures most likely would focus primarily on management's assertion of A. Existence B. Rights and obligations. C. Completeness. D. Valuation and allocation.

C. Completeness.

4. Tests designed to detect purchases made before the end of the year that have been recorded in the subsequent year most likely would provide assurance about management's assertion of A. Accuracy. B. Occurrence. C. Cutoff. D. Classification.

C. Cutoff.

5. An auditor traced a sample of purchase orders and the related receiving reports to the purchases journal and the cash disbursements journal. The purpose of this substantive procedure most likely was to A. Identify unusually large purchases that should be investigated further. B. Verify that cash disbursements were for goods actually received. C. Determine that purchases were properly recorded. D. Test whether payments were for goods actually ordered.

C. Determine that purchases were properly recorded.

10. The accounts payable department receives the purchase order form to accomplish all of the following except to: A. Compare invoice price to purchase order price. B. Ensure that the purchase had been properly authorized. C. Ensure that the goods had been received by the party requesting the goods. D. Compare quantity ordered to quantity purchased.

C. Ensure that the goods had been received by the party requesting the goods.

20. Which of the following is the most effective control activity to detect vouchers prepared for the payment of goods that were not received? A. Counting of goods upon receipt in the storeroom. B. Comparison of goods received with goods requisitioned in the receiving department. C. Matching of purchase order, receiving report, and vendor invoice for each voucher in the accounts payable department. D. Verification of vouchers for accuracy and approval in the internal audit department.

C. Matching of purchase order, receiving report, and vendor invoice for each voucher in the accounts payable department.

8. Assertions about classes of transactions and events for the period under audit include: A. Existence, completeness, and accuracy. B. Existence, completeness, and classification. C. Occurrence, completeness, and cutoff. D. Occurrence, completeness, and valuation and allocation.

C. Occurrence, completeness, and cutoff.

25. The invoice processing function is responsible for A. Initiation and approval of requests for goods or services by authorized individuals. B. Recording all vendor invoices, cash disbursements, and adjustments in vendor accounts. C. Processing of vendor invoices for goods and services received. D. None of the above.

C. Processing of vendor invoices for goods and services received.

31. All the following are examples of disclosure items for purchasing process and related accounts, EXCEPT: A. Payables by type. B. Purchases from and payables to related parties. C. Short-term purchase contracts and usual purchase commitments. D. Dependence on a single vendor or a small number of vendors.

C. Short-term purchase contracts and usual purchase commitments.

16. Identify the assertion that is represented by the following statement: "Accounts payable and accrued expenses are included in the financial statements at appropriate amounts." A. Existence. B. Completeness. C. Valuation and allocation. D. Rights and Obligations.

C. Valuation and allocation.

34. Testing depreciation calculations for a sample of property, plant, and equipment tests the assertion of A. Existence. B. Completeness. C. Valuation and allocation. D. Rights and obligations.

C. Valuation and allocation.

15. When using confirmations to provide evidence about the completeness assertion for accounts payable, the appropriate population would most likely be A. Payees of checks written in the month after year-end. B. Those vendors with balances in the year-end accounts payable subsidiary ledger. C. Vendors with whom the entity has done business with during the year. D. Invoices filed in the entity's open (unpaid) invoice file at year-end.

C. Vendors with whom the entity has done business with during the year.

COVER-U Assertions relevant to Presentation and Disclosure

CVRU "C" Completeness: All disclosures that should have been included in the financial statements have been included. "V" Valuation, Allocation and Accuracy: Financial information and other information are disclosed fairly and at appropriate amounts. "R" Rights and Obligations and Occurrence: Disclosed events and transactions have occurred and pertain to the entity. "U" Understandability and Classification: Financial information and other information are disclosed fairly and at appropriate amounts.

Which of the following procedures would an auditor most likely perform while evaluating audit findings at the conclusion of an audit? Obtain assurance from the entity's attorney that all material litigation has been disclosed in the financial statements. Verify the clerical accuracy of the entity's proof of cash and its bank cutoff statement. Determine whether reportable conditions have been corrected. Calculate an estimate the total of uncorrected misstatements in the financial statements.

Calculate an estimate the total of uncorrected misstatements in the financial statements.

Adverse opinion

Can cause tock to be delisted. Used when the auditor believes that the overall financial statements are so materially misstated or misleading that they do not present fairly the financial position or results of operations and cash flows in conformity with GAAP

An entity's internal control requires that for every check request there be an approved voucher, supported by a prenumbered purchase order and a prenumbered receiving report. To determine whether checks are being issued for unauthorized expenditures, an auditor most likely would select items for testing from he population all

Canceled chceks

drawbacks to non statistical method

Cannot project an error onto the population. exception: if you use a random selection process vs a haphazard process, then you can extrapolate.

When an auditor finds a debit to accounts payable, which of the following accounts is most likely to be credited? Accounts Receivable. Accrued liabilities. Cash. Cost of goods sold.

Cash.

Distinguish between changes that affect consistency and those that may affect comparability but not consistency. Give an example of each.

Changes that affect the consistency of the financial statements may involve any of the following: a.Change in accounting principle b.Change in reporting entity c.Corrections of errors involving accounting principles.

Which of the following is prohibited by the AICPA Code of Professional Conduct? Advertising in a general newspaper to attract clients. Auditing a competitor of the current audit client. Charging and accepting a contingent fee for a financial statement review engagement. Purchasing a product and reselling that product at a higher price.

Charging and accepting a contingent fee for a financial statement review engagement.

As an in-charge auditor, you are reviewing a summary of control weaknesses, standing alone, should cause you the least concern?

Check are signed by only one person.

Analytical Procedures

Comparing recorded accounting data with historical information, budget expectations or external benchmarks.

All assets, liabilities, and equity interest that should've been recorded have been recorded

Completeness

All disclosures that should've been included in the financial statements have been included

Completeness

All transactions and accounts that should be presented in the financial statements are so included

Completeness

All transactions and events that should've been recorded have been recorded

Completeness

Existing transactions are recorded

Completeness

In auditing accounts payable, and auditor's procedures most likely would focus primarily on management's assertion of

Completeness

Which of the following assertions is of principle concern to the auditors in the examination of accounts payable? Existence. Completeness. Valuation. Authorization.

Completeness.

If payables turnover has increased significantly since the prior year, this is an indication that which of the followign assertions for accounts payable might be violated?

Completenss

Probably the most important federal legislation governing activities involving computers is

Computer Fraud and Abuse Act of 1986

What are some of the major challenges facing the accounting profession:

Concern exists about fraudulent financial reporting because it can undermine the entire financial reporting process. Failure to provide information to users that is accurate can lead to inappropriate allocations of resources in our economy. In addition, failure to detect massive fraud can lead to additional governmental oversight of the accounting profession.

What procedure would an auditor most likely perform to assist in the evaluation of loss contingencies?

Obtain a letter of audit inquiry from the client's lawyer.

3 important factors for sampling risks

Confidence level Tolerable error Expected error

Tendency to put more weight on info that is consistent with initial beliefs/preferences

Confirmation

Which of the following is not a procedure that is designed to provide evidence about the existence of loss contingencies? Obtaining a lawyers' letter. Confirming accounts payable. Reviewing the minutes of board of directors' meetings. Review correspondence with banks.

Confirming accounts payable.

The refusal of an entity's attorney to provide a representation on the legality of a particular act committed by the entity is generally A. Sufficient reason to issue a "subject to" qualified opinion B. Proper grounds to withdraw from the engagement without further consideration C. Considered to be a scope limitation D. Insufficient reason to modify the auditor's report because of the attorney's obligation of confidentiality

Considered to be a scope limitation

When an auditor of a nonpublic company has concluded there is substantial doubt about an entity's ability to continue as a going concern for a reasonable period of time beyond the date the financial statements will be released (1/26/X2), the auditor's responsibility includes: Preparing prospective financial information to verify whether management's plans can be effectively implemented. Projecting conditions and events from one year prior to this year's date (12/31/X0) to 12/31/X1. Issuing an adverse or negative assurance opinion, depending upon materiality, due to the possible effects on the financial statements. Considering the adequacy of disclosure about the entity's possible inability to continue as a going concern.

Considering the adequacy of disclosure about the entity's possible inability to continue as a going concern.

Observation (Audit Procedures)

Consists of looking at a process or procedure being performed by others.

A principal auditor decides not to refer to the audit of another CPA who audited a subsidiary of the principal auditor's client. After making inquires about the other CPA's professional reputation and independence, the principal auditor most likely would

Contact the other CPA and review the audit programs and working papers pertaining to the subsidiary.

Which type of data flow diagram presents a high level view of a system process?

Context Diagram

Auditors often integrate procedures for PRESENTATION and DISCLOSURE objectives with: Tests for planning objectives Tests for balance-related objectives A) Yes Yes B) No No C) Yes No D) No Yes

D) Tests for PLANNING objectives (NO) Tests for BALANCE-related objectives (YES)

Distinguish between contingent liabilities and commitments.

Contingent liabilities are potential future obligations to an outside party for an unknown amount resulting from activities that have already taken place. Commitments are agreements to commit the company to a set of fixed conditions in the future regardless of what happens to profits or the economy as a whole.

A potential future obligation to an outside party for an unknown amount resulting from activities that have already taken place.

Contingent liability

Define the term contingent liability and discuss the criteria accountants and auditors use to classify these accounting events.

Contingent liability: a potential future obligation to an outside party for an unknown amount resulting from activities that have already taken place. Three conditions are required for a contingent liability to exist: (1) there is a potential future payment to an outside party or the impairment of an asset that resulted from an existing condition; (2) there is uncertainty about the amount for the future payment or impairment; and (3) the outcome will be resolved by some future event or events. Accounting standards describe three levels of likelihood of occurrence and the appropriate financial statement treatment for each likelihood as follows:. a. Probable-future event likely to occur and amount can be reasonably estimated then the financial statement accounts are adjusted. If amount cannot be reasonably estimated, then a footnote disclosure is necessary. b. Reasonably possible-chance of occurring is more than remote, but less than probable. Footnote disclosure is necessary. c. Remote-chance of occurrence is slight, no disclosure is necessary.

5 components of Coso framework

Control Environment, Entitys Risk Assessment Process, Control Activities, Information and Communication, and Monitoring Activities

What does it mean if there is an operation control deficiency?

Control doesnt operate as designed or person performing the control doesnt have the authority or qualification to perform the control effectively

What does it mean if there is a design control defiency?

Control is missing or improperly designed

Which of the following control activities is not usually performed in the accounts payable department?

Controlling the mailing of the check and remittance advice

An auditor requests a client to send letters of audit inquiry to attorneys who have been consulted concerning litigation, claims, and assessments. The primary reason for this request is to obtain what?

Corroboration of the information furnished by management concerning litigation, claims, and assessments.

Transactions and events have been recorded in the correct accounting period

Cutoff

What is the relationship among the financial statement cycle?

Cycles have no beginning or end except at the origin and final disposition of a company

12. Which of the following accounts is not affected by cash disbursement transactions? A. Cash. B. Accounts payable. C. Purchase discounts. D. Purchase returns.

D

34. A voucher A. is a bill from the vendor. B. is a document that records the receipt of goods. C. is a document that requests goods from an authorized individual in the entity. D. serves as the basis for recording a vendor's invoice in the purchases journal.

D

35. Operating control over the check signature plate normally should be the responsibility of the A. Secretary. B. Chief accountant. C. Vice President of Finance. D. Treasurer.

D

36. In testing controls over cash disbursements, an auditor most likely would determine that the person who signs the checks also A. reviews the monthly bank reconciliation. B. returns the checks to accounts payable. C. is denied access to the supporting documents. D. is responsible for mailing the checks.

D

39. Tests of controls for the occurrence assertion for purchases include all of the following except: A. evaluating proper segregation of duties. B. testing a sample of vouchers for an authorized purchase order. C. testing a sample of vouchers for matching receiving reports. D. tracing a sample of vouchers to purchases journal.

D

40. An internal control questionnaire indicates that an approved receiving report is required to accompany every check request for payment of merchandise. Which of the following procedures provides the greatest assurance that this control is operating effectively? A. Select and examine receiving reports and ascertain that the related canceled checks are dated no earlier than the receiving reports. B. Select and examine receiving reports and ascertain that the related canceled checks are dated no later than the receiving reports. C. Select and examine canceled checks and ascertain that the related receiving reports are dated no earlier than the checks. D. Select and examine canceled checks and ascertain that the related receiving reports are dated no later than the checks.

D

59. Budd, the purchasing agent for Lake Hardware Wholesalers, has a relative who owns a retail hardware store. Budd arranged for hardware to be delivered by manufacturers to the retail store on a C.O.D. basis, thereby enabling his relative to buy at Lake's wholesale prices. Budd was probably able to accomplish this because of Lake's poor internal control over A. purchase requisitions. B. cash receipts. C. perpetual inventory records. D. purchase orders.

D

Audit procedures related to CONTINGENT LIABILITIES are initially focused on: A) accuracy. B) completeness. C) existence. D) occurrence

D) occurrence

Audit procedures related to contingent liabilities are initially focused on: A) accuracy. B) completeness. C) existence. D) occurrence.

D) occurrence.

61. Which of the following is an internal control that would prevent a paid disbursement voucher from being presented for payment a second time? A. Vouchers should be prepared by individuals who are responsible for signing disbursement checks. B. Disbursement vouchers should be approved by at least two responsible management officials. C. The date on a disbursement voucher should be within a few days of the date the voucher is presented for payment. D. The official signing the check should compare the check with the voucher and should "cancel" the voucher documents by marking them "paid."

D

66. Which of the following questions would most likely be included in an internal control questionnaire concerning the completeness assertion for purchases? A. Is an authorized purchase order required before the receiving department can accept a shipment or the vouchers payable department can record a voucher? B. Are purchase requisitions prenumbered and independently matched with vendor invoices? C. Is the unpaid voucher file periodically reconciled with inventory records by an employee who does not have access to purchase requisitions? D. Are purchase orders, receiving reports, and vouchers prenumbered and periodically accounted for?

D

68. Which of the following describes a temporary difference? A. A difference that will be corrected in an amended tax return. B. A difference arising from an uncertain tax position. C. A fundamental difference in what constitutes revenue or expense for GAAP and tax purposes. D. A timing difference between the recognition of revenue or expense under GAAP and tax purposes.

D

Which of the following would be a subsequent discovery of facts which would NOT require a response by the auditor? A) Discovery of the inclusion of material nonexistent sales B) Discovery of the failure to write off material obsolete inventory C) Discovery of the omission of a material footnote D) Discovery of management's intent to increase selling prices in the future

D) Discovery of management's intent to increase selling prices in the future

Which of the following is not considered a commitment? A) Agreements to purchase raw materials B) Pension plans C) Agreements to lease facilities at set prices D) Each of the above is a commitment

D) Each of the above is a commitment

Commitments include all but which of the following? A) agreements to purchase raw materials B) pension plans C) agreements to lease facilities at set prices D) Each of the above is a commitment.

D) Each of the above is a commitment.

Which of the following statements regarding the letter of representation is not correct? A) It is prepared on the client's letterhead. B) It is addressed to the CPA firm. C) It is signed by high-level corporate officials, usually the president and chief financial officer. D) It is optional, not required, that the auditor obtain such a letter from management.

D) It is optional, not required, that the auditor obtain such a letter from management.

Which of the following is not a reason why the auditor requests that the client provide a letter of representation? A) Professional auditing standards require the auditor to obtain a letter of representation. B) It impresses upon management its responsibility for the accuracy of the information in the financial statements. C) It provides written documentation of the oral responses already received to inquiries of management. D) It provides written documentation, which is a higher quality of evidence than management's oral responses to inquiries.

D) It provides written documentation, which is a higher quality of evidence than management's oral responses to inquiries.

Inquiries of management regarding the possibility of UNRECORDED contingencies will be useful in uncovering: Management's intentional failure to disclose existing contingencies. When management does not comprehend accounting disclosure requirements. A) Yes Yes B) No No C) Yes No D) No Yes

D) Management's intentional failure to disclose existing contingencies. (No) When management does not comprehend accounting disclosure requirements. (Yes)

If the auditor concludes that there are contingent liabilities, he or she must evaluate the SIGNIFICANCE of the potential liability and the NATURE of the disclosure needed in the financial statements. Which of the following statements is NOT true? A) The potential liability is sufficiently well known in some instances to be included in the financial statements as an actual liability. B) Disclosure may be unnecessary if the contingency is highly remote or immaterial. C) A CPA firm often obtains a separate evaluation of the potential liability from its own legal counsel rather than relying on management or management's attorneys. D) The client's attorneys must remain independent when evaluating the likelihood of losing the lawsuit

D) The client's ATTORNEYS MUST REMAIN independent when evaluating the likelihood of losing the lawsuit

The auditor's responsibility for "REVIEWING the subsequent events" of a public company that is about to issue new securities is normally limited to the period of time: A) beginning with the balance sheet date and ending with the date of the auditor's report. B) beginning with the start of the fiscal year under audit and ending with the balance sheet date. C) beginning with the start of the fiscal year under audit and ending with the date of the auditor's report. D) beginning with the balance sheet date and ending with the date the registration statement becomes effective

D) beginning with the BALANCE SHEET date and ending with the date the REGISTRATION STATEMENT becomes effective

The auditor's responsibility for "reviewing the subsequent events" of a public company that is about to issue new securities is normally limited to the period of time: A) beginning with the balance sheet date and ending with the date of the auditor's report. B) beginning with the start of the fiscal year under audit and ending with the balance sheet date. C) beginning with the start of the fiscal year under audit and ending with the date of the auditor's report. D) beginning with the balance sheet date and ending with the date the registration statement becomes effective.

D) beginning with the balance sheet date and ending with the date the registration statement becomes effective.

The standard letter of inquiry to the client's legal counsel should be prepared on: A) plain paper (no letterhead) and be unsigned. B) lawyer's stationery and signed by the lawyer. C) auditor's stationery and signed by an audit partner. D) client's stationery and signed by a company official.

D) client's stationery and signed by a company official.

Which of the following is the most efficient audit procedure for the detection of unrecorded liabilities at the balance sheet date? A) obtain an attorney's letter from the client's attorney B) confirm large accounts payable balances at the balance sheet date C) examine purchase orders issued for several days prior to the close of the year D) compare cash disbursements in the subsequent period with the accounts payable trial balance at year-end

D) compare cash disbursements in the subsequent period with the accounts payable trial balance at year-end

Which of the following would be a subsequent discovery of facts which would not require a response by the auditor? A) discovery of the inclusion of material nonexistent sales B) discovery of the failure to write off material obsolete inventory C) discovery of the omission of a material footnote D) decrease in the value of investments

D) decrease in the value of investments

At the completion of the audit, management is asked to make a written statement that it is not aware of any undisclosed contingent liabilities. This statement would appear in the: A) management letter. B) letter of inquiry. C) letters testamentary. D) management letter of representation.

D) management letter of representation.

Which of the following is not one of the categories of items included in the client letter of representation? A) subsequent events B) Completeness of information C) recognition, measurement, and disclosure D) materiality

D) materiality

An attorney is responding to an independent auditor as a result of the client's letter of inquiry. The attorney may appropriately LIMIT the response to: A) asserted claims and litigation. B) asserted, overtly threatened, or pending claims and litigation. C) items which have an extremely high probability of being resolved to the client's detriment. D) matters to which the attorney has given substantive attention in the form of legal consultation or representation

D) matters to which the attorney has given SUBSTANTIVE attention in the form of legal consultation or representation

An attorney is responding to an independent auditor as a result of the client's letter of inquiry. The attorney may appropriately limit the response to: A) asserted claims and litigation. B) asserted, overtly threatened, or pending claims and litigation. C) items which have an extremely high probability of being resolved to the client's detriment. D) matters to which the attorney has given substantive attention in the form of legal consultation or representation.

D) matters to which the attorney has given substantive attention in the form of legal consultation or representation.

After an auditor has issued an audit report on a NONPUBLIC entity, there is no obligation to make any further audit tests or inquiries with respect to the audited financial statements covered by that report UNLESS: A) material adverse events occur after the date of the auditor's report. B) final determination or resolution was made of a contingency which had been disclosed in the financial statements. C) final determination or resolution was made on matters which had resulted in a qualification in the auditor's report. D) new information comes to the auditor's attention concerning an event that occurred prior to the date of the auditor's report that may have affected the auditor's report

D) new information comes to the auditor's attention concerning an event that occurred prior to the date of the auditor's report that may have affected the auditor's report

After an auditor has issued an audit report on a nonpublic entity, there is no obligation to make any further audit tests or inquiries with respect to the audited financial statements covered by that report unless: A) material adverse events occur after the date of the auditor's report. B) final determination or resolution was made of a contingency which had been disclosed in the financial statements. C) final determination or resolution was made on matters which had resulted in a qualification in the auditor's report. D) new information comes to the auditor's attention concerning an event that occurred prior to the date of the auditor's report that may have affected the auditor's report.

D) new information comes to the auditor's attention concerning an event that occurred prior to the date of the auditor's report that may have affected the auditor's report.

Auditing standards (SAS No. 99 and SAS No. 54) require the auditor to communicate all management frauds and illegal acts to the audit committee: A) only if the act is immaterial. B) only if the act is material. C) only if the act is highly material. D) regardless of materiality.

D) regardless of materiality.

The auditor has a responsibility to review transactions and activities occurring AFTER the balance sheet date to determine whether anything occurred that might affect the statements being audited. The procedures required to verify these transactions are commonly referred to as the review for: A) contingent liabilities. B) subsequent year's transactions. C) late unusual occurrences. D) subsequent events

D) subsequent events

The auditor has a responsibility to review transactions and activities occurring after the year-end to determine whether anything occurred that might affect the statements being audited. The procedures required to verify these transactions are commonly referred to as the review for: A) contingent liabilities. B) subsequent year's transactions. C) late unusual occurrences. D) subsequent events.

D) subsequent events.

Which of the following would the auditor find most useful in relation to its previous audit procedures from the client representation letter? A) to impress upon the audit firm its responsibility for the audit B) to impress upon management its responsibility for the financial statement assertions C) to remind management of potential misstatements or omissions in the financial statements D) to document the responses from management to inquiries about various aspects of the audit

D) to document the responses from management to inquiries about various aspects of the audit

13. All of the following are inherent risk factors for the purchasing process except: A. Whether the supply of raw materials is adequate. B. How volatile raw material prices are. C. Misstatements detected in prior audits. D. A new IT system placed in operation during the year.

D. A new IT system placed in operation during the year.

26. The following definition, that "amounts relating to recorded purchase have been recorded appropriately and properly accumulated from journals and ledgers" relates to which assertion? A. Occurence. B. Classification. C. Completeness. D. Accuracy.

D. Accuracy.

36. To achieve effective control over fixed asset additions, a company should establish activities that require A. Capitalization of the cost of fixed asset additions in excess of a specific dollar amount. B. Performance of recurring fixed asset maintenance work solely by maintenance department employees. C. Any fixed asset additions that are not used in the business to be classified as investments. D. Authorization and approval of major fixed asset additions.

D. Authorization and approval of major fixed asset additions.

3. Which of the following procedures would an auditor most likely perform in searching for unrecorded payables? A. Reconcile receiving reports with related cash payments made just prior to year-end. B. Contrast the ratio of accounts payable to purchases with the prior year's ratio. C. Vouch a sample of creditor balances to supporting invoices, receiving reports and purchase orders. D. Compare cash payments occurring after the balance sheet date with the accounts payable trial balance.

D. Compare cash payments occurring after the balance sheet date with the accounts payable trial balance.

27. Which of the following controls activities is NOT usually performed in the accounts payable department? A. Reception of properly authorized goods and services. B. Adjustments in individual vendor accounts. C. Indicating the asset accounts to be debited and the liability accounts to be credited. D. Processing of payments to vendors.

D. Processing of payments to vendors.

12. Which of the following controls would most effectively ensure that recorded purchases are free of material misstatements? A. The receiving department compares the quantity ordered on purchase orders with the quantity received on receiving reports. B. Vendor invoices are compared with purchase orders by an employee who is independent of the receiving department. C. Receiving reports require the signature of the individual who authorized the purchase. D. Purchase orders, receiving reports, and vendor invoices are independently matched when preparing vouchers.

D. Purchase orders, receiving reports, and vendor invoices are independently matched when preparing vouchers.

17. Auditors perform a test to verify that all merchandise received has been recorded, in part, to satisfy the completeness assertion with regard to accounts payable. Which of the following would represent the population of documents for this test? A. Purchase requisitions. B. Payment vouchers. C. Vendor invoices. D. Receiving reports.

D. Receiving reports.

18. Auditor confirmation of accounts payable balances at the balance sheet date may be unnecessary because? A. This is a duplication of cutoff tests. B. Accounts payable balances at the balance sheet date may not be paid before the audit is completed. C. Correspondence with the audit client's attorney will reveal all legal actions by vendors for nonpayment. D. There is likely to be other reliable external evidence to support the balances.

D. There is likely to be other reliable external evidence to support the balances.

The date of the management representation letter should coincide with the A. Date of the auditor's report B. Date of the engagement agreement C. Balance sheet date D. Date of the latest subsequent event referred to in the notes to the financial statements

Date of the auditor's report

The task of providing greater detail in successive levels of data flow diagrams is called

Decomposition

An auditor's opinion enhances the _______________ that intended users can place in financial statements

Degree of confidence

An auditor concludes that a client's illegal act, which has a material effect on the financial statements, has not been properly accounted for or disclosed.

Depending on the materiality of the effect on the financial statements, the auditor should express either a qualified opinion or an adverse opinion.

To minimize the opportunities for fraud, unclaimed cash payroll should be: Deposited in a safe deposit box. Held by the payroll custodian. Deposited in a special bank account. Held by the controller.

Deposited in a special bank account.

The primary reason for preparing a reconciliation between interest-bearing obligations outstanding during the year and interest expense presented in the financial statements is to: Evaluate internal control over securities. Determine the validity of prepaid interest expense. Ascertain the reasonableness of imputed interest. Detect unrecorded liabilities.

Detect unrecorded liabilities.

The occurrence assertion for accounts payable includes

Determining whether all accounts payable actually are liabilities

The cutoff assertion for accounts payable includes

Determining whether all accounts payable are recorded in the proper period

A Type II subsequent event usually requires A. An adjustment to the financial statements but no special disclosure is required B. Neither an adjustment to the financial statements nor disclosure in the footnotes C. Disclosure in the footnotes D. An adjustment to the financial statements and the footnotes

Disclosure in the footnotes

Which of the following procedures and methods are important in assessing a company's ability to continue as a going concern? Discussions with management regarding future plans related to sales activities, cost controls, and marketing efforts. Reviewing quarter on the internal control questionnaire specifically asking the client to evaluate the ability to continue.

Discussions with management regarding future plans related to sales activities, cost controls, and marketing efforts. - Yes Reviewing quarter on the internal control questionnaire specifically asking the client to evaluate the ability to continue. - No

A CPA reviews a client's payroll procedures. The CPA would consider internal control to be less than effective if a payroll department supervisor was assigned the responsibility for: Reviewing and approving time reports for subordinate employees. Distributing payroll checks to employees. Hiring subordinate employees. Initiating requests for salary adjustments for subordinate employees.

Distributing payroll checks to employees.

The use of one audit report date for normal subsequent events and a later date for one or more subsequent events.

Dual-dated audit report

The concept of materiality would be least important to an auditor when considering the: Total value of the account being audited. Adequacy of disclosure of a client's illegal act. Discovery of weaknesses in a client's internal control. Effects of a direct financial interest in the client upon the CPA's independence.

Effects of a direct financial interest in the client upon the CPA's independence

Enquiry

Either oral or written questions by the auditor. May be made internally to mgmt/employees in terms of obsolescence and collection of receivables or externally to lawyers concerning the probable outcome of litigation.

The most frequent position of the perpetrator in 2016 was

Employee

In almost all audits, overall assurance can e increased by also auditing the _____________ of A/R

Ending balance

The accounts payable department receives the purchase order form to accomplish all of the following except to:

Ensure that the goods had been received by the party requesting the goods.

Which of the following is the most important consideration of an auditor when examining the stockholders' equity section of a client's balance sheet? Changes in the capital stock account are verified by an independent stock transfer agent. Stock dividends and/or stock splits during the year under audit were approved by the stockholders. Stock dividends are capitalized at par or stated value on the dividend declaration date. Entries in the capital stock account can be traced to a resolution in the minutes of the board of directors' meetings.

Entries in the capital stock account can be traced to a resolution in the minutes of the board of directors' meetings.

Reconciliation

Establishing the agreement between two sets of related accounting records

Confirmation

Establishing the validity of a balance sheet by direct communication with an outside party

Vouching

Establishing the validity of a transaction by examining supporting documents

Which of the following is true?

Ethical use of computers means realizing that the availability of a system does not convey its unrestricted use

Inspection of Records or Documents (Audit Procedures)

Examination of internal or external records or documents that are in paper form, electronic form, or other media.

Which of the following procedures is least likely in the audit of capital stock? Examine all outstanding stock certificates for completeness. Account for the proceeds from stock issues. Reconcile shares outstanding with the general ledger. Evaluate compliance with stock option plans.

Examine all outstanding stock certificates for completeness.

Which of the following tests of details most likely would help an auditor determine whether accounts payable have been misstated? Examining reported purchase returns that appear too low. Examining vendor statements for amounts not reported as purchases. Search for customer-returned goods that were not reported as returns. Reviewing bank transfers recorded as cash received from customers.

Examining vendor statements for amounts not reported as purchases.

Which of the following phrases would an auditor most likely include in the auditor's report when expressing a qualified opinion because of inadequate disclosure?

Except for the omission of the information discussed in the preceding paragraph.

An auditor's decision concerning whether or not to "dual-date" the audit report is based upon the auditor's willingness to: Extend auditing procedures. Accept responsibility for year-end adjusting entries. Permit inclusion of a note captioned: event (unaudited) subsequent to the date of the auditor's report. Assume responsibility for resolving all events subsequent to the issuance of the auditor's report.

Extend auditing procedures.

There is substantial doubt about an entity's ability to continue as a going concern for a reasonable period time. In evaluating the entity's plans for dealing with the adverse effects of future conditions and events, the auditor most likely would consider what as a mitigating factor?

Extension of due dates of existing loans.

Confirmation

Form of enquiry that enables the auditor to obtain information directly from an independent source outside the entity. Normally the entity makes the request with the auditor controlling the mailing of enquiry.

Signs of a material weakness

Fraud, material or not, committed by senior management. Restaement of previously issued financial statements to reflect correction of misstatement. Identification by the auditor of an error of material misstatement not detected by management (last link in the change). Ineffective oversight by the audit committee

Discuss why the adoption of international accounting and auditing standards might be beneficial to investors and auditors.

Given the global nature of the financial markets, investors, both in the U.S. and abroad, frequently make investments in companies that are located all over the world. While many companies located outside the U.S. already prepare financial statements in accordance with International Financial Reporting Standards (IFRS), financial statements of U.S.-based entities are based on U.S. generally accepted accounting principles, and differences in the basis of presentation makes the analysis of U.S. and non-U.S.-based company financial statements difficult. Similarly, differences exist in auditing standards issued across the globe, so the adoption of International Statements on Auditing (ISAs) would mean auditors from around the globe are conducting their audits using the same set of standards. The embrace of IFRS and ISAs will help investors in their analysis of audited financial statements prepared across the globe.

positives of m.u.s.

Great with large populations with few expected errors (ex. inventory, a/r) Dont have to deal with standard deviation or stats like classical variable method automatically results in a sample selection that is stratified

Competence as a certified public accountant includes all of the following except: Having the technical qualifications to perform an engagement. Possessing the ability to supervise and to evaluate the quality of staff work. Guaranteeing the accuracy of the work performed. Consulting others if additional technical information is needed.

Guaranteeing the accuracy of the work performed.

A canceled check is not considered particularly reliable evidence since the check was prepared within the client's organization

Incorrect-the check was processed externally by two organizations 1) who it was written too 2) banking system

Program Flowcharts

IT professionals create these to help them plan the logic for each processing routine in large programs

Auditor considers the impact of the issue on the financial statements, the level of complexity and uncertainty surrounding the issue, whether there are any related issues that should be concerned, and the impact of the issue on planned audit procedures

Identify and define the issue

An auditor will ordinarily examine invoices from lawyers primarily in order to A. Identify possible unasserted litigation, claims, and assessments B. Estimate the dollar amount of contingent liabilities C. Assess the legal ramifications of litigation in progress D. Substantiate accruals

Identify possible unasserted litigation, claims, and assessments

Material loss contingencies should be recorded in the financial statements if available information indicates it is probable that a loss had been sustained prior to the balance sheet date and the amount of such loss can be reasonably estimated. For a public company these considerations will affect the audit report as follows: If a loss meets these criteria, the auditor may issue an unqualified opinion but is required to point out the contingency in an explanatory paragraph of the report. If a loss meets these criteria and is disclosed in the financial statement notes, the auditor may issue an unqualified opinion, but is required to point out the contingency in an explanatory paragraph of the report. If a loss meets these criteria and is disclosed in the financial statement notes, the auditor may issue an unqualified opinion, but should consider adding an explanatory paragraph as a means of emphasizing the disclosure. If a loss is probable but the amount cannot be reasonably estimated and is disclosed in the notes to the financial statements, the auditor may issue an unqualified opinion.

If a loss is probable but the amount cannot be reasonably estimated and is disclosed in the notes to the financial statements, the auditor may issue an unqualified opinion.

1. Make no reference in the audit report

If some other firm made an unqualified opinion, you review and agree, then you take all credit and all potential risk

The four phases of the SDLC end with this phase

Implementation, follow-up, and maintenance

For all audits of financial statements made in accordance with auditing standards, the use of analytical procedures is required to some extent.

In Planning stage - Yes As a substantive test - No In completion stage - Yes

CPA Firm A has performed most of the audit of Consolidated Company's financial statements and qualifies as the group auditor. CPA Firm B did the remainder of the work. Firm A wishes to assume full responsibility for Firm B's work. Which of the following statements is correct? Such assumption of responsibility violates the profession's standards. In such circumstances, when appropriate requirements have been met, Firm A should issue a standard unmodified opinion on the financial statements. In such circumstances, when appropriate requirements have been met, Firm A should issue an unmodified opinion on the financial statements but should make appropriate reference to Firm B in the audit report. CPA firm A should normally qualify its audit report on the basis of the scope limitation involved when another CPA firm is involved.

In such circumstances, when appropriate requirements have been met, Firm A should issue a standard unmodified opinion on the financial statements.

Property, Plant and Equipment

Includes acquisitions and disposals and related depreciation expense

Financing Cycle

Includes debt and equity financing, repayments to borrowers, interest expense, and dividends

Investments Cycle

Includes investments in debt and equity and the income received from investments

Payroll and Personnel

Includes payroll(salaried and hourly) and personnel functions

Inventory Cycle

Includes perpetual inventory, physical counts, and manufacturing costs

Expenditure Cycle

Includes purchases, payables, and cash disbursements

Revenue Cycle

Includes sales revenues, receivables, and cash receipts

Sampling Risk Type 2 definition

Incorrect acceptance, risk assessed too low. Sample contained no errors when in reality there were a lot of errors in the account. Type 2 in ineffective if you dont catch errors in an account that could lead to a material msistatement

An auditor has substantial doubt about the entity's ability to continue as a going concern. Management's plans for dealing with the adverse effects of these conditions most likely would include what?

Increase ownership equity.

A review of the financial statements and the entire set of audit files by an independent reviewer to whom the audit team must justify the evidence accumulated and the conclusions reached.

Independent review

Which of the following is a general purpose financial reporting framework? Generally accepted auditing standards. Auditing Standards of the Public Company Accounting Oversight Board. International Standards of Auditing. International Financial Reporting Standards.

International Financial Reporting Standards.

Recognition that people's motivations and perceptions can lead them to provide biased/misleading info

Interpersonal understanding

Which of the following procedures would an auditor most likely perform to obtain evidence about the occurrence of subsequent events? Determine whether inventory ordered before the year-end was included in the physical count. Inquire about payroll checks that were recorded before year-end but cashed after year-end. Investigate changes in capital stock recorded after year-end. Review tax returns prepared by management after year-end.

Investigate changes in capital stock recorded after year-end.

Tests of details of balances

Involve examining support for the closing balance directly.

Tests of details of transactions

Involve examining support for the individual debits and credits posted to an account.

Business Process Reenginnering

Involves redesigning business process from scratch

The Bill of Lading

Is a written contract of the receipt and shipment of goods between the seller and the carrier

The auditors include an emphasis-of-matter paragraph in a nonpublic company audit report with an unmodified opinion in order to emphasize that the entity being reported upon is a subsidiary of another business enterprise. The inclusion of this paragraph: Is appropriate and would not negate the unmodified opinion. Is considered a qualification of the opinion. Is a violation of generally accepted reporting standards if this information is disclosed in notes to the financial statements. Necessitates a revision of the opinion paragraph to include the phrase "with the foregoing explanation."

Is appropriate and would not negate the unmodified opinion.

A basis for modification paragraph in the audit of the financial statements of a nonpublic company: Is only included with qualified, adverse, or disclaimers of opinion. Is presented only in audit reports with unmodified opinions. Has a section title: Emphasis-of-Matter. Must be included in all nonpublic company audit reports.

Is only included with qualified, adverse, or disclaimers of opinion.

When financial statements are affected by a material departure from generally accepted accounting principles, the auditors should: Issue an unmodified opinion with a basis for modification paragraph. Withdraw from the engagement. Issue an "except for" qualification or an adverse opinion. Issue an "except for" qualification or a disclaimer of opinion.

Issue an "except for" qualification or an adverse opinion.

Disclaimer of Opinion

It is issued when the auditor is unable to be satisfied that the overall financial statements are fairly presented 1. Sever scope limitation **Modify intro, delete scope, add paragraph before opinion, modify opinion** 2. Independence issue **One paragraph only, stating lack of independence**

Why is the monthly reconciliation of bank accounts by an independent person an important internal control over cash balances?

It provides an opportunity for an internal verification of the cash receipts and cash disbursement transactions, investigation of reconciling items on the bank reconciliation, and the verification of the ending cash balance.

In what ways was it felt that the pronouncements issued by the Financial Accounting Standards Board would carry greater weight than the opinions issued by the Accounting Principles Board:

It was believed that FASB Pronouncements would carry greater weight than APB Opinions because of significant differences between the FASB and the APB, namely: (1) The FASB has a smaller mem¬bership, (2) full-time compensated members; (3) the FASB has greater autonomy, (4) increased independence; (5) the FASB has broader representation than the APB.

The auditor's primary means of obtaining corroboration of management's information concerning litigation is a A. Letter of corroboration from the auditor's lawyer upon review of the legal documentation B. Letter of audit inquiry to the entity's lawyer C. Confirmation of claims and assessments from an officer of the court presiding over the litigation D. Confirmation of claims and assessments from the other parties to the litigation

Letter of audit inquiry to the entity's lawyer

Confidence level

Level of assurance. Probability that your outcome is corect. higher the confidence level required= larger the sample size needed = lower the risk

The auditor will most likely perform extensive tests for possible understatement of: Multiple Choice Revenues. Assets. Liabilities.

Liabilities.

The auditor will most likely perform extensive tests for possible understatement of: Revenues. Assets. Liabilities. Capital.

Liabilities.

Which of the following is least likely to result in an adverse opinion? Change in accounting principle. Limitation in the scope of the audit. Uncertainty. Related party transaction.

Limitation in the scope of the audit.

An auditor issued an audit report that was dual dated for a subsequent event occurring after the completion of fieldwork but before issuance of the auditor's report. What is the auditor's responsibility for events occurring subsequent to the completion of fieldwork?

Limited to the specific event referenced.

With respect to issuance of an audit report which is dual-dated for a subsequent event occurring after the completion of fieldwork but before issuance of the auditors' report, the auditors' responsibility for events occurring subsequent to the date of the audit report is: Extended to include all events occurring until the date of the last subsequent event referred to. Limited to the specific event referred to. Limited to all events occurring through the date of issuance of the report. Extended to include all events occurring through the date of submission of the report to the client.

Limited to the specific event referred to.

All of these are examples typical costs that a company will usually incur when switching to a new computer systems except the costs of:

Lost Customers

Tests of Controls

Made to provide evidence about the effectiveness of the design and operations of internal control structure policies and procedures.

Who makes assertions? List the assertions

Management 1) Existence and occurrence 2) Rights and Obligations 3) Completeness 4) Cutoff 5) Valuation, allocation, and accuracy 6) Presentation and disclosure

Which of the following is not included in a standard unqualified opinion on internal control over financial reporting performed under PCAOB requirements?

Management and audit committee are responsible for maintaining effective internal control over financial reporting.

Hall accepted an engagement to audit the year 1 financial statements of XYZ Company. XYZ completed the preparation of the year 1 financial statements on February 13, year 2, and Hall began the audit work on February 17, year 2. Hall completed the audit work on March 24, year 2, and completed the report on March 28, year 2. The client's representation letter normally would be dated: February 13, year 2. February 17, year 2. March 24, year 2. March 28, year 2.

March 24, year 2.

Which of the following is a control procedure that is usually applied to accounts payable? Periodic confirmation of accounts payable. Mailing statements to vendors detailing their account. Periodic aging of accounts payable. Matching invoices with receiving documents before disbursements are authorized.

Matching invoices with receiving documents before disbursements are authorized.

Which of the following is the most effective control activity to detect vouchers prepared for the payment of goods that were not received?

Matching of purchase order, receiving report, and vendor invoice for each voucher in the accounts payable department

Explain how materiality differs for failure to follow GAAP and for lack of independence.

Materiality for lack of independence in audit reporting is easiest to define. If the auditor lacks independence as defined by the AICPA Code of Professional Conduct, it is always considered highly material and therefore a disclaimer of opinion is always necessary. That is, either the CPA is independent or not independent. For failure to follow GAAP, there are three levels of materiality: immaterial, material, and highly material

The scope of an audit is not restricted when an attorney's response to an auditor as a result of a client's letter of audit inquiry limits the response to what?

Matters to which the attorney has given substantive attention in the form of legal representation

An attorney responding to an auditor as a result of the client's letter of audit inquiry may appropriately limit the response to: Items which have high probability of being resolved to the client's detriment. Asserted claims and pending or threatened litigation. Legal matters subject to unsettled points of law, uncorroborated information, or other complex judgments. Matters to which the attorney has given substantive attention in the form of legal consultation or representation.

Matters to which the attorney has given substantive attention in the form of legal consultation or representation.

The best means of verification of cash, inventory, office equipment, and nearly all other assets is a physical count of the units, only a physical count gives the auditors complete assurance as to the accuracy of the amounts listed on the balance sheet

Misleading-Accounting office equipment does not establish the cost or ownership *Just as important as physical existence

2. Make reference in the report (modified wording report)

Modify intro, scope, opinion (no extra paragraph) You refer to other auditors... you agree, not taking credit

3. Qualify the opinion

Modify the scope and opinion, add paragraph before opinion. Rare. Divided up, share responsibility.. there are two qualified opinions.

What following event occurring after the issuance of an auditor's report most likely would cause the auditor to make further inquiries about the previously issued financial statements?

New information is discovered concerning undisclosed lease transactions of the audited period.

An auditor compares information on canceled checks with information contained in the cash disbursements journal. The objective of this test is to determine that

No discrepancies exist between the data on checks and the data in the journal

Can the auditors reduce inherent risk by performing audit procedures?

No- inherent risk will be there anyways

Which of the following is not a contingent liability with which an auditor is particularly concerned? Notes receivable discounted- Product warranties-

Notes receivable discounted-Yes Product warranties-Yes

Sample deviation

Number of deviations/ sample size

An auditor wishes to perform tests of controls on an entity's cash disbursements procedures. If the control activities leave no audit trail of documentary evidence, the auditor most likely will test the procedures by

Observation and inquiry

An auditor wishes to perform tests of controls on a client's cash disbursements relating to accounts payable. If the control procedures leave no audit trail of documentary evidence, the auditor most likely will test the procedures by: Confirmation and observation. Observation and inquiry. Analytical procedures and confirmation. Inquiry and analytical procedures.

Observation and inquiry.

Which of the following procedures would an auditor least likely perform before the balance sheet date?

Observation of merchandise Inventory

Which of the following auditing procedures is ordinarily performed last? Reading of the minutes of the directors' meetings. Confirming accounts payable. Obtaining a management representation letter. Testing of the purchasing function.

Obtaining a management representation letter.

The existence of audit risk is recognized by the statement in the auditor's standard report that the auditor

Obtains reasonable assurance about whether the financial statements are free of material misstatement.

In assessing control risk for purchases, an auditor voucher register to the supporting documents. Which assertion would this test of controls most likely support?

Occurance

Recorded transactions exists and deals with whether recorded transactions have actually occurred

Occurrence

Transactions and events that have been recorded have occurred and pertain to the entity

Occurrence

Disclosed events and transactions have occurred and pertain to the entity

Occurrence and rights and obligations

Assertions about the classes of transactions and events for the period under audit include

Occurrence, completeness, and cutoff

which of the following statements is true?

Original entries are recorded in journals, and journal totals are posted to ledgers

Auditors may choose not to confirm accounts payable because: Confirmation obtains evidence identical to that obtained by cutoff tests. Other reliable external evidence to support the balances is likely to be available. A reading of the corporate minutes reveals that confirmation is unnecessary. The balances due will have changed between the year-end and the date of confirmation.

Other reliable external evidence to support the balances is likely to be available.

Which of the following summarizes the threshold at which auditors are required to request management to record any identified factual misstatements that are: Material. Material or immaterial. Significant deficiencies. Other than trivial.

Other than trivial.

Tendency to overestimate one's own abilities to perform tasks or to make accurate assessments of risks or other judgments and decisions

Overconfidence

Inspection of Tangible Assets (Audit Procedures)

Physical Examination of the tangible assets.

Which of the following is a primary objective of the purchasing process?

Paying vendors at the optimal time

Considers a number of factors, such as whether they understand the form and substance of the transaction/event, whether the relevant authoritative literature has been applied consistently by the client to similar situations, whether the auditor has been able to corroborate the facts/assumptions that are important to the analysis, and whether the auditor has identified any discrepancies/inconsistencies in the facts/info obtained

Perform the analysis and identify potential alternatives

In connection with the audit of the financial statements of a manufacturing company, an auditor is observing the physical inventory of finished goods, which consists of expensive, highly complex electronic equipment. Discuss the validity and limitations of the audit evidence provided by the procedure

Physical evidence is one of the reliable sources of evidence in this case when the inventory consists of individual valued items you should >>>>>>>

Control Activities Defintion

Policies and procedures that help ensure that magnements directives are carried out to address risk.

It would be appropriate for the payroll accounting department to be responsible for which of the following functions? Approval of employee time records. Maintenance of records of employment, discharges, and pay increases. Preparation of periodic governmental reports as to employees' earnings and withholding taxes. Distribution of paychecks to employees.

Preparation of periodic governmental reports as to employees' earnings and withholding taxes.

Which of the following procedures relating to the examination of accounts payable could the auditor delegate entirely to the entity's employees?

Prepare a schedule of accounts payable

All of these are levels of a data flow diagram except:

Process Diagram

Assets are included at the amounts estimated to be realized; only applies to asset amounts; objective concerns whether an account balance has been reduced for declines from historical cost to net realizable value or when accounting standards require FMV accounting treatment

Realizable value

Auditors are responsible for _____________, but not ________________ assurance

Reasonable; absolute

When a client declines to disclose essential information in the financial statements or notes, the auditor of the financial statements should: Provide the information in the audit report, if practicable, and qualify the opinion because of a limitation on the scope of the audit. Provide the information in the audit report, if practicable, and qualify the opinion because of a departure from GAAP. Issue a disclaimer of opinion because the client has interfered with the auditor's function of assessing the adequacy of disclosure. Issue an unmodified opinion, but inform the reader by including the omitted information in the audit report.

Provide the information in the audit report, if practicable, and qualify the opinion because of a departure from GAAP.

An auditor wants to verify that for a given inventory acquisition the merchandise was ordered by the company. The auditor should look at which document

Purchase Order

Budd, the purchasing agent for Lake Hardware Wholesalers, has a relative who owns a retail hardware store. Budd arranged for hardware to be delivered by manufactures to the retail store on a C.O.D. basis, thereby enabling his relative to buy at Lake's wholesale prices. Budd was probably able to accomplish this because of Lake's poor internal control over

Purchase orders

Which of the following accounts is not affected by cash disbursement transactions

Purchase returns

Purchase cutoff procedures should be signed to test whether or not all inventory

Purchased and received before the year-end was recorded before year-end

If auditor is precluded by management or those charged with governance from obtaining sufficient appropriate evidence to evaluate whether noncompliance that may be material to the financial statements has occurred or is likely to have occurred, the auditor should express a _________________ on the financial statements on the basis of the _______________.

Qualified opinion/disclaim an opinion; scope limitation

Audit Evidence is gathered to use:

RAR 1) Reduce Audit Risk 2) Assess RMM (Risk Material Misstatement) 3) Restrict Detection Risk

A loss having material effect on financial statements due to an collectible trade account receivable due to a customer's bankruptcy, occurred suddenly due to a natural disaster ten days after the balance sheet date, but one month before the issuance of the financial statements and the auditor's report.

Requires financial statement disclosure but no adjustment to financial statements because it happened after the financial statement date.

What is sarbanes section 404

Requires management of a publicly traded company to issue a report that accepts responsiblity for establish and maintaining adequate internal controls over financial reporting (ICFR)

During its fiscal year, a company issued, at a discount, a substantial amount of first-mortgage bonds. When performing audit work in connection with the bond issue, the independent auditor should: Confirm the existence of the bondholders. Review the minutes for authorization. Trace the net cash received from the issuance to the bond revenue account. Inspect the records maintained by the bond trustee.

Review the minutes for authorization.

The purpose of analytical procedures at the completion of the audit includes all of the following except A. Revising the audit plan B. Recalculating some of the ratios examined during audit planning C. Reviewing adequacy of evidence gathered to investigate unusual fluctuations D. Considering overall reasonableness of the financial statements

Revising the audit plan

If a CPA violates the AICPA Code of Professional Conduct, the AICPA Trial Board may do all of the following, except: Multiple Choice Admonish the offending member. Suspend the offending member. Expel the offending member. Revoke the offending member's CPA certificate.

Revoke the offending member's CPA certificate.

Entity holds/controls the rights to assets, and liabilities are the obligations of the entity

Rights and obligation

In auditing the balance sheet, most revenue and expense accounts are also audited. Which accounts are most likely to be audited when auditing Accounts Receivable? Sales and Cost of Goods Sold. Interest and Bad Debt Expense. Sales and Bad Debt Expense. Interest and Cost of Goods Sold.

Sales and Bad Debt Expense.

Primary focus on most audits and collections on trade A/R in the cash receipts journal is the primary operating inflow to cash in the bank

Sales and collection cycle

The auditor is most likely to verify accrued commnissions payable in conjunction with the

Sales cutoff review

Which of these are the five feasibility areas that a design team should examine?

Schedule, economic, operational., technical, legal

A desire to investigate beyond the obvious, with a desire to corroborate

Search for knowledge

Substantive procedures to examine the cutoff assertion for accounts payable include

Selecting a sample of receiving reports around year end-end and comparing dates on related vouchers to dates in the purchases journal

Substantive procedures to examine the occurrence assertion for accounts payable include

Selecting a sample of vouchers and agreeing them to authorized purchase orders

Substantive procedures to examine the completeness assertion for accounts payable include

Selecting a sample of vouchers and tracing them to the purchases journal

Vouching

Selecting entries in the accounting records and inspecting the documentation that served as the basis for entries, to determine the propriety and validity of the recorded transactions.

Self-confidence to resist persuasion and to challenge assumptions/conclusions

Self-esteem

The purpose of segregating the duties of distributing payroll checks and hiring personnel is to: Separate the custody of assets from the accounting for those assets. Establish clear lines of authority and responsibility. Separate duties within the accounting function. Separate the authorization of transactions from the custody of related assets.

Separate the authorization of transactions from the custody of related assets.

A voucher

Serves as the basis for recording a vendor's invoice in the purchases journal

The auditor would be least likely to be concerned about internal control as it relates to: Land and buildings. Common stock. Shareholder meetings. Minutes of board of directors' meetings.

Shareholder meetings.

AICPA Clarity Project

Shifts focus to "opinion types" instead of "report types" (ex: unqualified to unmodified) Extra report paragraphs: ( ex: explanatory paragraphs change to emphasis of a matter or other matter Report structure and headings: much more detailed/clear about what auditors do/don't do 1. MGMT response 2. Auditors response 3. Opinion

Distinguish between FASB Accounting Standards Updates and FASB Statements of Financial Accounting Concepts:

Statements of financial accounting standards contained in Accounting Standards updates constitute generally accepted accounting principles and dictate acceptable financial accounting and reporting practices as promulgated by the FASB. The first standards statement was issued by the FASB in 1973. Statements of financial accounting concepts do not establish generally accepted accounting principles. Rather, the concepts statements set forth fundamental objectives and concepts that the FASB intends to use as a basis for developing future standards. The concepts serve as guidelines in solving existing and emerging accounting problems in a consistent, sound manner. Both the standards statements and the concepts statements may develop through the same process from discussion memorandum, to exposure draft, to a final approved statement.

Auditor must express an opinion on effectiveness of ICFR. T/F

T, except for companies under 75 million in market capital

Which type of subsequent event requires consideration by management and evaluation by the auditor? Subsequent events that have a direct effect on the financial statements & require adjustment- Subsequent events that have no direct effect on the financial statements but for which disclosure is considered-

Subsequent events that have a direct effect on the financial statements & require adjustment- Yes Subsequent events that have no direct effect on the financial statements but for which disclosure is considered-Yes

__________ ____________ are designed to detect material misstatements of relevant assertions.

Substantive procedures

Distinguish between the sufficiency (results) of audit evidence and the appropriateness (obtained) of audit evidence

Sufficiency is related to quantity and is a matter of judgement in every audit Appropriateness is related to the quality (reliability and relevance)

Auditor should obtain ________________________--- regarding material amounts and disclosures that are directly affected by laws and regulations

Sufficient appropriate evidence

1. Product costs should be matched directly with specific transactions and are recognized upon recognition of revenue. True False

TRUE

10. Accounts payable confirmations are used less frequently by auditors than accounts receivable confirmations. True False

TRUE

5. The principal business objectives of the purchasing process are acquiring goods and services and paying for those goods and services. True False

TRUE

6. The accounts payable department is responsible for ensuring that all vendor invoices, cash disbursements, and adjustments are recorded in the accounts payable records. True False

TRUE

7. After the controls are tested, the auditor sets the achieved level of control risk. True False

TRUE

8. Because of the low volume of purchase return transactions, the auditor normally does not test the controls associated with these transactions. True False

TRUE

9. Analytical procedures can be used to examine the reasonableness of accounts payable and accrued expenses. True False

TRUE

__________ ___ ____________ are designed to test the operating effectiveness of controls in preventing or detecting material misstatements.

Tests of controls

Confirm accounts receivable

Tests of details of account balances, transactions and disclosures

Why is there a greater emphasis on the detection of fraud in tests of details of cash balances than for other balance sheet accounts?

The amount of cash flowing into and out of the cash account is frequently larger than for any other account in the financial statements. Furthermore, the susceptibility of cash to misappropriation is greater than other types of assets.

Explain the relationships among the initial assessed control risk, test of controls and substantive test of transactions for cash disbursements, and the test of details of cash balances.

The appropriate tests for the ending balance in the cash accounts depend heavily on the initial assessment of control risk, test of controls, and test of transactions for disbursements. The company's controls over cash disbursements assist the auditor in determining that cash disbursed is for approved company purposes, that cash disbursements are promptly recorded in the proper amount, and that cash cutoff at year-end is proper. If the results of the evaluation of internal control, test of controls, and test of transactions are adequate, it is appropriate to reduce the test of details of balances for cash, especially for the detailed tests of bank reconciliations. On the other hand, if the tests indicate that the client's controls are inadequate, extensive year-end testing may be necessary.

. In which of the following situations would an auditor ordinarily issue an unqualified audit opinion without an explanatory paragraph?

The auditor decides to make reference to the report of another auditor as a basis, in part, for the auditor's opinion

With what types of contingencies might an auditor be concerned?

The auditor is generally concerned with contingencies arising from pending litigation for patent infringement, income tax disputes, product warranties, notes receivable discounted, guarantees of obligations of others, and unused balances of outstanding letters of credit.

Conducting and Inquiry... (Audit Procedures)

The auditor should: -Consider who the appropriate individual is to question -Ask clear, concise, and relevant questions. -Use open or closed questions appropriately -Listen actively and effectively -Consider the reactions and responses and ask follow-up questions -Evaluate the response

How does the auditor's opinion differ between scope limitations caused by client restrictions and limitations resulting from conditions beyond the client's control? Under which of these two will the auditor be most likely to issue a disclaimer of opinion? Explain.

The auditor's opinion may be qualified by scope limitations caused by client restrictions or by limitations resulting from conditions beyond the client's control. The former occurs when the client will not, for example, permit the auditor to confirm material receivables or physically observe inventories. The latter may occur when the engagement is not agreed upon until after the client's year-end when it may not be possible to physically observe inventories or confirm receivables. A disclaimer of opinion is issued if the scope limitation is so material that the auditor cannot determine if the overall financial statements are fairly presented. If the scope limitation is caused by the client's restriction, the auditor should be aware that the reason for the restriction might be to deceive the auditor. For this reason, a disclaimer is more likely for client restrictions than for conditions beyond anyone's control. When there is a scope restriction that results in the failure to verify material, but not pervasive accounts, a qualified opinion may be issued. This is more likely when the scope limitation is for conditions beyond the client's control than for restrictions by the client.

On February 17, 2017, a CPA completed all the evidence gathering procedures on the audit of the financial statements for the Buckheizer Technology Corporation for the year ended December 31, 2016. The audit is satisfactory in all respects except for the existence of a change in accounting principles from FIFO to LIFO inventory valuation, which results in an explanatory paragraph on consistency. On February 26, the auditor completed the tax return and the draft of the audit report. The final audit report was completed, attached to the financial statements, and delivered to the client on March 7. What is the appropriate date on the auditor's report?

The auditor's report should be dated February 17, 2017, the date on which the auditor concluded that he or she had sufficient appropriate evidence to support the auditor's opinion.

What is the primary objective of financial reporting:

The basic objective is to provide financial information about the reporting entity that is useful to present and potential equity investors, lenders, and other creditors in making decisions about providing resources to the entity.

Bond transactions are normally confirmed with: Individual holders of retired bonds. Recomputation procedures performed using interest expense. The bond trustee. Comparisons of retired bonds with those outstanding.

The bond trustee.

Completeness Assertion

The completeness assertion relates to whether all transactions and events that occurred during the period have been recorded. ***Opposite of occurrence assertion... failure to complete is to leave an account understated... Someone has left something out on purpose.

If the auditor determines that a subsequent event that affects the current period financial statements occurred after fieldwork was completed but before the audit report was issued, what date(s) may the auditor use on the report? The date of the original last day of fieldwork only. The date of the subsequent event only. The date on which the last day of fieldwork occurred along with the date of the subsequent event.

The date of the original last day of fieldwork only.-Yes The date of the subsequent event only.-No The date on which the last day of fieldwork occurred along with the date of the subsequent event.-Yes

The date of the management representation letter should coincide with what other date?

The date of the auditor's report.

What is the "expectations gap": What is the profession doing to try to close this gap:

The expectations gap is the difference between what people think accountants should be doing and what accountants think they can do. It is a difficult gap to close. The accounting profession recognizes it must play an important role in narrowing this gap. To meet the needs of society, the profession is continuing its efforts in developing accounting standards, such as numerous pronouncements issued by the FASB, to serve as guidelines for recording and processing business transactions in the changing economic environment.

In which of the following circumstances would an auditor be most likely to express an adverse opinion on the client's financial statements?

The financial statements are not in conformity with the FASB statements regarding the capitalization of leases

The Sarbanes Oxley Act was enacted to combat fraud and curb poor reporting practices. What are some key provisions of this legislation:

The following are some of the key provisions of the Sarbanes-Oxley Act: • Establishes an oversight board for accounting practices. The Public Company Accounting Over-sight Board (PCAOB) has oversight and enforcement authority and establishes auditing, quality control, and independence standards and rules. • Implements stronger independence rules for auditors. Audit partners, for example, are required to rotate every five years and auditors are prohibited from offering certain types of consulting services to corporate clients. • Requires CEOs and CFOs to personally certify that financial statements and disclosures are accurate and complete and requires CEOs and CFOs to forfeit bonuses and profits when there is an accounting restatement. • Requires audit committees to be comprised of independent members and members with finan-cial expertise. • Requires codes of ethics for senior financial officers. ***In addition, Section 404 of the Sarbanes-Oxley Act requires public companies to attest to the effectiveness of their internal controls over financial reporting

Describe the information included in the introductory, scope, and opinion paragraphs in a separate audit report on the effectiveness of internal control over financial reporting. What is the nature of the additional paragraphs in the audit report?

The introductory, scope, and opinion paragraphs are modified to include reference to management's report on internal control over financial reporting, and the scope of the auditor's work and opinion on internal control over financial reporting. The introductory and opinion paragraphs also refer to the framework used to evaluate internal control. Two additional paragraphs are added between the scope and opinion paragraphs that define internal control and describe the inherent limitations of internal control.

Which of the following is least likely to be included in a public company audit report with an unqualified opinion? A section on critical matters. The name of the engagement partner. A title with the word "Independent." A basis for opinion paragraph.

The name of the engagement partner.

What is the objective of financial reporting:

The objective of general purpose financial reporting is to provide financial information about the reporting entity that is useful to present and potential equity investors, lenders, and other creditors in decisions about providing resources to the entity through equity investments and loans or other forms of credit. Information that is decision-useful to capital providers (investors) may also be useful to other users of financial reporting who are not investors.

Which of the following is an internal control that would prevent a paid disbursement voucher from being presented for payment a second time?

The official signing the check should compare the check with the voucher and should "cancel" the voucher documents by marking them "paid"

Which individuals will generally not be considered independent for the purposes of preparing monthly bank reconciliations?

The people in charge of receipt and deposit of cash, issuance of checks, other handling of cash, and record keeping.

What are the purposes of the scope paragraph under the auditor's responsibility in the auditor's report? Identify the most important information included in the scope paragraph.

The purpose of the scope paragraph under the auditor's responsibility is to inform the financial statement users of the nature of the audit procedures performed. The information in the scope paragraph includes: 1. Procedures used to obtain audit evidence about the amounts and disclosures in the financial statements. 2. The audit procedures selected depend on the auditor's judgment, and consider the auditor's assessment of the risks of material misstatement, whether due to fraud or error. 3. As part of this risk assessment, the auditor considers internal control over financial reporting in the design of the audit procedures. 4. An audit includes evaluating the appropriateness of the accounting policies used, the reasonableness of significant estimates, and the overall presentation of the financial statements.

Business Risk

The risk assumed by investors or creditors that is associated with the company's survival and profitability

Inherent Risk

The risk of material misstatement of a financial statement assertion, assuming there were no related control

Which of the following would most likely be an appropriate addressee for an audit report? The shareholders of the corporation whose financial statements were examined. A third party who requested that a copy of the audit report be sent to her. The president of the corporation whose financial statements were examined. The chief financial officer.

The shareholders of the corporation whose financial statements were examined.

Engagement Risk

The risk of the loss of the auditors' reputation that happens when client goes bankrupt or management lacks integrity

Control Risk

The risk that a material misstatement that could occur in an account will not be prevented or detected on a timely basis by internal control

Audit Risk

The risk that the auditors may unknowingly fail to appropriately modify their opinion on financial statements that are materially misstated.

Sampling Risk

The risk that the auditors' conclusion based on a sample might be different from the conclusion they would reach if the test were applied to the entire population. For tests of controls, sampling risk include risks of assessing control risk too high and too low; for substantive testing, sampling risk include the risks of incorrect acceptance and rejection.

Detection Risk

The risk that the auditors' procedures will lead them to conclude that a financial statement assertion is not materially misstated when in fact such misstatement does exist

To forecast cash receipts, information must be collected

The sales process

Compare the wording in the standard unmodified opinion audit report for a nonpublic entity under AICPA auditing standards in Figure 3-1 with the wording for a public company audit under PCAOB auditing standards in Figure 3-3 . How are the reports similar? How are they different?

The standard unmodified opinion audit report for a non-public entity under AICPA auditing standards and the standard unqualified report for a public company under PCAOB auditing standards are very similar in substance. The introductory paragraphs are similar, although the public company report includes the responsibilities of management and the auditor. In contrast, the report for the non-public entity in Figure 3-1 has separate paragraphs for management's and the auditor's responsibility. These paragraphs provide additional information on the nature of these responsibilities. The scope paragraphs in each report are similar. However, there are differences in the description of the nature of the auditor's testing. The report for the non-public company indicates that the procedures are based on the auditor's judgment and consider the risks of material misstatement. The report for the non- public company also indicates that the auditor considers internal control in designing the audit procedures, and not for the purpose of expressing an opinion on internal control.

Compare the wording in the standard unmodified opinion audit report for a​ non-public entity under AICPA auditing standards with the wording for a public company audit under PCAOB auditing standards. How are the reports​ similar? How are they​ different?

The standard unmodified report for a non-public entity under AICPA auditing standards and the standard unqualified report for a public company under PCAOB auditing standards are very similar in substance The introductory paragraphs are similar, although the public company report includes the responsibilities of management and the auditor. In contrast, the report for the non public entity has separate paragraphs for management's and auditor's responsibility. These paragraphs provide additional information on the nature of these responsibilities. The scope paragraphs in each report are similar, however, there are differences in the description of the nature of the auditor's testing. The report for the non-public company indicates that the procedures are based on the auditor's judgment and consider the risks of material misstatement. The report for the non-public company also indicates that the auditor considers internal control in designing the audit procedures and not for the purpose of expressing an opinion on internal control. The content of the opinion paragraph is virtually the same for a nonpublic entity and a public entity.

Discuss two of the three matters which Sarbanes-Oxley requires auditors of public companies to report to the audit committee.

The three items that must be reported to the audit committee are: • all fraud and illegal acts, • significant internal control deficiencies and material weaknesses in the design or operation of internal control to those charged with governance, and • additional information such as all alternative treatments of financial information within requirements of accounting standards that have been discussed with management, ramifications of the alternative disclosures and treatments, and the treatment preferred by the auditor; initial selection of and changes in significant accounting policies or their application during the current audit period, as well as reasons for any changes; and methods used to account for any significant unusual transactions and the effect of significant accounting policies in controversial emerging areas.

Much of what has been termed cybercrime has merely involved the computer but probably would be more accurately classified as other types of crimes. A notable exception to this involves:

Theft of computer time

Business-Without-Boundaries

This business model is a result of networked enterprises and globalization

An environmental clean-up lawsuit is pending against your client. What information about the lawsuit would you as the auditor need in order to determine the "correct" accounting?

Three conditions are required for a contingent liability to exist: (1) there is a potential future payment to an outside party or the impairment of an asset that resulted from an existing condition; (2) there is uncertainty about the amount for the future payment or impairment; and (3) the outcome will be resolved by some future event or events. Accounting standards describe three levels of likelihood of occurrence and the appropriate financial statement treatment for each likelihood as follows: a. Probable-future event likely to occur and amount can be reasonably estimated then the financial statement accounts are adjusted. If amount cannot be reasonably estimated, then a footnote disclosure is necessary. b. Reasonably possible-chance of occurring is more than remote, but less than probable. Footnote disclosure is necessary. c. Remote-chance of occurrence is slight No disclosure is necessary.

Effective internal control over the payroll function would include which of the following? Total time recorded on time clock cards should be reconciled to job reports by employees responsible for those specific jobs. Payroll department employees should be supervised by the management of the personnel department. Payroll department employees should be responsible for maintaining employee personnel records. Total time spent on jobs should be compared with total time indicated on time clock punch cards.

Total time spent on jobs should be compared with total time indicated on time clock punch cards.

Objectives that must be met before the auditor can conclude that the transactions are properly recorded

Transaction-related audit objectives

Operating control over the check signature plate normally should be the responsibility of the

Treasurer

A basic assumption that underlies financial reporting is that an entity will continue as a going concern. True False

True

A going concern issue requires a modification of the three-paragraph standard unqualified audit report (public company). true or false?

True

An important primary purpose of the auditor's review of the entity's procurement system should be to determine the effectiveness of the activities to protect against

Unauthorized persons issuing purchase orders

When the auditors discover an understatement of liabilities, they would most likely also expect to find an: Understatement of assets. Understatement of owners' equity. Overstatement of expenses. Understatement of revenues.

Understatement of assets.

Unrecorded liabilities are most likely to be found during the review of which of the following documents?

Unpaid bills

Describe what is meant by reports involving the use of other auditors. What are the three options available to the primary auditor responsible for the opinion, and when should each be used?

When another CPA has performed part of the audit, the primary auditor issues one of the following types of reports based on the circumstances. 1. No reference is made to the other auditor. This will occur if the other auditor audited an immaterial portion of the financial statements, the other auditor is known or closely supervised, or if the principal auditor has thoroughly reviewed the other auditor's work. 2. Issue a shared opinion in which reference is made to the other auditor. This type of report is issued when it is impractical to review the work of the other auditor or when a portion of the financial statements audited by the other CPA is material in relation to the total. 3. The report may be qualified if the principal auditor is not willing to assume any responsibility for the work of the other auditor. A disclaimer may be issued if the segment audited by the other CPA is highly material.

PASS KEY - Relevent Assertions

When auditing revenue transactions, aditor generally focuses on testing the existence/occurence assertion(rather than completeness) because revenues are more likely to be overstated(existence) than understated(completeness). For expenses focus is on testing completeness because expenses are more likely to be understated rather than overstated.

Business Process Outsourcing

When companies contract with other companies to do certain task, such as human resources, finance and accounting, customer services, training, and IT

Off-shoring

When companies have production or work completed in countries like India, China, Canada, Mexico, or Malaysia

When an auditor discovers more than one condition that requires departure from or modification of the standard unmodified opinion audit report, what should the auditor's report include?

When the auditor discovers more than one condition that requires a departure from or a modification of a standard opinion audit report, the report should be modified for each condition. An exception is when one condition neutralizes the other condition. An example would be when the auditor is not independent and there is also a scope limitation. In this situation the lack of independence overshadows the scope limitation. Accordingly, the scope limitation should not be mentioned.

Which of the following statements is correct? Client prepared records (e.g., the general ledger) may be retained by the CPA until fees due to the CPA are received. CPA working papers are the joint property of the CPA and the client. Working papers prepared by the auditor solely for the engagement need not be returned to the client. CPA working papers that include copies of client's records are not available to third parties under any circumstances.

Working papers prepared by the auditor solely for the engagement need not be returned to the client.

Substantial Doubt about Going Concern

You are not giving an option on profitability- so if client is in adherence with GAAP then it is unqualified-- client needs to have disclosed this **add paragraph after opinion**

11. If a conflict of interest arises during the course of providing professional services, a professional accountant: a. Must consider withdrawing from the professional engagement b. Is required to withdraw from the professional engagement, but does not have to refund fees previously paid by its client c. Is required to withdraw from the professional engagement and is required to refund fees previously paid by the client d. Is not required to withdraw from the professional engagement if the conflict of interest did not exist at the inception of the relationship and it was not reasonably foreseeable at that time

a

13. An accountant recent met with the CFO of a major corporation. The accountant and this prospective client are contemplating entering into a professional relationship. The prospective corporate client has a history of violating the Foreign Corrupt Practices Act's provisions concerning bribery of foreign government officials. The accountant's job is to ensure that the company fully complies with this law. To accomplish this, the accountant will perform unannounced reviews of the company's accounting records and internal controls from time to time. The accountant's daughter told the CFO that her father is "an amazing accountant," which facilitated the professional introduction. The accountant's daughter recently was promoted to serve as this corporation's Director of International Sales-Asia Division. In all likelihood, the accountant: a. Should not agree to provide these services to this prospective client b. May provide services to this client, as long as the accountant's daughter does not also participate directly in the corporation's presentation of corporate financial transactions in the form of financial statements c. May provide all requested services because a conflict of interest does not currently exist and is not reasonably foreseeable d. May provide services to this client because safeguards against conflicts of interest readily are available

a

14-13 Which of the following is an effective internal control over cash payments? a. Signed checks should be mailed under the supervision of the check signer. b. Spoiled checks that have been voided should be disposed of immediately. c. Checks should be prepared only by persons responsible for cash receipts and cash disbursements. d. A check-signing machine with two signatures should be utilized.

a

14-21 Which of the following internal control procedures most likely would justify a reduced assessed level of control risk concerning the existence or occurrence of plant and equipment acquisitions? a. Periodic physical inspection and verification of plant and equipment by the an internal independent staff. b. Comparison of current-year plant and equipment account balances with prior-year actual balances. c. The review of prenumbered purchase orders to detect unrecorded trade-ins. d. Approval of periodic depreciation entries by a supervisor independent of the accounting department.

a

14-26 When using confirmations to provide evidence about the completeness assertion for accounts payable, the appropriate population most likely would be a. vendors with whom the client has previously done business. b. amounts recorded in the accounts payable subsidiary ledger. c. payees of checks drawn in the month after year-end. d. invoice filed in the client's open invoice file.

a

14-28 Which of the following combinations of procedures would an auditor be most likely to perform to obtain evidence about plant-asset addition? a. Inspecting documents and physically examining assets. b. Re-computing calculations and obtaining written management representations. c. Observing operating activities and comparing balances to prior-period balances. d. Confirming rights and obligations and corroborating transactions through inquiries of client personnel.

a

14-38 Determining that proper amounts of depreciation for fixed assets are expensed provides assurance about management's assertions of valuation or allocation and a. presentation and disclosure. b. completeness. c. rights and obligations. d. existence or occurrence.

a

14-42 In performing a search for unrecorded retirements of fixed assets, an auditor most likely would a. inspect certain items of equipment in the plant and trace those items to the accounting records. b. review the subsidiary ledger to ascertain whether depreciation was taken on each item of equipment during the year. c. trace additions to the "other assets" account to search for equipment that is still on hand but no longer being used. d. vouch select certain items of equipment from the accounting records and locate them in the plant.

a

14-5 For effective internal control purposes, which of the following individuals should be responsible for mailing signed checks? a. Receptionist. b. Treasurer. c. Accounts payable clerk. d. Payroll clerk.

a

14-8 A specific audit objective is to ensure that an approved receiving report is required to accompany every check request for payment of merchandise. Which of the following TOC procedures provides the greatest assurance that this control is operating effectively? a. Selection and examination of canceled checks and ascertainment that the related receiving reports are dated no later than the checks. b. Selection and examination of canceled checks and ascertainment that the related receiving reports are dated no earlier than the checks. c. Selection and examination of receiving reports and ascertainment that the related canceled checks are dated no earlier than the receiving reports. d. Selection and examination of receiving reports and ascertainment that the related canceled checks are dated no later than the receiving reports.

a

14. A CPA agreed to render professional services to all three of the shareholders in a corporation. All three shareholders asked the CPA to determine the amount of corporate earnings that should be distributed as dividends and the amount that should be maintained as Retained Earnings to spur the future growth of the corporation. All three told the CPA that they want to "allocate cash flow to maximize everyone's after-tax wealth." Two weeks after the CPA began this assignment, one of the shareholders told the CPA privately that she "desperately needed immediate cash flow from dividends because her mother recently had a stroke and needs to be able to afford to give her mother proper medical care. This shareholder asked that her mother's medical condition be kept confidential, to respect her mother's wishes. The CPA: a. Acted properly at the outset of this professional engagement b. Acted improperly at the outset of this professional engagement because representation of fellow shareholders automatically creates a dual-client conflict of interest c. Acted improperly at the outset of this professional engagement, but now has a duty to complete the engagement to minimize further harm to her clients d. Acted improperly at the outset of this professional engagement and has a duty to inform the accountancy licensing board about this misconduct

a

24. The determination of whether an American CPA has, or does not have, a conflict of interest is resolved by: a. A CPA on his or her own b. The Conflicts Advisory Board of the AICPA c. A board of the AICPA other than the Conflicts Advisory Board d. A mediation process between the CPA and its client

a

3. A CPA's conflicts of interest are determined by: a. The CPA herself b. The accountancy licensing board of the state in which she works as an accountant c. The AICPA Conflicts Enforcement Committee d. The AICPA committee that evaluates issues of independence, objectivity, and conflicts of interest

a

7. An auditing firm has a standard fee policy that it maintains internally and uses in bidding on auditing projects. However, in preparing bids, it increases its audit fee by up to 20% for corporate audit clients that have December 31 fiscal year-ends and lowers its audit fees by up to 30% for government entities that have fiscal years that end during the summer months. The auditing firm justifies this pricing practice by noting that its personnel are overwhelmed with work during the end-of-year "busy season," but often are idle during the summer months. This auditor: a. Does not have a conflict of interest b. Has a dual-client conflict of interest c. Has an accountant-client conflict of interest, even if this pricing differential is cost-justified d. Is not acting with integrity, unless it discloses this pricing policy to all affected clients

a

9. The IFAC code of conduct recognizes the concept of "implied consent." Implied consent, in the context of conflicts of interest, means that: a. A professional accountant may assume that a client has consented to a conflict of interest based on the surrounding circumstances b. A professional accountant may insist, before commencing work on a project, that a client consent to a potential conflict of interest c. Actual consent to a present-day conflict of interest also will be considered to constitute consent to any future conflicts of interest that may arise d. The likelihood of a conflict arising in the near future can be reasonably inferred from surrounding circumstances

a

A Type I subsequent event usually requires: A) an adjustment to the financial statements. B) no adjustment to the financial statements. C) withdrawal from the engagement. D) no action.

a

A major customer of an entity suffers a fire after year-end, but just prior to completion of audit fieldwork. The entity believes that this event could have a significant direct effect on the financial statements. The auditor should: A) advise management to disclose the event in the notes to the financial statements. B) disclose the event in the auditor's report. C) withhold submission of the auditor's report until the extent of the direct effect on the financial statements is known. D) advise management to adjust the financial statements.

a

A written representation from an entity's management that, among other matters, acknowledges responsibility for the fair presentation of financial statements should normally be signed by the: A) chief executive officer and the chief financial officer. B) chief financial officer and the chairman of the board of directors. C) chairman of the audit committee of the board of directors. D) chief executive officer, the chairman of the board of directors and the entity's lawyer.

a

After fieldwork audit procedures are completed for a public company and private company whose financial statements are expected to be widely distributed, a partner of the CPA firm who has not been involved in the audit performs an engagement quality review of the working papers. This review focuses on: A) an evaluation of the significant judgments made by the engagement teams and the conclusions reached in formulating the audit report. B) irregularities involving the entity's management and its employees. C) the materiality of the adjusting entries proposed by the audit staff. D) the communication of internal control weaknesses to those charged with governance.

a

After the financials have been issued with an accompanying audit report on a nonpublic entity, there is no obligation to make any further audit tests or inquiries with respect to the audited financial statements covered by that report unless: A) new information comes to the auditor's attention concerning an event that occurred prior to the date of the auditor's report that may have affected the auditor's report. B) material adverse events occur after the date of the auditor's report. C) final determination or resolution was made on matters that had resulted in a qualification in the auditor's report. D) final determination or resolution was made of a contingency that had been disclosed in the financial statements and no liability arose from the resolution.

a

An auditor is concerned with completing various phases of the examination after the balance sheet date. This "subsequent period" involving formal audit procedures extends to the date of the: A) issuance of the financial statements. B) final review of the audit working papers. C) auditor's report. D) delivery of the auditor's report to the entity.

a

An auditor issued an audit report that was dual dated for a subsequent event that occurred after the completion of fieldwork but before issuance of the auditor's report. The auditor's responsibility for events occurring subsequent to the completion of fieldwork was: A) limited to the specific event referenced. B) limited to include only events occurring before the date of the last subsequent event referenced. C) extended to subsequent events occurring through the date of issuance of the report. D) extended to include all events occurring since the completion of fieldwork.

a

An auditor's decision concerning whether or not to "dual date" the audit report is based upon the auditor's willingness to: A) extend auditing procedures. B) accept responsibility for all events between year-end and the audit report date. C) permit inclusion of a footnote captioned: event (unaudited) subsequent to the date of the auditor's report. D) assume responsibility for events subsequent to the issuance of the auditor's report.

a

Communications between the auditor and those charged with governance should include all of the following except: A) a summary of specific audit procedures used. B) a summary of uncorrected misstatements. C) consultations with other accountants. D) major issues discussed with management before the auditor was retained.

a

Harvey, CPA, is preparing an audit program for the purpose of ascertaining the occurrence of subsequent events that may require adjustment or disclosure essential to a fair presentation of the financial statements in conformity with generally accepted accounting principles. Which one of the following procedures would be least appropriate for this purpose? A) Confirm, as of the completion of fieldwork, accounts receivable that have increased significantly from the year-end date. B) Read the minutes of the board of directors. C) Inquire of management concerning events that may have occurred. D) Ask legal counsel about any developments relating to litigation, claims, or assessments against the company.

a

The auditor's primary means of obtaining corroboration of management's information concerning litigation is a: A) letter of audit inquiry to the entity's lawyer. B) letter of corroboration from the auditor's lawyer upon review of the legal documentation. C) confirmation of claims and assessments from the other parties to the litigation. D) confirmation of claims and assessments from an officer of the court presiding over the litigation.

a

The purpose of analytical procedures at the completion of the audit includes all of the following except: A) revising the audit plan. B) assessing the conclusions reached on the financials statement components. C) evaluating the overall financial statement presentation. D) recalculating some of the ratios examined during audit planning to evaluate how these ratios may have been affected by audit adjustments.

a

When obtaining evidence regarding litigation against an entity, the CPA would be least interested in determining: A) an estimate of when the matter will be resolved. B) the period in which the underlying cause of the litigation occurred. C) the probability of an unfavorable outcome. D) an estimate of the potential loss.

a

Which of the following conditions or events most likely would cause an auditor to have substantial doubt about an entity's ability to continue as a going concern? A) Cash flows from operating activities are negative. B) Research and development projects are postponed. C) Significant related party transactions are pervasive. D) Stock dividends replace annual cash dividends.

a

Which of the following is generally requested in a legal letter? A) A request that the attorney comment on unasserted claims where his or her views differ from management's evaluation. B) A list of all attorneys that performed any work for the entity during the year. C) A statement indicating that the attorney is responsible for the fair presentation of unasserted claims in the entity's financial statements. D) A request that the attorney provide a copy of all invoices given to the entity during the year.

a

Which of the following items should an auditor communicate to those charged with governance in a publicly traded company? A) Significant unusual transactions, critical estimates, and management's consultation with other accountants about significant accounting matters. B) Significant unusual transactions and critical estimates, but not management's consultation with other accountants about significant accounting matters. C) Management's consultation with other accountants about significant accounting matters but not significant unusual transactions and critical estimates. D) Neither significant unusual transactions, critical estimates, nor management's consultation with other accountants about significant accounting matters.

a

Which of the following procedures should an auditor generally perform regarding subsequent events? A) Compare the latest available interim financial statements issued after year-end with the financial statements being audited. B) Send second requests to the entity's customers who failed to respond to initial accounts receivable confirmation requests. C) Communicate material weaknesses in internal controls to those charged with governance. D) Review the cutoff bank statements for several months after year-end.

a

Which of the following statements ordinarily is included among the written management representations obtained by the auditor? A) Compensating balances and other arrangements involving restrictions on cash balances have been disclosed. B) Management acknowledges responsibility for illegal actions committed by employees. C) Sufficient evidential matter has been made available to permit the issuance of an unqualified opinion. D) Management acknowledges that there are no material misstatements in the internal controls.

a

Logical Data Flow Diagram

a DFD that shows what participants do in a given system

In conducting a feasibility study, technical feasibility refers to whether

a proposed system is attainable given the existing technology

Decision Tree

a schematic tree-shaped diagram used to determine a course of action

A professional code of conduct is

a set of rules outlining the rules and responsibilities of, or proper practices for, an individual within a profession

What is a payroll journal

a special journal where payroll hours, dollars and deductions are accumulated for subsequent posting to the general ledger account

Which of the following conditions or events most likely would cause an auditor to have substantial doubt about an entity's ability to continue as a going concern? a) Cash flows from operating activities are negative. b) Research and development projects are postponed. c) Significant related party transactions are pervasive. d) Stock dividends replace annual cash dividends.

a) Cash flows from operating activities are negative.

An auditor discovers a likely fraud during an audit but concludes that the overall effect of the fraud is not sufficiently material to affect the audit opinion. The auditor should probably a) Disclose the fraud to the appropriate level of the client's management. b) Disclose the fraud to appropriate authorities external to the client. c) Discuss with the client the additional audit procedures that will be needed to identify the exact amount of the fraud. d) Modify the audit program to include tests specifically designed to identify the fraud and its impact on the financial statements.

a) Disclose the fraud to the appropriate level of the client's management.

The acceptable level of detection risk is inversely related to the a) Extent of the substantive procedures. b) Risk of misapplying auditing procedures. c) Overall materiality. d) Risk of failing to discover material misstatements.

a) Extent of the substantive procedures.

In which of the following circumstances would an auditor usually choose between issuing a qualified opinion or a disclaimer of opinion on a client's financial statements? a) Inability of the auditor to obtain sufficient competent evidence. b) Inadequate disclosure of accounting policies. c) Unreasonable justification for a change in accounting principle. d) Departure from generally accepted accounting principles.

a) Inability of the auditor to obtain sufficient competent evidence.

Risk of material misstatement refers to a combination of which two components of the audit risk model? a) Inherent risk and control risk. b) Control risk and detection risk. c) Audit risk and inherent risk. d) Audit risk and control risk.

a) Inherent risk and control risk.

Which of the following statements best describes management's and the external auditor's respective levels of responsibility for a public company's financial statements? a) Management has the primary responsibility to ensure that the company's financial statements are prepared in accordance with GAAP, and the auditor provides reasonable assurance that the statements are free of material misstatement. b) Management has the primary responsibility to ensure that the company's financial statements are prepared in accordance with GAAP, and the auditor provides a guarantee that the statements are free of material misstatement. c) Management and the external auditor share equal responsibility for the fairness of the entity's financial statements in accordance with GAAP. d) Neither management nor the external auditor has significant responsibility for the fairness of the entity's financial statements in accordance with GAAP.

a) Management has the primary responsibility to ensure that the company's financial statements are prepared in accordance with GAAP, and the auditor provides reasonable assurance that the statements are free of material misstatement.

Evidence is reliable if it a) Signals the true state of a management assertion. b) Applies to the period being audited. c) Relates to the audit assertion being tested. d) Is consistent with management's assertions.

a) Signals the true state of a management assertion.

Which of the following is the most important reason for an auditor to gain an understanding of an audit client's system of internal control over financial reporting? a) Understanding a client's system of internal control can help the auditor assess risk and identify areas where financial statement misstatements might be more likely. b) Understanding a client's system of internal control is not a required part of the audit process. c) Understanding a client's system of internal control can help the auditor make valuable recommendations to management at the end of the engagement. d) Understanding a client's system of internal control can help the auditor sell consulting services to the client.

a) Understanding a client's system of internal control can help the auditor assess risk and identify areas where financial statement misstatements might be more likely.

Tech Company has disclosed an uncertainty due to pending litigation. The auditor's decision to issue a qualified opinion on Tech's financial statements would most likely result from: a) a lack of sufficient evidence. b) a lack of insurance coverage for possible losses from such litigation. c) an inability to estimate the amount of loss. d) the entity's lack of experience with such litigation.

a) a lack of sufficient evidence.

Tech Company has disclosed an uncertainty due to pending litigation. The auditor's decision to issue a qualified opinion on Tech's financial statements would most likely result from: a) a lack of sufficient evidence. b) a lack of insurance coverage for possible losses from such litigation. c) the entity's lack of experience with such litigation. d) an inability to estimate the amount of loss.

a) a lack of sufficient evidence.

When an auditor is asked to express an opinion on an entity's rent and royalty revenues, he or she may: a) accept the engagement, provided the auditor's opinion is expressed in a special report. b) accept the engagement, provided distribution of the auditor's report is limited to the entity's management. c) not accept the engagement unless also engaged to audit the full financial statements of the entity. d) not accept the engagement because to do so would be tantamount to agreeing to issue a piecemeal opinion.

a) accept the engagement, provided the auditor's opinion is expressed in a special report.

Which of the following best describes relationships among auditing, attest, and assurance services? a) Auditing is a type of assurance service. b) Auditing and attest services represent two distinctly different types of services—there is no overlap. c) Assurance is a type of attest service. d) Attest is a type of auditing service.

a) auditing is a type of assurance service

When the audited financial statements of the prior year are presented together with those of the current year, the continuing auditor's report should cover a) both years. b) only the current year. c) only the current year, but the prior year's report should be presented. d) only the current year, but the prior year's report should be referred to.

a) both years.

If the independent auditors decide that it is efficient to consider how the work performed by the internal auditors may affect the nature, timing, and extent of audit procedures, they should assess the internal auditors' a) competence and objectivity. b) efficiency and experience. c) independence and review skills. d) training and supervisory skills.

a) competence and objectivity.

Which of the following misstatements is most likely to be uncovered during an audit of a client's bank reconciliation? a) failure to record a collection of a note receivable by the bank on the client's behalf b) duplicate payment of a vendor's invoice c) billing a customer at a lower price than indicated by company policy d) payment to an employee for more than the hours actually worked

a) failure to record a collection of a note receivable by the bank on the client's behalf

Comparative financial statements for a public company include the prior year's statements, which were audited by a predecessor auditor. The predecessor's report is not presented along with the comparative financial statements. If the predecessor's report was unqualified, the successor should: a) indicate in the auditor's report that the predecessor auditor expressed an unqualified opinion. b) obtain a letter of representations from the predecessor concerning any matters that might affect the successor's opinion. c) request that the predecessor auditor reissue the prior year's report. d) express an opinion on the current year's statements alone and make no reference to the prior year's statements.

a) indicate in the auditor's report that the predecessor auditor expressed an unqualified opinion.

On February 17, 2012, you discovered that, on November 30, 2011, a flood destroyed the entire uninsured inventory in one of Schmidt's warehouses.

a. Adjust the December 31, 2011 financial statements.

Generally, loss contingencies that are judged to be remote: A) should be disclosed in the footnotes. B) should be recorded in the financial statements. C) should not be disclosed in the footnotes. D) should be recorded in the financial statements and the footnotes

c

Rights and Obligations Assertion (Period End/Balance Sheet)

address whether the entity holds or controls the rights to assets included on the financial statements, and that liabilities are the obligations of the entity.

Accuracy Assertion

addresses whether amounts and other data relating to recorded transactions and events have been recorded appropriately. ***Use GAAP

Existence Assertion (Period End/Balance Sheet)

addresses whether ending balances of assets, liabilities, and equity interests included in the financial statemens ACTUALLY EXIST at the date of the financial statements.

The study of trends, percentage changes, ratios, and other relationships among financial and non financial data is term _________________ _________________.

analytical procedures

What is the primary purpose of analytical procedures performed during the completion phase of the audit?

analytical procedures are required during the competition phase as a final review for material misstatements or financial problems

Confirmation (Audit Procedures) + Info Frequently Confirmed by Auditors

audit evidence obtained by the auditor as a direct written response to the auditor from a third party (the confirming party) in paper form or by electronic or other medium. Frequently Confirmed...Source of Info Cash Balance...Bank A/R Balance...Individual Customer Inventory on Consignment...Consignee N/P...Lending Institution Insurance Contract...Insurance Company

Self-direction, moral independence, and conviction to decide for oneself, rather than accepting the claims of others

autonomy

14-10 When an acquisition is on a FOB origin basis, the inventory and related accounts payable must be recorded in the current period if the goods were a. received prior to the balance sheet date. b. shipped prior to the balance sheet date. c. both shipped and received prior to the balance sheet date. d. paid for in advance.

b

14-16 A file for recording individual purchases, cash disbursements, and purchase returns and allowances for each vendor is the a. cash disbursements transactions file. b. accounts payable master file. c. purchase transaction file. d. summary purchase report.

b

14-19 A client's internal control requires that an approved voucher, a prenumbered purchase order, and a prenumbered receiving report accompany every check payment. To determine whether checks are being issued for unauthorized expenditures, an auditor most likely would select items for testing from the population of all a. purchase orders. b. canceled checks. c. receiving reports. d. approved vouchers.

b

14-20 In assessing control risk for purchases, an auditor vouches a sample of entries in the voucher register to the supporting documents. Which assertion would this test of controls most likely support? a. Completeness. b. Existence or occurrence. c. Valuation or allocation. d. Rights and obligations.

b

14-27 Which of the following TOB procedure is least likely to detect an unrecorded liability? a. Analysis and re-computation of interest expense. b. Analysis and re-computation of depreciation expense. c. Mailing of standard bank confirmation forms. d. Reading of the minutes of meetings of the board of directors.

b

14-3 Internal control is strengthened when the dollar amount of merchandise ordered is omitted from the copy of the purchase order sent to the a. Department that initiated the requisition. b. Receiving department. c. Purchasing agent. d. Accounts payable department.

b

14-30 A major consideration in verifying the ending balance in property, plant, and equipment accounts is the possibility of existing legal encumbrances. TOB procedures to identify possible legal encumbrances would satisfy the specific audit objective for a. existence. b. understandability. c. occurrence. d. valuation.

b

14-35 When using confirmations to provide evidence about the completeness assertion for accounts payable (unrecorded liabilities), the appropriate population most likely would be a. amounts recorded in the accounts payable subsidiary ledger. b. vendors with whom the client has previously done business. c. payees of checks drawn in the month after the year end. d. invoices filed in the client's open invoice file.

b

14-36 In testing plant and equipment balances, an auditor may inspect new additions listed on the analysis of plant and equipment. This procedure is designed to obtain evidence concerning management's assertions about Existence Presentation or Occurrence and Disclosure a. Yes Yes b. Yes No c. No Yes d. No No

b

14-37 In performing a search for unrecorded retirements of fixed assets, an auditor most likely would a. inspect the property ledger and the insurance and tax records, and then tour the client's facilities. b. tour the client's facilities, and then inspect the property ledger and the insurance and tax records. c. analyze the repair and maintenance account, and then tout the client's facilities. d. tour the client's facilities, and then analyze the repair and maintenance account.

b

14-39 Which of the following explanations concerning plant assets most likely would satisfy an auditor who questions management about significant debits to the accumulated depreciation accounts? a. The estimated remaining useful lives of plant assets were revised. b. Plant assets were retired during the year. c. The prior year's depreciation expense was erroneously understated. d. Overheads allocations were revised at year end.

b

14-4 Which of the following is the most effective TOC procedure to detect vouchers prepared for the payment of goods that were not received? a. Counting of goods upon receipt in storeroom. b. Matching of purchase order, receiving report, and vendor invoice for each voucher in the accounts payable department. c. Comparison of goods received with goods requisitioned in receiving department. d. Verification of vouchers for accuracy and approval in internal audit department.

b

14-43 To determine whether accounts payable are complete, an auditor performs a test to verify that all merchandise received is recorded. The population of documents for this test consists of all a. payment vouchers. b. receiving reports. c. purchase requisitions. d. vendors' invoices.

b

14-44 An auditor performs TOB to determine whether all merchandise for which the client was billed was received. The population for this test consists of all a. merchandise received. b. vendor's invoices. c. canceled checks. d. receiving reports.

b

14-46 An auditor's purpose in reviewing the renewal of a note payable shortly after the balance sheet date most likely is to obtain evidence concerning management's assertions about a. existence or occurrence. b. presentation and disclosure. c. completeness. d. valuation or allocation.

b

14-51 Which of the following analytical procedures is least effective in uncovering unrecorded tax liabilities? a. Comparison of sales taxes to sales totals. b. Comparison of withholding taxes to total income. c. Comparison of payroll taxes to payroll totals. d. Comparison of income taxes to total income.

b

14-53 Natural resources such as oil and gas present some unique problems for the auditor. Which of the following does not present an unique problem for the auditor? a. It is often difficult for the auditor to identify the costs associated with the discovery of the natural resources. b. It is often difficult for the auditor to use a geologist to determine the reasonableness of the depletion rate bring used for the natural resources. c. It is often difficult for the auditor to estimate the amount of commercially available resources to be used in determining a depletion rate. d. It is often difficult for the auditor to estimate the amount reclamation costs.

b

16. An employee at an information systems consulting firm recently asked a partner at the firm for a 20% pay raise. The partner said that this request was excessive. To resolve this issue amicably, the employee and the partner agreed that the firm's managing partner, who is well-respected, would mediate this issue and determine the fair amount of the pay raise. All parties agreed to this arrangement. The managing partner who is deciding this issue is a CPA. She: a. Does not have to disclose her conflict of interest to this employee because she is not a practicing CPA b. Does not have to make any disclosures because none are needed c. Does have to disclose her conflict of interest, but she does not need to get the employee's consent d. Does have to disclose her conflict of interest and does need to get the employee's explicit consent

b

19. If a CPA leaves a job in public accounting to commence work as a member of a company's internal accounting staff, this CPA: a. No longer has to be concerned about conflicts of interest b. Still has to be concerned about conflicts of interest involving former clients as well as conflicts of interest arising at her new workplace c. Only has to be concerned about conflicts of interest involving her former clients because industry accountants are not subject to the AICPA's conflict of interest rules d. Does not have to be concerned about conflicts of interest that may arise between loyalty to her new employer and loyalty to her former employer

b

23. A CPA's duty to be free of conflicts of interest is most closely related to the AICPA Code of Professional Conduct's Principle of: a. Due care b. Objectivity c. Professional skepticism d. Nonsubordination of professional judgment

b

31. The concept of "discounting," in the context of accountant conflicts of interest, means that: a. An accountant, in evaluating the reliability of a client statement, applies greater professional skepticism b. A client, in evaluating the reliability of its accountant's statement, applies greater skepticism c. An accountant weighs immediate satisfaction more heavily than deferred satisfaction d. A prospective client, in listening to an accountant, weighs immediate satisfaction more heavily than deferred satisfaction

b

4. In deciding whether a conflict of interest exists, the relevant standard to be applied is whether: a. The CPA subjectively believes that a conflict of interest exists b. A reasonable observer, with reasonable knowledge of the relevant circumstances, would conclude that a conflict of interest exists c. In an accountant-client conflict of interest situation, at least one of the clients believes, reasonably or unreasonably, that a conflict of interest exists d. In a dual-client conflict of interest situation, both of the clients themselves believe, reasonably or unreasonably, that a conflict of interest exists

b

8. A CPA has been asked by an engaged couple to help them budget the amount that they can afford to spend on their upcoming wedding. Both the groom and the bride approached the CPA together and met with him together. The CPA has no prior relationship with either of these individuals. The groom and the bride each have agreed to pay one-half of the CPA's fees. The CPA: a. Currently does not have an accountant-client conflict of interest, but must be alert to the possibility that one will arise b. Currently does not have a dual-client conflict of interest, but must be alert to the possibility that one will arise c. Currently has a conflict of interest and should not accept this professional engagement d. Currently has a conflict of interest, and should agree to render professional services to this couple only if they give their specific consent to waive this conflict of interest

b

An attorney is responding to an independent auditor as a result of the entity's letter of inquiry. The attorney may appropriately limit the response to: A) asserted claims and litigation. B) matters to which the attorney has given substantive attention in the form of legal consultation or representation. C) asserted, overtly threatened, or pending claims and litigation. D) items that have an extremely high probability of being resolved to the entity's detriment.

b

An example of a Type I subsequent event is: A) a tornado that destroys an entity's factory after the balance sheet date. B) an event after the balance sheet date that confirms the auditor's belief (documented prior to the end of the entity's fiscal year) that a large portion of the entity's inventory is obsolete. C) notification of an IRS audit after the balance sheet date. D) the entity's Board of Directors unexpectedly resigns after the balance sheet date.

b

If a lawyer refuses to furnish corroborating information regarding litigation, claims, and assessments, the auditor should: A) honor the confidentiality of the client-lawyer relationship. B) consider the refusal to be a scope limitation. C) seek to obtain the corroborating information from management. D) disclose this fact in a footnote to the financial statements.

b

On February 25, a CPA issued an auditor's report expressing an unqualified opinion on financial statements for the year ended January 31. On March 2, the CPA learned that, on February 11, the entity incurred a material loss on an uncollectible trade receivable as a result of the ongoing deterioration of the financial condition of the entity's principal customer, which finally led to the customer's bankruptcy. Management then refused to adjust the financial statements for this subsequent event. The CPA determined that the information is reliable and that there are creditors currently relying on the financial statements. The CPA's next course of action most likely would be to: A) notify the entity's creditors that the financial statements and the related auditor's report should no longer be relied upon. B) notify each member of the entity's board of directors about management's refusal to adjust the financial statements. C) issue revised financial statements and distribute them to each creditor known to be relying on the financial statements. D) issue a revised auditor's report and distribute it to each creditor known to be relying on the financial statements.

b

The refusal of an entity's attorney to provide a representation on the legality of a particular act committed by the entity is generally: A) sufficient reason to issue a "subject to" qualified opinion. B) considered to be a scope limitation. C) insufficient reason to modify the auditor's report because of the attorney's obligation of confidentiality. D) proper grounds to withdraw from the engagement without further consideration.

b

When a question arises about an entity's continued existence, the auditor should consider factors tending to mitigate the significance of negative information concerning the entity's means for maintaining adequate cash flow. An example of such a factor is the: A) possibility of purchasing certain assets rather than leasing them. B) capability of extending the due dates of existing debt. C) appropriateness of changing depreciation methods from double declining balance to straight line. D) marketability of property and equipment that management plans to keep.

b

Which of the following auditing procedures most likely would assist an auditor in identifying conditions and events that may indicate substantial doubt about an entity's ability to continue as a going concern? A) Inspecting title documents to verify whether any assets are pledged as collateral. B) Confirming with third parties the details of arrangements to maintain financial support. C) Reconciling the cash balance per books with the cutoff bank statement and the bank confirmation. D) Comparing the entity's depreciation and asset capitalization policies to other entities in the industry.

b

Which of the following conditions or events most likely would cause an auditor to have substantial doubt about an entity's ability to continue as a going concern? A) Significant related party transactions are pervasive. B) Usual trade credit from suppliers is denied. C) Arrearages in preferred stock dividends are paid. D) Restrictions on the disposal of principal assets are present.

b

Which of the following matters is an auditor required to communicate to those charged with governance? A) The basis for assessing control risk at low. B) The disagreements with management, if any. C) The auditor's preliminary judgments about materiality levels. D) The justification for performing substantive procedures at interim dates.

b

Which of the following procedures would an auditor most likely perform to obtain evidence about an entity's subsequent events? A) Reconcile bank activity for the month after the balance sheet date with cash activity reflected in the accounting records. B) Ask legal counsel about any developments relating to litigation, claims, or assessments against the company. C) Review the treasurer's monthly reports on temporary investments owned, purchased, and sold. D) Examine on a test basis the purchase invoices and receiving reports for several days after the inventory date.

b

Which of the following ratios is least likely to assist the auditor in determining whether the entity is experiencing financial difficulties? A) Net worth/total liabilities. B) Cash/total assets. C) Total liabilities/total assets. D) Net income before taxes/net sales.

b

Which of the following statements extracted from an entity's lawyer's letter concerning litigation, claims, and assessments most likely would cause the auditor to request clarification? A) "I believe that the possible liability to the company is nominal in amount." B) "I believe that the action can be settled for less than the damages claimed." C) "I believe that the plaintiff's case against the company is without merit." D) "I believe that the company will be able to defend this action successfully."

b

Which of the following statements is correct about an auditor's required communication with management and those charged with governance? A) Any matters communicated to those charged with governance are also required to be communicated to the entity's management. B) The auditor is required to discuss considerations relating to assessment of going concern. C) The auditor does not have any requirement to communicate with anyone outside of management. D) Weaknesses in internal control previously reported to those charged with governance are required to be communicated to those charged with governance after each subsequent audit until the weaknesses are corrected.

b

Which of the following statements is correct concerning an auditor's required communication with those charged with governance? A) This communication is required to occur before the auditor's report on the financial statements is issued. B) This communication should provide timely observations arising from the audit that are significant to the financial reporting process. C) Any significant matter communicated to those charged with governance also should be communicated to management. D) Significant audit adjustments proposed by the auditor and recorded by management need not be communicated to those charged with governance.

b

When reporting on comparative financial statements for a private company, which of the following circumstances should ordinarily cause the auditor to change the previously issued opinion on the prior year's financial statements? a) A scope limitation caused a qualified opinion on the prior year's financial statements, but the current year's opinion is properly unmodified. b) A departure from generally accepted accounting principles caused an adverse opinion on the prior year's financial statements, and those statements have been properly restated. c) A change in accounting principle causes the auditor to make a consistency modification in the current year's audit report. d) The prior year's financial statements are restated following the purchase of another company in the current year.

b) A departure from generally accepted accounting principles caused an adverse opinion on the prior year's financial statements, and those statements have been properly restated.

Engagement letters include all of the following except: a) A list of additional services that will be provided. b) A list of adjusting journal entries. c) Information about the audit fee. d) Arrangements involving the use of specialists.

b) A list of adjusting journal entries.

When an entity moves into a significant new line of business, all of the following increase except: a) Client risk. b) Acceptable audit risk. c) Risk of material misstatement. d) Entity business risk.

b) Acceptable audit risk.

A successor auditor should request the new client to authorize the predecessor auditor to allow a review of the predecessor's a) Engagement letter. b) Audit working papers. c) Engagement letter and audit working papers. d) It would not be typical to allow a review of either the engagement letter or the audit working papers.

b) Audit working papers.

Which of the following best describes the general character of the three generally accepted auditing standards classified as standards of field work? a) The need to maintain an independence of mental attitude in all matters relating to the audit. b) Criteria for audit planning and evidence gathering. c) Criteria for the content of the auditor's report on financial statements and related footnote disclosures. d) Description of the competence, independence, and professional care of persons performing the audit.

b) Criteria for audit planning and evidence gathering.

On the basis of audit evidence gathered and evaluated, an auditor decides to increase the assessed level of risk of material misstatement from that originally planned. To achieve an overall audit risk level that is substantially the same as the planned audit risk level, the auditor would a) Decrease amount of substantive testing. b) Decrease detection risk. c) Increase detection risk. d) Increase materiality levels.

b) Decrease detection risk.

The risk that an auditor will conclude, based on substantive procedures, that a material error does not exist in an account balance when, in fact, such an error does exist is referred to as a) Sampling risk. b) Detection risk. c) Nonsampling risk. d) Inherent risk.

b) Detection risk.

In which of the following circumstances would an auditor usually choose between issuing a qualified opinion or a disclaimer of opinion on a client's financial statements? a) Unreasonable justification for a change in accounting principle. b) Inability of the auditor to obtain sufficient competent evidence. c) Departure from generally accepted accounting principles. d) Inadequate disclosure of accounting policies.

b) Inability of the auditor to obtain sufficient competent evidence.

Which of the following statements best describes what is meant by an unqualified audit opinion? a) Issuance of an unqualified auditor's report indicates that the auditor is not qualified to express an opinion that the client's financial statements are fairly presented in accordance with agreed-upon criteria. b) Issuance of a standard unqualified auditor's report indicates that in the auditor's opinion the client's financial statements are fairly presented in accordance with agreed-upon criteria, with no need for the inclusion of qualifying phrases. c) Issuance of an unqualified auditor's report indicates that in the auditor's opinion the client's financial statements are not fairly enough presented in accordance with agreed-upon criteria to qualify for a clean opinion. d) Issuance of an unqualified auditor's report indicates that the auditor is expressing different opinions on each of the basic financial statements regarding whether the client's financial statements are fairly presented in accordance with agreed-upon criteria.

b) Issuance of a standard unqualified auditor's report indicates that in the auditor's opinion the client's financial statements are fairly presented in accordance with agreed-upon criteria, with no need for the inclusion of qualifying phrases.

As the acceptable level of detection risk decreases, an auditor may change the a) Timing of tests of controls by performing them at an interim date rather than at year-end. b) Nature of substantive procedures from less effective to more effective procedures. c) Timing of tests of controls by performing them at several dates rather than at one time. d) Assessed level of risk of material misstatement to a higher amount.

b) Nature of substantive procedures from less effective to more effective procedures.

The Public Company Accounting Oversight Board's role is to a) Conduct the final review of auditors' work before the auditor's opinion is issued. b) Oversee the auditors of public companies in order to protect the interests of investors. c) Conduct audits of governmental entities. d) Sanction auditors who fail to follow GAAS.

b) Oversee the auditors of public companies in order to protect the interests of investors.

Which of the following is not a part of the role of internal auditors? a) Assisting the external auditors. b) Providing reports on the reliability of financial statements to investors and creditors. c) Consulting activities. d) Operational audits.

b) Providing reports on the reliability of financial statements to investors and creditors.

For publicly-held companies, which of the following is integrated into the audit of financial statements? a) Budgetary information audit. b) The audit of internal controls. c) Audit of management forecasts. d) Audit of interim financial statements.

b) The audit of internal controls.

Which of the following best describes the auditor's responsibility for "other information" included in the annual report to stockholders that contains financial statements and the auditor's report? a) The auditor should extend the examination to the extent necessary to verify the "other information." b) The auditor has no obligation to corroborate the "other information" but should read the "other information" to determine whether it is materially inconsistent with the financial statements. c) The auditor has no obligation to read the "other information." d) The auditor must modify the auditor's report to state that the other information "is unaudited" or "is not covered by the auditor's report."

b) The auditor has no obligation to corroborate the "other information" but should read the "other information" to determine whether it is materially inconsistent with the financial statements.

An auditor is required to establish an understanding with a client regarding the responsibilities for each engagement. This understanding generally includes a) Management's responsibility to guarantee that there are no material misstatements due to fraud. b) The auditor's responsibility to plan and perform the audit to provide reasonable, but not absolute, assurance of detecting material errors or fraud. c) Management's responsibility for providing the auditor with an assessment of the risk of material misstatement due to fraud. d) The auditor's responsibility for the fairness of the financial statements.

b) The auditor's responsibility to plan and perform the audit to provide reasonable, but not absolute, assurance of detecting material errors or fraud.

Which of the following best describes the concept of audit risk? a) The risk of the auditor being sued because of association with an auditee. b) The risk that the auditor will provide a "clean" opinion on financial statements that are, in fact, materially misstated. c) The overall risk that a material misstatement exists in the financial statements. d) The risk that auditors use audit procedures that are inappropriate.

b) The risk that the auditor will provide a "clean" opinion on financial statements that are, in fact, materially misstated.

Recalculation (Audit Procedures)

checking the mathematical accuracy of documents or records... can be performed manually or through the use of information technology...

Eagle Company, a public company, had a computer failure and lost part of its financial data. As a result, the auditor was unable to obtain sufficient audit evidence relating to Eagle's inventory account. Assuming the inventory account is at least material, the auditor would most likely choose either: a) a qualified opinion or an adverse opinion. b) a qualified opinion or a disclaimer of opinion. c) a qualified opinion with no explanatory paragraph or a qualified opinion with an explanatory paragraph. d) an unqualified opinion with no explanatory paragraph or an unqualified opinion with an explanatory paragraph.

b) a qualified opinion or a disclaimer of opinion.

When planning an audit, an auditor should a) evaluate detected misstatements. b) determine overall materiality for audit purposes. c) conclude whether changes in compliance with prescribed internal controls justify reliance on them. d) consider whether the extent of substantive procedures may be reduced based on the results of tests of controls.

b) determine overall materiality for audit purposes.

Operational auditing is oriented primarily toward a) past protection provided by existing internal control. b) efficiency and future improvements to accomplish the goals of management. c) verification that an entity's financial statements are fairly presented. d) the accuracy of data reflected in management's financial records.

b) efficiency and future improvements to accomplish the goals of management.

As lower acceptable levels of both audit risk and materiality are established, the auditor should plan more work on individual accounts to a) decrease the risk of overreliance. b) find smaller errors. c) find larger errors. d) increase the tolerable misstatements in the accounts.

b) find smaller errors.

For what primary purpose does the auditor obtain an understanding of the entity and its environment? a) To determine the audit fee. b) To plan the audit and determine the nature, timing, and extent of audit procedures to be performed. c) To limit audit risk to an appropriately high level. d) To decide which facts about the entity to include in the audit report.

b) to plan the audit and determine the nature, timing and extent of audit procedures to be performed

On February 17, 2012, you discovered that, on February 16, 2012, a flood destroyed the entire uninsured inventory in one of Schmidt's warehouses.

b. Disclose the information in a footnote in the December 31, 2011 financial statements.

"There are no violations or possible violations of laws or regulations whose effects should be considered for disclosure in the financial statements or as a basis for recording a loss contingency." The foregoing passage most likely is from a(n): A) entity engagement letter. B) report on compliance with laws and regulations. C) management representation letter. D) attestation report on internal controls.

c

"There have been no communications from regulatory agencies concerning noncompliance with or deficiencies in, financial reporting practices that could have a material effect on the financial statements." The foregoing passage is most likely from a: A) report on internal control. B) special report. C) management representation letter. D) letter for underwriters.

c

1. When a CPA leaves her job working as an IRS Revenue Agent to accept a job in the Tax Department of a public accounting firm, she primarily: a. Has to be concerned about conflicts of interest between her new employer and its competitors b. Has to be concerned about conflicts of interest between her new employer and other employers whose job offers she rejected c. Has to be concerned about conflicts of interest arising between one of her clients and her former employer d. Does not have to be concerned about using information learned at the IRS in assisting new clients because a CPA has a duty to use all of her knowledge to best serve her clients

c

14-12 Under which of the following circumstances would it be advisable for the auditor to confirm accounts payable with creditors? a. Internal control over accounts payable is adequate, and there is sufficient evidence on hand to minimize the risk of a material misstatement. b. Confirmation response is expected to be favorable, and accounts payable balances are of immaterial amounts. c. Creditor statements are not available, and internal control over accounts payable is unsatisfactory. d. The majority of accounts payable balances are with associated companies.

c

14-14 The internal control which requires that "checks are pre-numbered and accounted for" satisfies the specific objective of a. accuracy. b. existence. c. completeness. d. cut-off.

c

14-15 The point at which most companies first recognize the acquisition and related liability on their records is when the a. purchase requisition is completed. b. purchase order is completed. c. receiving report is completed. d. vendor's invoice is paid.

c

14-17 An auditor traced a sample of purchase orders to the purchases journal and the cash disbursement journal for the TOC specific objective of a. identify unusually large purchases that should be investigated further. b. verifying that cash disbursements were for goods actually received. c. determining that purchases were properly recorded. d. test whether payments were for goods actually ordered.

c

14-18 In auditing accounts payable, an auditor's TOC procedures would focus primarily on management's assertion of a. existence or occurrence. b. presentation and disclosure. c. completeness. d. valuation or allocation.

c

14-23 Which of the following audit procedures is best for identifying unrecorded accounts payable? a. Examining unusual relationships between monthly accounts payable balances and recorded cash payments. b. Reconciling vendors' statements to the file of receiving reports to identify items received just prior to the balance sheet date. c. Reviewing cash disbursements recorded subsequent to the balance sheet date to determine whether the related payables apply to the prior period. d. Investigated payables recorded just prior to and just subsequent to the balance sheet date to determine whether they are supported by receiving reports.

c

14-24 In auditing accounts payable, an auditor's procedures most likely would focus primarily on management's assertion of a. existence or occurrence. b. presentation and disclosure. c. completeness. d. valuation or allocation.

c

14-25 In testing for unrecorded retirements of equipment, auditor most likely would a. select items of equipment from the accounting records and then locate them during the plant tour. b. compare depreciate journal entries with similar prior-year entries in search of fully depreciated equipment. c. inspect items of equipment observed during the plant tour and then trace them to the equipment master file. d. scan the general journal for unusual equipment additions and excessive debits to repairs and maintenance expense.

c

14-32 Which of the following TOB procedures would an auditor most likely perform to verify the management's assertions of existence or occurrence and valuation or allocation of recorded accounts payable? a. Investigating the open purchase order file to ascertain that prenumbered purchase orders are used and accounted for. b. Reviewing the client's mail for a reasonable period of time after the year end to search for unrecorded vendor's invoices. c. Vouching selected entries in the accounts payable subsidiary ledger to purchase orders and receiving reports. d. Confirm accounts payable balances with known suppliers who have zero balances.

c

14-40 When auditing prepaid insurance, an auditor discovered that the original insurance policy on plant equipment is not available for inspection. The insurance policy's absence most likely indicated the possibility of a(n) (Hint: Think of the beneficiary) a. insurance premium due but not recorded. b. deficiency in the coinsurance provision. c. lien on the plant equipment. d. understatement of insurance expense.

c

14-45 An auditor traced a sample of purchases orders and the related receiving reports to the purchases journal and the cash disbursements journal. The purpose of this TOB procedure most likely was to a. identify unusually large purchases that should be investigated further. b. verify that cash disbursements were for goods actually received. c. determine that purchases were properly recorded. d. test whether payments were for goods actually ordered.

c

14-48 When there are numerous property and equipment transactions during the year, an auditor who plans to assess control risk at a low level usually performs a. tests of controls and extensive tests of property and equipment balances at the end of the year. b. analytical procedures for current year property and equipment transactions. c. tests of controls and limited tests of property and equipment balances at the end of the year. d. attribute sampling tests of property and equipment balances at the end of the year.

c

14-49 Equipment acquisitions that are misclassified as maintenance expense most likely would be detected by an internal control procedure that provides for a. segregation of duties for employees in the accounts payable department. b. independent verification of invoices for disbursements recorded as equipment acquisitions. c. analysis of variances with a formal budgeting system. d. authorization by the board of directors of significant equipment acquisitions.

c

14-54 Which of the following is true of capital (finance) leases as compared to operating leases? a. Only rent expense is reflected in the income statement. b. The leased asset does not appear o the balance sheet. c. Liabilities include the lease obligation. d. Future minimum lease obligations are not required to be disclosed.

c

14-6 To determine whether the recording of purchases transactions is complete, an auditor performs a TOC procedure to verify that all merchandise received is recorded. The population of documents for this test consists of all a. vendor invoices. b. purchase orders. c. receiving reports. d. canceled checks.

c

When an auditor increases the assessed level of risk of material misstatement because certain control procedures were determined to be ineffective, the auditor would most likely increase the a) Extent of tests of controls. b) Level of detection risk. c) Extent of substantive tests. d) Level of inherent risk.

c) Extent of substantive tests.

18. Two CPAs are good friends as well as partners together in a CPA firm. They recently invested their money together to buy an apartment building. One of the tenants in the apartment building is an employee of their CPA firm, and another tenant is the father of one of the two CPAs. The father's lease terminates soon and the father wishes to renew the lease at the same rental amount that he currently is paying. Under the AICPA's Code of Professional Conduct: a. Two conflicts of interest exist b. The father's status as a tenant creates a conflict of interest that cannot be corrected by safeguards c. The father's status as a tenant creates a conflict of interest that can be remedied by safeguards d. No conflicts of interest exist

c

2. Many companies adopt policies that preclude key employees from owning stock in company suppliers. Companies establish this type of policy to: a. Make it more difficult for key employees to leave their jobs to commence working for a supplier b. Make it more difficult for key employees to share confidential trade secrets with a supplier c. Prevent conflicts of interest that might be difficult to detect d. Prevent conflicts of interest, even though they usually are easy to detect

c

21. A CPA provides services to two clients in the same industry. Both clients have been informed of this fact and have acknowledged their approval in writing. As a result: a. The CPA nonetheless is violating professional standards concerning conflicts of interest b. The duty of confidentiality has been waived and does not apply to the two accountant-client relationships mentioned c. Must be careful to not share specific information learned from one client with the other client d. Must be careful to not utilize general industry experience gained from serving one client to benefit the other client

c

26. The materiality of a conflict of interest should be evaluated from the perspective of: a. Whether a reasonable client would consider the conflict to be material b. Whether a client, acting unreasonably or reasonably, would consider the conflict to be material c. Whether an objective observer who is reasonably informed about the surrounding facts would conclude that a conflict of interest is material d. Whether the CPA subjectively, from his or her own vantage point, has concerns about two or more material interests clashing

c

29. The "Fund of Funds" court decision stands for the proposition that: a. Auditors inherently have conflicts of interest with their clients b. Segregating one team of auditors at a CPA firm from another team does not necessarily prevent conflict of interest issues from affecting the firm's ethical obligations c. The duty of loyalty to one client sometimes has to override the duty of confidentiality owed to a different client d. The duty of loyalty to a client sometimes has to override the duty of confidentiality owed to that same client

c

32. An accountant's conflict of interest: a. Always can be waived by a client b. Never can be waived by a client c. Sometimes cannot be waived by a client d. Cannot be waived by the doctrine of implied consent, according to the IFAC Code of Conduct

c

5. A CPA has to focus on whether a conflict of interest exists: a. Only at the outset of a professional relationship b. Only after a professional relationship has commenced c. Both at the outset of a professional relationship and throughout the conduct of a professional relationship d. In all professional relationships other than the provision of voluntary services to a charitable, civic, or religious organization

c

A Type II subsequent event usually requires: A) an adjustment to the financial statements and the footnotes. B) an adjustment to the financial statements but no special disclosure is required. C) disclosure in the footnotes. D) neither an adjustment to the financial statements nor disclosure in the footnotes.

c

A disclosure of a contingent liability in the footnotes is made rather than adjusting the financial statement accounts when: A) the outcome of the event is judged to be probable and the loss can be reasonably estimated. B) the loss can be reasonably estimated, but the outcome is unknown. C) the outcome of the event is judged to be reasonably possible or the loss cannot be reasonably estimated. D) the outcome is unknown and the loss is reasonably estimable but the entity does not want to book the loss.

c

An auditor should obtain evidential matter relevant to all the following factors concerning third-party litigation against an entity except the: A) period in which the underlying cause for legal action occurred. B) probability of an unfavorable outcome. C) jurisdiction in which the matter will be resolved. D) existence of a situation indicating an uncertainty as to the possible loss.

c

An entity has violated a minor requirement of its bond indenture that could result in the trustee requiring immediate payment of the principal amount due. The entity refuses to seek a waiver from the bond trustee. Request for immediate payment is not considered likely. Under these circumstances, the auditor must: A) require classification of bonds payable as a current liability. B) contact the bond trustee directly. C) evaluate whether there is substantial doubt about the entity's ability to continue as a going concern. D) obtain an opinion from the company's attorney as to the likelihood of the trustee's enforcement of the requirement.

c

As part of an audit, a CPA often requests a management representation letter from the entity. Which one of the following is not a valid purpose of such a letter? A) To provide audit evidence. B) To emphasize to the entity their responsibility for the fairness of the financial statements. C) To satisfy himself or herself that a certain account balance is fairly stated when certain customary auditing procedures are not performed. D) To provide possible protection to the CPA against a charge of knowledge in cases where fraud is subsequently discovered to have existed in the accounts.

c

For a CPA who performs auditing services, the most significant conflict of interest that is likely to arise is between: a. Two clients who both sell the same product in different geographic markets b. Two clients who both are aggressively pursuing sales to the same prospective customer c. The client's interest and the public interest d. The CPA's interest and another CPA's interests

c

For which of the following matters should an auditor obtain written management representations? A) Management's cost-benefit justifications for not correcting internal control weaknesses. B) Management's knowledge of future plans that may affect the price of the entity's stock. C) Management's addressing control deficiencies communicated to the audit committee during previous engagements. D) Management's acknowledgment of its responsibility for employees' violations of laws.

c

If an auditor dates the auditor's report on financial statements for the year ended December 31, 2013, as of February 10, 2014, except for Note J, as to which the date is March 3, 2014, the auditor is acknowledging responsibility to actively search for and ensure proper handling by management of: A) all subsequent events occurring through March 3, 2014. B) all subsequent events occurring through February 10, 2014. C) all subsequent events occurring through February 10, 2014 and the specific subsequent event referred to in Note J through March 3, 2014. D) only the specific subsequent event referred to in Note J as of March 3, 2014.

c

Subsequent events for which the auditor has a responsibility to actively search are defined as events that occur subsequent to the: A) balance sheet date. B) date of the auditor's report. C) balance sheet date but prior to the issuance of the financial statements. D) date of the auditor's report and concern contingencies that are not reflected in the financial statements.

c

The adverse effects of events causing an auditor to believe there is substantial doubt about an entity's ability to continue as a going concern would most likely be mitigated by evidence relating to the: A) ability to expand operations into new product lines in the future. B) feasibility of plans to purchase leased equipment at less than market value. C) marketability of assets that management plans to sell. D) committed arrangements to convert preferred stock to long-term debt.

c

The date of the management representation letter generally coincides with the: A) date of the latest subsequent event referred to in the notes to the financial statements. B) balance sheet date. C) date of the auditor's report. D) date of the engagement agreement.

c

When auditing contingent liabilities, which of the following procedures would be least effective? A) Reading the minutes of the board of directors. B) Obtaining a legal letter. C) Examining customer confirmation replies. D) Examining invoices for legal services.

c

Which of the following events occurring after the issuance of an entity's financial statements and the auditor's report most likely would cause the auditor to make further inquiries about the previously issued financial statements? A) An uninsured natural disaster occurs that may affect the entity's ability to continue as a going concern. B) A contingency is resolved that had been disclosed in the audited financial statements. C) New information is discovered concerning undisclosed lease transactions in the audited period. D) A subsidiary that accounts for 25 percent of the entity's consolidated net income is sold.

c

Which of the following expressions is least likely to be included in a management representation letter? A) No events have occurred subsequent to the balance sheet date that require adjustment to or disclosure in the financial statements. B) The company has complied with all aspects of contractual agreements that would have a material effect on the financial statements in the event of noncompliance. C) Management acknowledges responsibility for illegal actions committed by its employees. D) Management has made available all financial statements and related data.

c

Which of the following procedures would an auditor ordinarily perform during the review of subsequent events? A) An analysis of related party transactions for the discovery of possible irregularities. B) A review of the cutoff bank statements for the period after the year-end. C) A reading of the available minutes of meetings of stockholders, directors, or other committees for the subsequent-events period. D) An investigation of material weaknesses in internal control previously communicated to the entity.

c

When reporting on comparative financial statements for a private company, which of the following circumstances should ordinarily cause the auditor to change the previously issued opinion on the prior year's financial statements? a) A change in accounting principle causes the auditor to make a consistency modification in the current year's audit report. b) A scope limitation caused a qualified opinion on the prior year's financial statements, but the current year's opinion is properly unmodified. c) A departure from generally accepted accounting principles caused an adverse opinion on the prior year's financial statements, and those statements have been properly restated. d) The prior year's financial statements are restated following the purchase of another company in the current year.

c) A departure from generally accepted accounting principles caused an adverse opinion on the prior year's financial statements, and those statements have been properly restated.

Which of these statements concerning illegal acts by clients is correct? a) An auditor has no responsibility to detect illegal acts by clients that have an indirect effect on the financial statements. b) An auditor considers illegal acts from the perspective of the reliability of management's representations rather than their relation to audit objectives derived from financial statement assertions. c) An auditor's responsibility to detect illegal acts that have a direct and material effect on the financial statements is the same as that for errors and fraud. d) An audit in accordance with generally accepted auditing standards normally includes audit procedures specifically designed to detect illegal acts that have an indirect but material effect on the financial statements.

c) An auditor's responsibility to detect illegal acts that have a direct and material effect on the financial statements is the same as that for errors and fraud.

To provide for the greatest degree of independence in performing internal audit activities, the internal audit function most likely should report to the a) Vice-President - Finance. b) Corporate controller. c) Audit committee of the board of directors. d) Corporate stockholders.

c) Audit committee of the board of directors.

The achieved (actual) level of audit risk a) Can always be accurately assessed by the auditor. b) Should be greater than or equal to acceptable audit risk. c) Can never be known with certainty. d) Is the same for all audit engagements.

c) Can never be known with certainty.

Evaluating a prospective client requires which of the following steps? a) Communicate with the SEC. b) Preplan the audit. c) Determine if the firm is independent of the client. d) Communicate with the AICPA.

c) Determine if the firm is independent of the client.

The risk of material misstatement differs from detection risk in that it a) Arises from the misapplication of auditing procedures. b) May be assessed in either quantitative or qualitative terms. c) Exists independently of the actions of the auditor. d) Can be changed at the auditor's discretion

c) Exists independently of the actions of the auditor.

Which of the following statements best describes a relationship between sample size and other elements of auditing? a) If materiality increases, so will the sample size. b) If the desired level of assurance increases, sample sizes can be smaller. c) If materiality decreases, sample size will need to increase. d) There is no relationship between sample size and materiality or the desired level of assurance.

c) If materiality decreases, sample size will need to increase.

The auditor can respond to an increased risk of fraud by doing all of the following except: a)Evaluating whether the accounting policies selected may be indicative of fraudulent financial reporting through earnings management. b) Assigning more experienced personnel to the audit. c) Increasing detection risk. d) Taking steps to obtain more reliable evidence.

c) Increasing detection risk.

Which of the following factors most likely would lead a CPA to conclude that a potential audit engagement should be rejected? a) The details of most recorded transactions are not available after a specified period of time. b) Internal control activities requiring segregation of duties are subject to management override. c) It is unlikely that sufficient appropriate evidence is available to support an opinion on the financial statements. d) Management has a reputation for consulting with several accounting firms about significant accounting issues.

c) It is unlikely that sufficient appropriate evidence is available to support an opinion on the financial statements.

Increased fraud risk could result in all of the following except: a) Lower detection risk. b) Higher inherent risk. c) Lower control risk. d) Higher client risk.

c) Lower control risk.

The Audit Committee consists of a) Members of management. b) A subcommittee of the AICPA who establish the SAS. c) Members of the Board of Directors. d) Appointed government overseers.

c) Members of the Board of Directors.

Because of the risk of material misstatement, an audit of financial statements in accordance with generally accepted auditing standards should be planned and performed with an attitude of a) Objective cynicism. b) Independent differentialism. c) Professional skepticism. d) Impartial conservatism.

c) Professional skepticism.

The first PCAOB general standard requires that the examination of financial statements is to be performed by a person or persons having adequate technical training and a) Independence with respect to the financial statements and supplementary disclosures. b) Exercising professional care as judged by peer reviewers. c) Proficiency as an auditor, which likely has been acquired from previous experience. d) Objectivity as an auditor as verified by proper supervision.

c) Proficiency as an auditor, which likely has been acquired from previous experience.

The basic purpose of a financial statement audit is to a) Detect fraud. b) Examine individual transactions so that the auditor may certify as to their validity. c) Provide assurance regarding whether the auditee's financial statements are fairly stated. d) Assure the consistent application of correct accounting procedures.

c) Provide assurance regarding whether the auditee's financial statements are fairly stated.

Which of the following best describes the auditor's responsibility for "other information" included in the annual report to stockholders that contains financial statements and the auditor's report? a) The auditor has no obligation to read the "other information." b) The auditor should extend the examination to the extent necessary to verify the "other information." c) The auditor has no obligation to corroborate the "other information" but should read the "other information" to determine whether it is materially inconsistent with the financial statements. d) The auditor must modify the auditor's report to state that the other information "is unaudited" or "is not covered by the auditor's report."

c) The auditor has no obligation to corroborate the "other information" but should read the "other information" to determine whether it is materially inconsistent with the financial statements.

The accuracy of information included in footnotes accompanying the audited financial statements issued by a company whose shares are traded on a stock exchange is the primary responsibility of a) The stock exchange officials. b) The independent auditor. c) The company's management. d) The Securities and Exchange Commission.

c) The company's management.

Which of the following best describes the general character of the three PCAOB generally accepted auditing standards that are classified as standards of fieldwork? a) The competence, independence, and professional care of persons performing the audit. b) Criteria for the content of the auditor's report on financial statements and related footnote disclosures. c) The criteria of audit planning and evidence-gathering. d) The need to maintain independence in mental attitude in all matters relating to the audit.

c) The criteria of audit planning and evidence-gathering.

Which of the following best describes why an independent auditor is asked to express an opinion on the fair presentation of financial statements? a) It is difficult to prepare financial statements that fairly present a company's financial position and changes in cash flows without the expertise of an independent auditor. b) It is management's responsibility to seek available independent aid in the appraisal of the financial information shown in its financial statements. c) The opinion of an independent party is needed because a company is not likely to be considered objective with respect to its own financial statements. d) It is a customary courtesy that all stockholders of a company receive an independent report on management's stewardship in managing the affairs of the business.

c) The opinion of an independent party is needed because a company is not likely to be considered objective with respect to its own financial statements.

Which of the following statements about the study of auditing is NOT true? a) The study of auditing can be valuable to future accountants and business decision makers whether or not they plan to become auditors. b) The study of auditing focuses on learning the analytical and logical skills necessary to evaluate the relevance and reliability of information. c) The study of auditing focuses on learning the rules, techniques, and computations required to analyze financial statements. d) The study of auditing begins with the understanding of a coherent logical framework and techniques useful for gathering and analyzing evidence about others' assertions.

c) The study of auditing focuses on learning the rules, techniques, and computations required to analyze financial statements.

Which of the following procedures would not be used to obtain an understanding of the entity and its environment? a) Observe entity operations. b) Reperform entity processes. c) Verify proper valuation of inventory subject to technological obsolescence. d) Review prior year's audit documentation.

c) Verify proper valuation of inventory subject to technological obsolescence.

Auditing standards require auditors to make certain inquiries of management regarding fraud. Which of the following inquiries is required? a) Management's attitude about hiring ethical employees. b) Management's attitudes toward regulatory authorities. c) Whether management has any knowledge of fraud that has been perpetrated on or within the entity. d) Whether management has ever intentionally violated the securities laws.

c) Whether management has any knowledge of fraud that has been perpetrated on or within the entity.

Comparative financial statements for a public company include the prior year's statements, which were audited by a predecessor auditor. The predecessor's report is not presented along with the comparative financial statements. If the predecessor's report was unqualified, the successor should: a) request that the predecessor auditor reissue the prior year's report. b) obtain a letter of representations from the predecessor concerning any matters that might affect the successor's opinion. c) indicate in the auditor's report that the predecessor auditor expressed an unqualified opinion. d) express an opinion on the current year's statements alone and make no reference to the prior year's statements.

c) indicate in the auditor's report that the predecessor auditor expressed an unqualified opinion.

The Public Company Accounting Oversight Board a) is a quasi-governmental organization that has a policy to ignore public comment and input in the process of setting auditing standards. b) is a quasi-governmental organization that is independent of the SEC in setting auditing standards. c) is a quasi-governmental organization that has legal authority to set auditing standards for audits of public companies. d) is a quasi-governmental organization that has legal authority to set accounting standards for public companies.

c) is a quasi-governmental organization that has legal authority to set auditing standards for audits of public companies.

An auditor includes a separate paragraph in an otherwise unmodified financial statement audit report to emphasize that the entity being reported upon had significant transactions with related parties. The inclusion of this separate paragraph: a) necessitates a revision of the opinion paragraph to include the phrase "with the foregoing explanation." b) is considered an "except for" qualification of the opinion. c) is appropriate and would not negate the unmodified opinion. d) violates generally accepted auditing standards if this information is already disclosed in footnotes to the financial statements.

c) is appropriate and would not negate the unmodified opinion.

Assurance services may improve all of the following except a) Relevance. b) Credibility. c) Periodicity. d) Reliability.

c) periodicity

When reporting on financial statements prepared on the basis of accounting used for income tax purposes, the auditor should include in the report a paragraph that: a) justifies the use of the income tax basis of accounting. b) refers to the authoritative pronouncements that explain the income tax basis of accounting being used. c) states that the income tax basis of accounting is a basis of accounting other than generally accepted accounting principles. d) emphasizes that the financial statements have not been examined in accordance with generally accepted auditing standards.

c) states that the income tax basis of accounting is a basis of accounting other than generally accepted accounting principles.

When reporting on financial statements prepared on the basis of accounting used for income tax purposes, the auditor should include in the report a paragraph that: a) refers to the authoritative pronouncements that explain the income tax basis of accounting being used. b) emphasizes that the financial statements have not been examined in accordance with generally accepted auditing standards. c) states that the income tax basis of accounting is a basis of accounting other than generally accepted accounting principles. d) justifies the use of the income tax basis of accounting.

c) states that the income tax basis of accounting is a basis of accounting other than generally accepted accounting principles.

King, CPA, was engaged to audit the financial statements of Chang Company, a private company, after its fiscal year had ended. King neither observed the inventory count nor confirmed the receivables by direct communication with debtors but was satisfied that both were fairly stated after applying appropriate alternative procedures. King's financial statement audit report most likely contained a(n): a) qualified opinion. b) unmodified opinion with an emphasis-of-matter paragraph. c) unmodified opinion. d) disclaimer of opinion.

c) unmodified opinion.

Management believes and the auditor is satisfied, that a material loss probably will occur when pending litigation is resolved. Management is unable to make a reasonable estimate of the amount or range of the potential loss, but fully discloses the situation in the notes to the financial statements. If the auditor wishes to call attention to the matter and management does not make an accrual in the financial statements, the auditor should issue a(an) a) qualified report due to a scope limitation. b)qualified report due to a departure from GAAP. c) unqualified report with an explanatory/emphasis-of-matter paragraph. d) a standard unmodified auditor's report.

c) unqualified report with an explanatory/emphasis-of-matter paragraph.

Inspection

critical review of a document

10. The concept of "moral licensing," in the context of conflicts of interest, relates to: a. A person receiving actual consent to proceed in a professional relationship, despite the potential for a conflict of interest to arise b. A person receiving implied consent to proceed in a professional relationship, despite the potential for a conflict of interest to arise c. A person who potentially will benefit from a conflict of interest agreeing to make periodic payments to reimburse any party who is adversely affected by the conflict of interest d. A person making exaggerated claims about goods or services after having disclosed influences that create the appearance of a conflict of interest

d

12. An accountant and a prospective client are contemplating entering into a professional relationship in which the accountant will provide monthly bookkeeping services and annual tax return preparation services for this client's hair salon. The accountant's teenage daughter told this prospective client that her father is "an amazing accountant," which facilitated the professional introduction. The accountant's daughter works for this prospective client as a receptionist. In all likelihood, the accountant: a. May not provide services to this prospective client b. May provide only tax preparation services to this prospective client because a conflict of interest presently exists c. May provide only bookkeeping services to this prospective client because a conflict of interest presently exists d. May provide all requested services because a conflict of interest does not currently exist and is not reasonably foreseeable

d

14-1 Which of the following is an internal control that would prevent paid cash disbursement documents from being presented for payment a second time? a. Unsigned checks should be prepared by individuals who are responsible for signing checked. b. Cash disbursement documents should be approved by at least two responsible management officials. c. The date on cash disbursement documents should be within a few days of the date that the document is presented for payment. d. The official signing the check should compare the check with the documents and should deface the documents.

d

14-22 Which of the following statements describes an important distinction between the confirmation of accounts payable with suppliers and confirmation of accounts receivable with debtors? a. Confirmation of accounts payable with suppliers is more widely accepted auditing procedure than is confirmation of accounts receivable with debtors. b. Statistical sampling techniques are more widely accepted in the confirmation of accounts payable than in the confirmation of account receivables. c. As compared with the confirmation of accounts payable, the confirmation of accounts receivable will tend to emphasize accounts with zero balances at the balance sheet date. d. It is less likely that the confirmation request sent to the supplier will show the amount owed him than that the request sent to the debtor will show the amount due from him.

d

14-29 The audit procedures used to verify accrued liabilities differ from those used to verify accounts payable because a. accrued liability balances are less material than accounts payable balances. b. accrued liabilities at year-end will become accounts payable in the following year. c. evidence supporting accrued liabilities is non-existent, whereas evidence supporting accounts payable is readily available. d. accrued liabilities usually pertain to services of a continuing nature, whereas accounts payable are the result of completed transactions.

d

14-31 The starting point for the verification of current year acquisitions of property, plant, and equipment is normally a. the property, plant, and equipment account in the general ledger. b. the acquisitions journal. c. the purchase requisitions file. d. a schedule obtained from the client of all acquisitions recorded in the general ledger during the year.

d

14-33 Which of the following TOB procedures would an auditor most likely perform to search for unrecorded liabilities? a. Vouch a sample of cash disbursements recorded just after year end to next year's receiving reports and vendor invoices. b. Scan the cash disbursements entries recorded just before year end for indications of unusual transactions. c. Compare a sample of purchase orders issued just after year end with the year-end accounts payable trial balance. d. Trace a sample unmatched receiving report file in the current accounting period to accounts payable entries recorded in the next accounting period.

d

14-34 The cutoff TOB procedure designed to detect purchases made before the end of the year that have been intentionally recorded in the subsequent year's purchases provides assurance about management's assertion of a. presentation and disclosure. b. completeness. c. existence or occurrence. d. valuation or allocation.

d

14-41 In TOB of property and equipment, an auditor vouches significant debits from the repairs and maintenance expense account to determine a. noncapitalizable expenditure for repairs and maintenance have been recorded in the proper period. b. capitalizable expenditure for property and equipment have been properly charged to expense. c. noncapitalizable expenditure for repairs and maintenance have been properly charged to expense. d. capitalizable expenditure for property and equipment have not been charged to expense.

d

Which of the following would be considered a nonattest assurance service engagement? I. Expressing an opinion about the reliability of an entity's financial statements. II. Reviewing and commenting on an entity-prepared business plan. a) I only. b) II only. c) Both I and II. d) Neither I nor II.

d) Neither I nor II.

14-50 Which of the following TOB procedures would an auditor least likely perform to search for unrecorded liabilities? a. Reconcile the amounts in vendor monthly statements with the amounts in the client's accounts payable subsidiary ledger. b. Trace a sample of large-dollar amounts from vendor invoices to receiving reports, then trace to the purchases journal and cash disbursement journal in the next accounting period to determine if the liabilities relate to the current accounting period. c. Confirm a sample of accounts payable including accounts with small or zero balance. d. Vouch selected balances from the accounts payable trial balance to the voucher packets and vendor invoices.

d

14-52 When a business combination resulted in the recording of goodwill by the client, the auditors should not a. perform an annual TOB on goodwill impairment. b. review the allocation of the lump-sum acquisition cost among tangible assets, identifiable assets, and goodwill. c. determine whether goodwill is properly allocated to all the various reporting units acquired in the acquisition. d. ask the client's management to assist them in evaluating the reasonableness of the valuation and allocation.

d

14-7 Mr. A, the purchasing agent of a wholesale hardware company, has a relative who owns a retail hardware store. Mr. A arranged for hardware to be delivered by manufacturers to the retail store on a C.O.D. basis, thereby enabling his relative to buy at his hardware company's wholesale prices. Mr. A was probably able to accomplish this because of the wholesale company's poor internal control over a. purchase requisition. b. cash receipts. c. perpetual inventory records. d. purchase orders.

d

17. A CPA firm knows that clients find it difficult to change auditors after a professional relationship has been in place for several years because the termination of an auditor often makes investors concerned about the trustworthiness of a company's financial statements. To take advantage of this fact, this CPA firm routinely bids on new audit engagements at a fee that is likely to be 25 percent lower than the fees bid by its competitors. The CPA firm plans to recoup this fee differential by steadily raising its fees to above-market rates in subsequent years. The CPA firm's policy creates: a. A current conflict of interest b. A foreseeable future conflict of interest c. An appearance of a conflict of interest, but not an actual one d. No conflict of interest

d

22. An Oklahoma CPA provides management advisory services to a large public utility that provides electrical power throughout Oklahoma and is the only supplier of electricity to residential users in Oklahoma. The CPA has been asked to prepare a request for a utility rate increase that will be submitted to state regulators for their approval. The CPA: a. Has a conflict of interest that should be disclosed to his client b. Does not have to be concerned about conflicts of interest because the CPA only provides management advisory services c. Does not have to be concerned about conflicts of interest because utility rates ultimately will be set by independent state regulators d. Does not have a material conflict of interest because the requested rate increase will apply to all Oklahoma citizens who use this utility's electricity services

d

25. According to the IFAC Code of Conduct, the determination of whether a professional accountant has, or does not have, a conflict of interest is resolved by: a. The IFAC Ethics Committee, if the conflict of interest is challenged by the accountant's client b. Balancing costs to the client and to society against the financial benefits to the CPA c. Applying the Public Interest Principle d. By the professional accountant, using professional judgment

d

28. A CPA performs tax advisory services, but it does not provide tax return preparation services. For this CPA, the most significant conflict of interest that is likely to arise is between: a. The AICPA's Code of Professional Conduct and IRS Regulations regarding professional conduct b. The Internal Revenue Code and state tax codes c. The client and the IRS d. The duty of loyalty to the client and a duty to the public interest

d

30. In the "Fund of Funds" case, the plaintiff contended that: a. The defendant CPA firm owed it due care in performing professional services and failed to do so b. The defendant CPA firm violated the Integrity and Objectivity Rule c. The defendant CPA firm pursued its own self-interest over the public interest d. The defendant CPA firm had a duty to utilize all available information in performing professional services for the plaintiff

d

6. A CPA usually charges $3,000 to prepare a corporate income tax return, including all supplemental schedules. This CPA has agreed to charge the CFO of a mid-size corporation only $10 to prepare the CPA's personal tax return and $5,000 to prepare the corporation's tax return. This CPA: a. Is engaged in moral licensing b. Has an accountant-client conflict of interest c. Has a dual-client conflict of interest d. Is not acting with integrity

d

After an audit report containing an unqualified opinion on a nonpublic entity's financial statements is issued, the auditor learns that the entity has decided to sell the shares of a subsidiary that accounts for 30 percent of its revenue and 25 percent of its net income. The auditor should: A) determine whether the information is reliable and, if it is determined to be reliable, request that revised financial statements be issued. B) notify the entity that the auditor's report may no longer be associated with the financial statements. C) describe the effects of this subsequently discovered information in communications with persons known to be relying on the financial statements. D) take no action because the auditor has no obligation to make any further inquiries.

d

After issuance of the financial statements and the accompanying auditor's report, the auditor has no obligation to make any further inquiries with respect to audited financial statements covered by an auditor's report unless: A) a lawsuit in which risk of loss was considered remote is resolved in the company's favor. B) a development occurs that may affect the entity's ability to continue as a going concern. C) a material fraud is initiated by an employee after the report is issued. D) evidence of significant, non-arms-length, related party transactions that happened prior to year-end is discovered.

d

An auditor will ordinarily examine invoices from lawyers primarily in order to: A) substantiate accruals. B) assess the legal ramifications of litigation in progress. C) estimate the dollar amount of contingent liabilities. D) identify actual or potential litigation against the entity.

d

Due to the conflict of interest rules, a CPA may: a. Not provide professional services to two companies in the same industry b. Not provide professional services to two companies who do business with one another c. Not provide professional services to two companies who do business with one another if the sales or services are material in amount to one of the parties d. Generally provide professional services to two companies who do business with one another, even if the sales or services are material in amount to one or both parties

d

The management letter is used: A) to allow management to document, in writing, oral representations to the auditor. B) to confirm the terms of the audit engagement. C) to list all reportable conditions with respect to internal controls. D) to make recommendations to the entity based on observations made during the audit.

d

The primary reason an auditor requests letters of inquiry be sent to an entity's attorneys is to provide the auditor with: A) a description and evaluation of litigation, claims, and assessments that existed at the date of the balance sheet. B) an expert opinion as to whether a loss is possible, probable, or remote. C) the opportunity to examine the documentation concerning litigation, claims, and assessments. D) corroboration of the information furnished by management concerning litigation, claims, and assessments.

d

Which of the following auditing procedures is ordinarily performed last? A) Confirming accounts payable. B) Testing the purchasing function. C) Reading the minutes of directors' meetings for evidence of authorization of acquisition of fixed assets. D) Obtaining a management representation letter.

d

Which of the following is not an audit procedure that the independent auditor would perform with respect to litigation, claims, and assessments in audit procedures for subsequent events and contingent liabilities? A) Inquire of and discuss with management the policies and procedures adopted for identifying, evaluating, and accounting for litigation, claims, and assessments. B) Obtain a legal letter. C) Obtain a written representation from management that all litigation, asserted and unasserted claims, and assessments have been disclosed. D) Confirm directly with the entity's lawyer that all claims have been recorded in the financial statements.

d

Which of the following material events occurring subsequent to the balance sheet date would require an adjustment to the financial statements before they could be issued? A) Sale of long-term debt or capital stock. B) Loss of a plant as a result of a flood. C) Major purchase of a business that is expected to double sales volume. D) Settlement of litigation, in excess of the previously recorded liability.

d

Which of the following situations would require adjustment to or disclosure in the financial statements? A) A merger discussion. B) The application for a patent on a new production process. C) Discussions with a customer that could lead to a 40 percent increase in the entity's sales if agreement is successful. D) The bankruptcy of a customer who regularly purchased 30 percent of the company's output

d

Which of the following subsequent events will be least likely to result in an adjustment to the financial statements? A) Culmination of events affecting the realization of accounts receivable owned as of the balance sheet date. B) Culmination of events affecting the realization of inventories owned as of the balance sheet date. C) Material changes in the settlement of liabilities that were estimated as of the balance sheet date. D) Material changes in the quoted market prices of listed investment securities since the balance sheet date.

d

The engagement partner and manager review the work of engagement team members to evaluate which of the following? a) The work was performed and documented. b) The objectives of the procedures were achieved. c) The results of the work support the conclusions reached. d) All of these are correct.

d) All of these are correct.

Which of the following describes the PCAOB generally accepted auditing standard requiring a critical review of the work done and the judgment exercised by those assisting in an audit at every level of supervision? a) Proficiency. b)Audit risk. c) Inspection. d) Due care.

d) Due care.

A written understanding between the auditor and the client concerning the auditor's responsibility for the discovery of illegal acts is usually set forth in a(n) a) Client representation letter. b) Letter of audit inquiry. c) Management letter. d) Engagement letter.

d) Engagement letter.

Which of the following best places the events of the last decade in proper sequence? a) Sarbanes-Oxley Act, increased consulting services to auditees, Enron and other scandals, prohibition of most consulting work for auditees, establishment of PCAOB. b) Enron and other scandals, Sarbanes-Oxley Act, increased consulting services to auditees, prohibition of most consulting work for auditees, establishment of PCAOB. c) Increased consulting services to auditees, Sarbanes-Oxley Act, Enron and other scandals, prohibition of most consulting work for auditees, establishment of PCAOB. d) Increased consulting services to auditees, Enron and other scandals, Sarbanes-Oxley Act, prohibition of most consulting work for auditees, establishment of PCAOB.

d) Increased consulting services to auditees, Enron and other scandals, Sarbanes-Oxley Act, prohibition of most consulting work for auditees, establishment of PCAOB.

Hawkins requested permission to communicate with the predecessor auditor and review certain portions of the predecessor auditor's working papers. The prospective client's refusal to permit this will bear directly on Hawkins' decision concerning the a) Adequacy of the preplanned audit program. b) Ability to establish consistency in application of accounting principles between years. c) Apparent scope limitation. d) Integrity of management.

d) Integrity of management.

Which of the following concepts are pervasive in the application of generally accepted auditing standards, particularly the standards of field work and reporting? a) Expected misstatement. b) Control risk. c) Internal control. d) Materiality and audit risk.

d) Materiality and audit risk.

As generally conceived, the audit committee of a publicly held company should be made up of a) Representatives of the major equity interests (preferred stock, common stock). b) The audit partner, the chief financial officer, the legal counsel, and at least one outsider. c) Representatives from the client's management, investors, suppliers, and customers. d) Members of the board of directors who are not officers or employees.

d) Members of the board of directors who are not officers or employees.

The auditor must be independent of the auditee unless a) The lack of independence does not influence his or her professional judgment. b) Both parties agree that the independence issue is not a problem. c) The lack of independence is insignificant. d) None of the above—the auditor cannot lack independence.

d) None of the above—the auditor cannot lack independence.

What organization is responsible for setting auditing standards for audits of publicly-traded companies in the U.S.? a) AICPA. b) FASB. c) GASB. d) PCAOB.

d) PCAOB.

An auditor was unable to obtain audited financial statements or other evidence supporting an entity's investment in a large foreign subsidiary. Between which of the following reports should the auditor choose? a) Adverse and unqualified with an explanatory/emphasis-of-matter paragraph added. b) Disclaimer and unqualified with an explanatory/emphasis-of-matter paragraph added. c) Qualified and adverse. d) Qualified and disclaimer.

d) Qualified and disclaimer.

Which of the following best describes the relationship between business objectives, strategies, processes, controls, and transactions? a) To achieve its strategies, a business formulates objectives and implements processes, which are carried out through the entity's information and internal control systems. Transactions are conducted to ensure that the processes are properly executed, captured, and processed. b) To achieve its business processes, a business formulates objectives, which are carried out through the entity's strategies. The entity's information and internal control systems must be designed to ensure that the entity's strategies are properly executed, captured, and processed. c) To achieve its objectives, a business formulates strategies to implement its transactions, which are carried out through business processes. The entity's information and internal control systems must be designed to ensure that the processes are properly executed, captured, and processed. d) To achieve its objectives, a business formulates strategies and implements processes, which are carried out through business transactions. The entity's information and internal control systems must be designed to ensure that the transactions are properly executed, captured, and processed.

d) To achieve its objectives, a business formulates strategies and implements processes, which are carried out through business transactions. The entity's information and internal control systems must be designed to ensure that the transactions are properly executed, captured, and processed.

An auditor obtains knowledge about a new client's business and its industry in order to a) Make constructive suggestions concerning improvements to the client's internal control. b) Develop an attitude of professional skepticism concerning management's financial statement assertions. c) Evaluate whether the aggregation of known misstatements causes the financial statements taken as a whole to be materially misstated. d) Understand the events and transactions that may have an effect on the client's financial statements.

d) Understand the events and transactions that may have an effect on the client's financial statements.

Eagle Company, a public company, had a computer failure and lost part of its financial data. As a result, the auditor was unable to obtain sufficient audit evidence relating to Eagle's inventory account. Assuming the inventory account is at least material, the auditor would most likely choose either: a) an unqualified opinion with no explanatory paragraph or an unqualified opinion with an explanatory paragraph. b) a qualified opinion or an adverse opinion. c) a qualified opinion with no explanatory paragraph or a qualified opinion with an explanatory paragraph. d) a qualified opinion or a disclaimer of opinion.

d) a qualified opinion or a disclaimer of opinion.

An independent audit adds value to the communication of financial information because the audit a) guarantees that financial data are fairly presented. b) confirms the exact accuracy of management's financial representations. c) assures the readers of financial statements that any fraudulent activity has been corrected. d) lends credibility to the financial statements.

d) lends credibility to the financial statements

On April 5, 2012, you discovered that, on February 16, 2012 a flood destroyed the entire uninsured inventory in one of Schmidt's warehouses.

d. Request the client revise and reissue the December 31, 2011 financial statements. The revision should involve the addition of a footnote, but no adjustment, to the December 31, 2011 financial statements.

Tests of __________ ____ __________ address whether financial statement discloses are properly presented

details of disclosures

Tests of ________ _____ _________ address whether particular types of transactions have been properly accounted for during the period.

details of transactions

Rather than "assessing" ____________ _____________, auditors seek to restrict it through the performance of ____________ _______________. Detection risks exists because the auditors' substantive procedures are _________ ______ __________, due to both sampling and other factors.

detection risk substantive procedures Not 100% effective

Less control risk means an increase in the risk of material misstatements

false

Rather than restrict detection risk though the performance of more substantive procedures, auditors assess it.

false

The risk of material misstatement is composed of the three components of audit risk (T/F)

fasle- IR and CR

extrapolate

form an opinion based on a set of facts

High inherent risk may be indicated by __________________________, operating results that are highly ______________________________, going concern problems, large known and likely ___________________________, substantial turnover, questionable reputation, or ________________________________.

inconsistent profitability sensitive to economic factors misstatements detected in prior audits inadequate accounting skills in management

Sampling Risk Type 1 definition

incorrect rejection, risk assessed too high. Type 1 causes inefficiency because if you have to test more to find out if the sample was just unlucky or not, it wastes time as opposed to if you had picked a good sample in the first place.

Creditor monthly statement obtained from client's files, duplicate sales invoices in filled order files, times tickets filed in payroll department, bank statement in client's files, letter in customers files form collection agency on collectability of balance

indirectly from outsiders

When the auditors express an opinion that the financial statements are free from material misstatements, they are relying upon the effectivenesses of the client's ______________ _________ to prevent such misstatement and their _________ _________ to detect any material misstatement that may have occurred

internal control audit procedures

The amount of evidence that is considered sufficient varies _____________ with the competence of the evidence

inversely

When should you use control reliance strategy?

many controls, effective controls

Inherent risks many be high for non-rountine transactions because they are ___________________________ of transactions and occur only periodically. Specialized skills may be needed to perform activities such as __________________, ______________, and __________________________.

not part of normal flow physical inventories calculating depreciation adjusting FS for foreign currencies gains/losses.

A nonpublic client has provided required supplementary information with its audited financial statements. The auditor's proper reporting responsibility includes: other-matter paragraph should be added to the audit report. A separate report should be issued on the required supplementary information. An adverse opinion on the required supplementary information. The required supplementary information should not be referred to.

other-matter paragraph should be added to the audit report.

Completeness (Presentation and Disclosure)

relates to whether all disclosures that should have been included in the financial statements have been included... no material disclosures have been OMITTED from the footnotes...

Authorization Assertion

relates to whether all transactions have been properly authorized. Ex: purchase of a new manufacturing facility should be approved by the board of directors.

A letter signed by offices of the client company at that auditors request which sets forth certain assertions about the company's financial position and operations known as a ________________ ___________. The term ____________ relates to the quantity of evidence to be obtained.

representation letter sufficient

Upper deviation rate

represents how high the true deviation rate might actually be

major accounts in revenue system

sales, ar, cash, inventory, bad debts, returns

Examples of routine transactions are ___________________________. They are routine in nature restricts ______________ risks.

sales, purchases, cash distruments, cash receipt, payroll inherent

2) Discuss the validity and limitations of inquiry and oral evidence

same as above: but in critical/important matters we should ask the client to confirm answers in writing

Is reperformance/recalculation a test of contorl or substantive test

test of control

What is attribute sampling

test of controls. used in order to estimate the proportion of a population that possesses a specified characteristic.

the most important factor in planning for a major change in a computer-based system is

the participation of top management in the planning process

Substantive Procedures (Audit Procedures for Obtaining Audit Evidence)

used to detect material misstatements at the relevant assertion level. Substantive procedures include tests of details and substantive analytical procedures.

Risk Assessment Procedures (Audit Procedures for Obtaining Audit Evidence)

used to obtain an understanding of the entity and its environment, including its internal control, to assess the risks of material misstatement at the financial statement and relevant assertion levels.

List two lists or requests that should be included in a standard "inquiry of attorney" letter.

• A list, prepared by management, of (1) pending threatened litigation and (2) asserted or unasserted claims or assessments with which the attorney has had significant involvement. An alternative is for the letter to request the attorney to prepare the list. • A request that the attorney furnish information or comment about the progress of each item listed, the legal action the client intends to take, the likelihood of an unfavorable outcome, and an estimate of the amount or range of the potential loss. • A request for the identification of any unlisted pending or threatened legal actions or a statement that the client's list is complete. • A statement by the client informing the attorney of his or her responsibility to inform management whenever in the attorney's judgment there is a legal matter requiring disclosure in the financial statements. The letter of inquiry should also request the attorney to respond directly to the auditor that he or she understands this responsibility. • A request that the attorney identifies and describes the nature of any reasons for any limitations in the response.

Besides the search for contingent liabilities and the review for subsequent events, the auditor has four important final evidence accumulation responsibilities, all of which are required by current professional auditing standards. Discuss each of these four responsibilities.

• Final analytical procedures performed as a final review for material misstatements or financial problems and to help the auditor take a final objective look at the financial statements. • Evaluate the going concern assumption. • Obtain a client representation letter documenting management's most important oral representations during the audit. • Consider supplementary information included in published annual reports pertaining directly to the financial statements.

LIST four of the procedures commonly used to search for contingent liabilities.

• Inquire of management (orally and in writing) about the possibility of unrecorded contingencies. • Review current and previous years' internal revenue agent reports for income tax settlements. • Review the minutes of directors' and stockholders' meetings for indications of lawsuits or other contingencies. • Analyze legal expense for the period under audit, and review invoices and statements from legal counsel for indications of contingent liabilities. • Obtain a letter from each major attorney performing legal services for the client as to the status of pending litigation or other contingent liabilities. • Review audit documentation for any information that may indicate a potential contingency. • Examine letters of credit in force as of the balance sheet date and obtain a confirmation of the used and unused balances.


Related study sets

3.3 Economic Integration and Terms of Trade

View Set

Mergers, Corporate Governance, and Control Topic 8

View Set

Chapter 7 Production and Growth - Macro

View Set

Chapter 2: Global E-business and collaboration

View Set